Download Morphology, physiology of microorganisms. Virology

Document related concepts

Infection wikipedia , lookup

Social history of viruses wikipedia , lookup

Quorum sensing wikipedia , lookup

Molecular mimicry wikipedia , lookup

Biofilm wikipedia , lookup

Skin flora wikipedia , lookup

Plant virus wikipedia , lookup

Phospholipid-derived fatty acids wikipedia , lookup

Trimeric autotransporter adhesin wikipedia , lookup

Henipavirus wikipedia , lookup

Introduction to viruses wikipedia , lookup

Hepatitis B wikipedia , lookup

Hospital-acquired infection wikipedia , lookup

Bacteria wikipedia , lookup

Infection control wikipedia , lookup

Microorganism wikipedia , lookup

Virus quantification wikipedia , lookup

Triclocarban wikipedia , lookup

Virology wikipedia , lookup

Human microbiota wikipedia , lookup

History of virology wikipedia , lookup

Bacterial taxonomy wikipedia , lookup

Marine microorganism wikipedia , lookup

Bacterial cell structure wikipedia , lookup

Bacterial morphological plasticity wikipedia , lookup

Transcript
☰





Search
Explore
Log in
Create new account
Upload
×
Module 1 Morphology and physiology of microorganisms. Infection and Immunity.
General and special virology.
Text test questions
1. By laboratory diagnostics of spinal fluid under the microscope gramnegative diplococci
into leucocytes (incompleted phagocytosis) were found. What these
microorganisms
are?
A. Staphylococci
B. Monococci
C. Pneumococci
D. Streptococci
E. *Meningococci
2. By laboratory diagnostics of pure culture under the microscope spherical
microbes
which are located as a cluster in smear were obtained. What microorganisms
were
found?
A. Diplococci.
B. Micrococci.
C. Streptococci.
D. Tetracocci.
E. *Staphylococci.
3. After laboratory research of sputum of patient with pneumonia grampositive diplococci
were isolated. What microorganisms found out in a sputum?
A. Staphylococcus aureus
B. Streptococcus pyogenes
C. Streptococcus agalactiae
D. Staphylococcus intermedius
E. *Streptococcus pneumoniae
4. By microscopic examination of pus grapeshaped grampositive cocci were
found.
Specify what family these microorganisms belong to.
A. Planococcus
B. Enterococcus
C. Streptococcus
D. Micrococcus
E. *Staphylococcus
5. After microscopic examination of sputum lancet-shaped gram-positive
microorganisms
were found. What these microorganisms are?
A. Gonococci
B. Meningococci
C. Micrococci
D. Staphylococci
E. *Pneumococci
6. By the bacteriological examination of kindergarten workers bacteria of
Staphylococcus
family were isolated from one of them. What morphological and tinctorial
properties do
these microbes have?
A. tetrads of Gram-positive cocci
B. Gram-positive diplococci
C. Gram-positive cocci in chains
D. Gram-positive rods
E. grape-shaped Gram-positive cocci
7. After the conducted surgical operation a patient has got a sepsis as
complication which
caused by staphylococci. Which of following properties have staphylococci?
A. in a smear look like pair
B. single gram-negative rods
C. sporulate, does not aniline stained
D. gram-positive, in chains
E. *gram-positive, in irregular groups
8. At microscopic examination of material from patient with croupous
pneumonia lancetshaped diplococci, surrounded a general capsule were found.
Which microbes were
found?
A. S.pyogenes
B. S.saprophyticus
C. S.haemolyticus
D. S.epidermidis
E. *S.pneumoniae
9. At darkfield microscopic examination of material from the surface of the
pudendal
labium ulcer of patient T., 18 y.o,. were found thin spiral microorganisms.
What these
microorganisms are?
A. Treponema refringens
B. Treponema phagedenis
C. Treponema denticola
D. *Treponema pallidum
E. Treponema dentium
10. At the bacteriologic examination of feces the culture of Shigella
Flexneri is isolated.
What is morphology of these microorganisms?
A. in a smear look like pair
B. sporulate
C. gram-positive, in a smear look like chains
D. gram-positive, in a smear look like groups
E. *single gram-negative rods
11. At the bacteriologic examination of festerings discharge from an urethra
Gram-negative
coccal bacteria disposed in leucocytes were found. Which disease can cause
this
pathogen?
A. syphilis
B. scarlet fever
C. meningococcal infection
D. chancroid
E. *gonorrhoea
12. At the bacteriologic examination of pathological material from a patient
with
furunculosis, spherical gram-positive microorganisms which are located as a
cluster are
isolated. What are microorganisms?
A. Diplococci.
B. Micrococci.
C. Streptococci.
D. Tetracocci.
E. *Staphylococci.
13. Bacteria of Staphylococcus family were isolated from pus of wound. What
morphology
and appearance in a smear belong to this family?
A. in a smear look in pair
B. single gram-negative rods
C. sporulate, does not perceive aniline stain
D. gram-positive, in chains
E. *gram-positive, in irregular groups
14. At the bacteriological examination of healthy man from an oral cavity the
culture of
gram-positive diplococci with a capsule were isolated. The carrier of what
pathogenic
microorganism is this man?
A. Legionelia pneumoniae
B. Klebsiella pneumoniae
C. Streptococcus pyogenes
D. Streptococcus agalactiae
E. *Streptococcus pneumoniae
15. At a microscopy of a pus from a furuncle staphylococci are found. What
morphological
and tinctorial properties do these microbes have?
A. single Gram-positive cocci
B. Gram-positive diplococci
C. Gram-positive cocci in chains
D. Gram-positive rods
E. *grape-shaped Gram-positive cocci
16. From the material (an uterus discharge) two smears were prepared. At
bacteriologic
examination plenty of leucocytes and lancet-shaped diplococci which were into
leucocytes found out. What these microorganisms are?
A. Staphylococci
B. Pneumococci
C. Meningococci
D. Enterococci
E. *Gonococci
17. Boy, 7 years old, has vomiting, acute diarrhoea. The previous diagnosis
was cholera.
But after inoculation of feces on Endo medium dark red colonies with metallic
shine
grow. Under the microscope they are gram-negative, small, straight rods. What
these
microbes are?
A. Vibrio cholerae
B. Pseudomonas spp.
C. Shigella spp.
D. Salmonella spp.
E. *Escherichia coli
18. In a maternity hospital an examination is conducted on an in-hospital
infection.
Staphylococcus aureus was found on some objects. What properties belong to
this
microbe?
A. in a smear look like pair
B. single gram-negative rods
C. sporulate, does not perceive aniline stain
D. gram-positive, chains
E. *gram-positive, groups
19. In a school some cases of food poisoning are registered. The examination
of stuff and
students is conducted. Gram positive microorganisms which are located in
pair, short
chains or as clasters are found. What these microorganisms are?
A. Pneumococci
B. Staphylococcus
C. Micrococci
D. Spiral
E. *Enterococci
20. In kindergarten the bacteriological examination of children and stuff is
conducted to
обнаружить career of meningococci. What are morphological properties of
Meningococcus spp.?
A. look as a tetrads
B. look as a chain
C. look as a single coccus
D. *look as pairs
E. look as a grape
21. Pyogenic inflammation processes are caused by staphylococci in most
cases. Which
properties they have?
A. gram-negative in pair in a smear
B. single gram-negative rods
C. sporulate, does not perceive aniline stain
D. gram-positive in chains
E. *gram-positive in irregular groups
22. In a wound pus smear microorganisms of purple color, located in short
chains were
observed. What these microorganisms are?
A. Staphylococcus
B. Gonococci
C. Micrococci
D. Spiral
E. *Streptococci
23. On the top of fever patient’s blood is taken for microscopic examination.
Blood smear
was prepared and stained by Romanovsky-Giemsa method. At the microscopy dark
blue
thin microorganisms with 3-4 large curves were found among blood cells. What
these
microorganisms are?
A. Treponema pallidum
B. Leptospira interrogans
C. Borrelia duttoni
D. Borrelia burgdorferi
E. *Borrelia recurrentis
24. The bacteriologic examination of patient’s material taken from a girl
with a scarlet fever
is performed. Gram-positive spherical microorganisms were isolated. What
these
microbes are?
A. Vibrio cholerae
B. Staphylococcus
C. Proteus
D. Meningococcus
E. *Streptococcus
25. The first cases of acute intestinal disease, which was accompanied a
frequent liquid
stool (to 25 times per days) which reminded a "rice-water", vomiting,
dehydration of
organism, are registered in port town. The pure culture of microorganisms is
isolated.
There were gram-negative, small, slightly cuvered rods, mobile. What these
microbes
are?
A. Pseudomonas spp.
B. Escherichia coli
C. Shigella spp.
D. Salmonella spp.
E. *Vibrio cholerae
26. The microscopic examination of pus found out staphylococci. What
morphological
properties of this family?
A. look as a tetrads
B. look as a chain
C. look as a single coccus
D. look as pairs
E. *look as a grape
27. The microscopic examination of wound discharge found out staphylococci.
What
morphological properties of this family?
A. look as a tetrads
B. look as a chain
C. look as a single coccus
D. look as pairs
E. *look as a grape
28. The mother of one-month child complains about festerings discharge from
the
conjunctiva of child’s eyes. Plenty of leucocytes and coffe-shaped diplococci
which
were into leucocytes discovered at the microscopy of discharge. What these
microorganisms are?
A. Neisseria flava
B. Neisseria sicca
C. Neisseria meningitides
D. Neisseria subflava
E. *Neisseria gonorrhoeae
29. The pure culture of some pathogen is isolated. Biochemical identification
rotined that a
it is Escherichia coli. What morphological and tinctorial properties do this
microorganism have?
A. Gram-negative cocci
B. Gram-positive cocci
C. Gram-positive rods
D. Gram-negative spiral bacteria
E. *Gram-negative rods
30. Vomiting, diarrhoea, high temperature appeared for a 1,5-years-old child.
Feces were
inoculated on the Endo medium. At a microscopy found out gram-negative rods.
What
is the pathogen causing this disease?
A. Vibrio cholerae
B. Staphylococcus
C. Streptococci
D. Gonococci
E. *Escherichia coli
31. Ziehl-Neelsen staining of sputum of a patient with tuberculosis is
prepared. What
microscopic picture confirmes the diagnosis?
A. Threadlike microorganisms
B. Spiral microorganisms
C. Rod-like microbes in chains
D. Coccal microorganisms
E. *Thin red rods
32. "Parfocal" refers to microscopes with multiple objectives where
A. objectives are used in pairs for stereoscopic effects.
B. each objective has the same working distance above the specimen.
C. sequential objectives increase power by a factor of two.
D. All are true
E. *each objective is positioned to be in focus at the same stage height.
33. A 40X objective and an 10X ocular produce a total::п»їA 40X objective and
an 10X
ocular produce a total magnification of
A. 4
B. 50
C. 90
D. 120
E. *400
34. A 40X objective and an 10X ocular produce a total ::A 40X objective and
an 10X ocular
produce a total magnification of
A. 50
B. 90
C. 140
D. 30
E. *400
35. A bacillus bacterium with a single flagellum at each end is described as
A. Monotrichous
B. Lophotrichous
C. Peritrichous
D. Atrichous
E. *Amphitrichous
36. A microscope in which an image is formed by passing an electron beam
through a
specimen and focusing the scattered electrons with magnetic lenses is called
a
A. scanning electron microscope
B. phase-contrast microscope
C. light microscope
D. optical microscope
E. *transmission electron microscope
37. Agostino Bassi first showed that a disease, in silkworms, was caused by
microorganism.
What kind of microorganism caused the disease:
A. Virus
B. Bacterium
C. Protozoan
D. Prion
E. *Fungus
38. Among these bacteria gram-positives there are all, EXCEPT FOR:
A. Staphylococci
B. Streptococci
C. Clostridium botulini
D. Bacillus anthracis
E. *Meningococci
39. Another common bacterial shape is that of a rod, often called
A. coccus
B. pleomorphic
C. vibrio
D. spirochete
E. *bacillus
40. Another common bacterial shape is that of a rod, often called
A. coccus
B. pleomorphic
C. spiral
D. pole
E. *bacillus
41. At staining by Gram's method after flood slide with the iodine solution
preparation is
treated by:
A. Fuchsine
B. Washing with water
C. Crystal violet
D. Sulphuric acid
E. *Alcohol
42. Bacteria accomplish chemotaxis by
A. Steering toward better growth conditions.
B. Frequently stopping and tumbling to better sense good conditions.
C. Stopping movement when conditions are good.
D. All are true
E. *Making long, uninterrupted runs when conditions are good.
43. Bacteria do not always swim aimlessly but are attracted by such nutrients
such as sugar
and amino acids, and are repelled by harmful substances and bacterial waste
products.
Movement toward chemical attractants and away from repellents is called
A. gliding motility.
B. tumbling.
C. spinning
D. creeping
E. *chemotaxis.
44. Bacterial genus/genera of medical importance which produce endospores
is/are:
A. Bacterium.
B. Corynebacterium
C. Mycobacterium
D. Micrococcus
E. *Bacillus.
45. Cell wall of gram-positive bacteria consist of:
A. thin monolayer peptidiglycan
B. lipoproteins
C. lipopolysaccharides
D. outer membrane
E. *polilayer peptidiglican
46. Cells that do not have a membrane separating their nuclear material from
their
cytoplasm are called
A. heterozygous.
B. homozygous.
C. eukaryotic.
D. viruses.
E. *prokaryotic
47. Choose rod-shaped bacteria:
A. Meningococcus
B. Vibrio
C. Borrelia
D. Sarcina
E. *Clostridium
48. Credit for the first vaccine for the prevention of human disease is
generally given to:
A. Louis Pasteur for the prevention of rabies.
B. Louis Pasteur for the prevention of anthrax.
C. Robert Koch for the prevention of tuberculosis.
D. Salk for the prevention of polimyelitis.
E. *Edward Jenner for the prevention of small pox.
49. Differential Interference Contrast microscopy
A. compares two identical specimens on the same microscope.
B. illuminates the specimen with light of two different colors.
C. illuminates the specimen with both reflected and transmitted light.
D. illuminates the specimen with light in two different ways.
E. *illuminates the specimen with light of two different phases.
50. Each of the following statements concerning the Gr::Each of the following
statements
concerning the Gram stain is correct EXCEPT:
A. Escherichia coli stains pink because it has a thin peptidoglycan layer
B. Streptococcus pyogenes stains blue because it has a thick peptidoglycan
layer
C. Mycoplasma pncumoniae isn't visible in the Gram stain because it doesn't
have
a cell wall
D. No true answer
E. *Mycobacterium tuberculosis stains blue because it has a thick lipid layer
51. Fimbriae
A. cause bacteria move through fluids.
B. sense changes in nutrient concentration.
C. are pathways for the secretion of exoenzymes.
D. are pathways for the nutrition.
E. *attach bacteria to various surfaces.
52. Fixation is the process by which the structures of the cells are
preserved and fixed in
position. An advantage of chemical fixation over heat fixation is that it
A. magnifies the specimen
B. uses simple chemical substances
C. can make during short time
D. kills bacteria
E. *does not destroy internal structures
53. Gram positive cells
A. have a second, outer membrane that helps retain the crystal violet stain.
B. have a thick capsule that traps the crystal violet stain.
C. have inclusions that traps crystal violet stain.
D. have monolayer of peptidoglycan that traps crystal violet stain
E. *have multiple layers of peptidoglycan that help retain the crystal violet
stain.
54. Gram positive cells
A. have thick, homogeneous cell walls.
B. have large amounts of teichoic acids.
C. do not have an outer membrane.
D. do not have LPS
E. *all of the above are true.
55. Choose the correct answer/s about Gram positive cells
A. have thick, homogeneous cell walls.
B. have large amounts of teichoic acids.
C. do not have an outer membrane.
D. do not have an LPS.
E. *all answers are true.
56. Gram positive cells
A. have a second, outer membrane that helps retain the crystal violet stain.
B. have a thick capsule that traps the crystal violet stain.
C. have a periplasmic space that traps the crystal violet.
D. have one layer of peptidoglycan that help retain the crystal violet stain.
E. *have multiple layers of peptidoglycan that help retain the crystal violet
stain.
57. How does a bacterium control the direction of swim::How does a bacterium
control the
direction of swimming?
A. The length of the flagellum acts as a rudder to steer the bacterium.
B. The speed of rotation is faster when the bacterium is headed the correct
way.
C. The bacterium can stop and spin until it is pointed the correct way.
D. All are true
E. *The bacterium does not control the direction of its swimming.
58. In bacterial cells, ribosomes are packed into the cytoplasmic matrix and
also loosely
attached to the plasma membrane. What is the function of ribosomes?
A. Site of energy production
B. Site of genetic reproduction
C. Site of lipid synthesis
D. Site of polysaccharide synthesis
E. *Site of protein synthesis
59. Indicate gram-negative bacteria.
A. Neisseria and Treponema
B. Salmonella and Shigella
C. Vibrio and Spirocheta
D. No correct answer
E. *All answers are correct
60. Indicate gram-negative bacteria.
A. Clostridium botulinum
B. Bacillus anthracis
C. Micrococcus luteus
D. Staphylococcus epidermidis
E. *Neisseria meningitidis
61. Indicate gram-positive bacteria.
A. Escherichia coli
B. Neisseria meningitidis
C. Vibrio cholerae
D. Treponema palloidum
E. *Staphylococcus aureus
62. Indicate gram-positive bacteria.
A. Vibrio
B. Borrelia
C. Neisseria
D. Salmonella
E. *Streptococcus
63. Indicate gram-positive bacteria.
A. Salmonella and Shigella
B. Vibrio and Spirocheta
C. Borrelia and Leptospira
D. Neisseria and Treponema
E. *Micrococcus and Tetracoccus
64. Indicate gram-positive bacteria.
A. Neisseria and Treponema
B. Vibrio and Spirocheta
C. Borrelia and Leptospira
D. Salmonella and Shigella
E. *Bacillus and Clostridium
65. Living, unstained cells and organisms can be observed best using
A. fluorescent microscopy
B. transmission electron microscopy
C. Scan. Electron microscopy
D. light microscopy
E. *phase contrast microscopy
66. Living, unstained cells and organisms can be observed best using
A. fluorescent microscopy
B. TEM (transmission electron microscopy)
C. binocular magnifier
D. hand magnifier
E. *phase contrast microscopy
67. Louis Pasteur's studies on the unwanted production of acid from beet
sugar was the first
demonstration that
A. sugars are unstable and can breakdown into either ethanol or acid.
B. ethanol is unstable and can convert to acid.
C. microorganisms can be found in air.
D. microorganisms can be found in air.
E. *bacteria can cause specific chemical reactions.
68. Manner similar to human cells
A. Bacteria are prokaryotic (ie, they have one molecule of DNA, are haploid,
and
have no nuclear membrane), whereas human cells are eukaryotic (ie, they have
multiple chromosomes, are diploid, and have a nuclear membrane)
B. Bacterial and human ribosomes are of different sizes and chemical
compositions
C. Bacterial cells possess peptidoglycan, whereas human cells do not
D. No correct answer
E. *Bacteria derive their energy by oxidative phosphorylation within
mitochondria
in a
69. Mark rod-shaped bacteria:
A. Vibrio
B. Treponema
C. Borrelia
D. Staphylococcus
E. *Escherichia coli
70. Mark rod-shaped bacteria:
A. Sarcina
B. Spirilla
C. Streptococcus
D. Neisseria
E. *Mycobacterium
71. Monochromatic (one color) light is sometimes used to increase the
resolution of light
microscopes. Light of which color below would give you the best resolution?
A. Red
B. Orange
C. Green
D. Yellow
E. *Blue
72. Light of which color below would give you the best resolution of light
microscopes?
A. Red
B. Orange
C. Green
D. Brown
E. *Blue
73. Morphological classification of rod-shaped bacteria:
A. monobacilli and monobacteria
B. diplobacteria and diplobacilli
C. streptobacteria and streptobacilli
D. No correct answer
E. *all answers are correct
74. Morpholohgical classification of spherical bacteria:
A. monococci and diplococci
B. streptococci and staphylococci
C. tetracocci and sarcina
D. No correct answer
E. *all answers are correct
75. Phase Contrast microscopy::Phase Contrast microscopy
A. Continuously changes the phase of the incident light from the condenser to
improve contrast in the specimen.
B. Uses special lenses to distinguish between solid and liquid phases of the
cell.
C. Uses special lenses to change the color of light passing through them.
D. Can examinate only fixing objects.
E. *Uses circular filters in the condenser and objective to give contrast to
parts of
the cell with different refractive indices.
76. Phase Contrast microscopy
A. Continuously changes the phase of the incident light from the condenser to
improve contrast in the specimen.
B. Uses special lenses to distinguish between solid and liquid phases of the
cell.
C. Uses special lenses to change the color of light passing through them.
D. Uses fixed and stained smears.
E. *Uses circular filters in the condenser and objective to give contrast to
parts of
the cell with different refractive indices.
77. Plasmids are important to the genetics of many bacteria. This is because
A. they are inherited from one generation to the next.
B. they may carry genes that give their host a selective advantage.
C. they can render bacteria drug-resistant.
D. All are false
E. *all answers are correct
78. Poly-beta-hydroxybutyrate inclusion bodies
A. protect bacteria from excessive drying.
B. turn reddish brown when stained with iodine.
C. are composed of polymers of glucose.
D. protect bacteria from heating.
E. *store carbon for energy and biosynthesis.
79. Resolution is the ability of a lens to distinguish between small objects
close together.
What approximate resolution can be obtained with a light microscope
A. 0.3 nm
B. 0.5 nm
C. 0.8 nm
D. 1,0 nm
E. *0.1 nm
80. Scanning electron microscopy (SEM) is best used to study
A. small internal cell structures.
B. cytoplasma
C. nucleoid
D. all are true.
E. *surface morphology.
81. Show diplococci:
A. Vibrio
B. Treponema
C. Borrelia
D. Clostridium
E. *Neisseria
82. Some bacteria are considered pleomorphic. This means:
A. they are shaped like bent rods.
B. they have a corkscrew shape.
C. they are not either bacilli or cocci.
D. they are not either vibrio or spirochete
E. *they do not have just one shape.
83. Some bacteria are considered pleomorphic. This means
A. they are shaped like bent rods.
B. they have a corkscrew shape.
C. they are not either bacilli or cocci.
D. they are comashaped.
E. *they do not have just one shape.
84. Spherical bacteria arranged in a chain are known as
A. spirochetes
B. bacilli
C. staphylococci.
D. streptobacilli
E. *streptococci
85. Syphilis is caused by which of the following?
A. Helicobacter pylori
B. Proteus mirabilis
C. Serratia marcesans
D. Treponema vincentii
E. *Treponema pallidum
86. The 70S procaryotic ribosomes consist of
A. two 40S subunits.
B. a 40S and a 30S subunit.
C. a 50S and a 20S subunit.
D. two 30S subunits.
E. *a 50S and a 30S subunit.
87. The cell wall of Gram-positive bacteria is compos::The cell wall of Grampositive
bacteria is composed primarily of
A. chitin.
B. cellulose,
C. starch.
D. protein.
E. *peptidoglycan.
88. The construction and use of the compound microscope is attributed to:
A. Louis Pasteur.
B. Robert Koch.
C. Ferdinand Cohn.
D. Paul Ehrlich.
E. *Antony van Leeuwenhoek.
89. The first observation that bacteria-like organisms could be found in
normal air was by
A. Anton Leeuwenhoek
B. Robert Koch
C. Joseph Meister
D. Edward Jenner
E. *Louis Pasteur
90. The first person to see bacteria did his work mostly in what century?
A. 14th
B. 16th
C. 18th
D. 19th
E. *17th
91. The first physician to make practical application of the germ theory of
disease to surgery
was
A. Louis Pasteur
B. Robert Koch
C. Edward Jenner
D. Anton von Leeuwenhoek
E. *Joseph Lister
92. The first techniques of sterilization were introdu::The first techniques
of sterilization
were introduced by:
A. Robert Koch.
B. Ferdinand Cohn.
C. John Needham.
D. Gerhardt Domagk.
E. *Louis Pasteur.
93. The flagellar filament grows by
A. enzymatic addition of subunits to the tip of the flagellum.
B. enzymatic addition of subunits to the base of the flagellum.
C. self-insertion of subunits along the length of the filament.
D. no correct answer
E. *self-assembly of subunits traveling through the hollow flagellum to the
tip.
94. The form of coffee corn is had:
A. staphylococcus and streptococcus
B. pneumococcus and monococcus
C. peptococcus and peptostreptococcus
D. tetracoccus and sarcina
E. *meningococcus and gonococcus
95. The fourth stage of making of a staining smear is:
A. Fixation
B. Washing
C. Drying
D. Preparation of smear
E. *Staining
96. The major defining difference(s) between a PROKARYOTIC and a EUKARYOTIC
cell is/are:
A. The absence of membrane-bound organelles in eukaryotes.
B. The lack of photosynthesis by prokaryotic cells.
C. The use of ATP as an energy source by the more complex eukaryotic cells.
D. The absence of chromosomes in prokaryotes.
E. *The absence of membrane-bound organelles in prokaryotes.
97. The most commonly encountered bacteria are roughly spherical. The
microbiological
term describing this shape is
A. bacillus
B. pleomorphic
C. vibrio
D. spirochete
E. *coccus
98. The most commonly encountered bacteria are roughly spherical. The
microbiological
term describing this shape is
A. bacillus
B. pleomorphic
C. ball
D. sphere
E. *coccus
99. The most important innovation (new idea) in Pasteur's 'swan neck flask'
experiments
was\:
A. a glass barrier prevented contamination.
B. heating media prevented microbial growth.
C. the experimenter could look for contamination without disturbing the
experiment.
D. All are true
E. *fresh air could directly contact the medium.
100.
The most important role of the prokaryotic cell wall is to
A. maintain the shape of the cell.
B. prevent ions from diffusing away from the cell.
C. block the effects of antibiotics like penicillin.
D. no correct answer
E. *protect the cell from osmotic pressures.
101.
Choose the correct answer about the substage condenser:
A. changes the wavelength of the light reaching the specimen
B. decreases the amount of light reaching the specimen
C. increases the amount of light reaching the specimen
D. no correct answer
E. *focuses light on the specimen
102.
The original distinction between the two types of cells, prokaryotic and
eukaryotic, was made on the basis of the
A. structure of the cell wall
B. absence or presence of mitochondria.
C. absence or presence of ribosomes
D. structure of the cell membrane.
E. *absence or presence of a nuclear membrane.
103.
The outer membrane of Gram negative cells is more permeable than the plasma
membrane because
A. LPS is larger than most membrane phospholipids.
B. lipoproteins stretch the outer membrane.
C. the core polysaccharide spans the lipid bilayer.
D. it contains LPS.
E. *porin proteins establish holes in the outer membrane.
104.
The presence of D-amino acids in the crosslinks of::The presence of D-amino
acids in the crosslinks of the peptidoglycan layer is most likely because
A. D-amino acids fit the structural constrains of the cell wall better than
L-amino
acids.
B. most L-amino acids have already been used for protein synthesis.
C. D-amino acids are easier to crosslink in the absence of ribosomes.
D. All are true
E. *most peptidases can only cleave L-amino acids.
105.
The primary use of Koch's postulates is to
A. clearly identify and characterize a particular microorganism.
B. isolate microorganisms from diseased animals.
C. develop vaccines for specific diseases.
D. environment suround the diseased animals.
E. *demonstrate that a disease is caused by a microorganism.
106.
The role of antibodies in fighting disease was first demonstrated by:
A.
B.
C.
D.
E.
vaccination of humans with rabies.
attenuation of rabies by passage in atypical host.
observation of phagocytosis of bacteria.
no correct answer
*injection of rabbit "antitoxin" to protect against diphtheria.
107.
The role of blood cells in fighting disease was first demonstrated by
A. Pasteur with his swan necked flasks.
B. Koch with acid fast staining of mycobacteria.
C. Chamberland with his filtration of virus through porcelain.
D. no correct answer
E. *Metchnikoff with his observation of phagocytosis.
108.
The significance of the plasma membrane is that
A. it selectively allows some molecules to pass into the organism
B. it prevents movement of molecules out of the organism
C. it is the site of protein synthesis
D. no correct answer
E. *all answers are correct
109.
The substage condenser
A. changes the wavelength of the light reaching the specimen
B. decreases the amount of light reaching the specimen
C. eluminate the specimen
D. magnificates size of specimen
E. *focuses light on the specimen
110.
The third amino acid in the peptidoglycan crosslinking chain is either
diaminopimilic acid or lysine because this amino acid must
A. be positively charged for a salt bridge to form.
B. be hydrophillic.
C. have a large R-side chain to fill space in the cell wall.
D. no correct answer
E. *have a free amino group for peptide bond formation.
111.
The third stage of making of a staining smear is:
A. Washing
B. Staining
C. Drying
D. Preparation of smear
E. *Fixation
112.
To make a vaccine against chicken cholera that would not kill the chicken,
Pasteur
A. treated the sample with heat to kill the microorganisms.
B. used a related but different microorganism from animals.
C. used very small, non-lethal amounts of material.
D. no correct answer
E. *attenuated the strain by repeatedly passaging it in culture.
113.
Transmission electron microscopy is best for high magnification viewing of
A. internal structure of live, motile cells.
B. surface structure of fixed cells.
C. surface membranes of live, motile cells.
D. no correct answer
E. *internal structure of fixed cells.
114.
Typical drumstick appearance of bacilli is observed in:
A. Clostridium perfringens.
B. Clostridium tetani.
115.
116.
117.
118.
119.
120.
121.
122.
C. Clostridium histolyticum.
D. Clostridium difficilae
E. *Clostridium botulinum.
What bacteria have the spiral form:
A. staphylococcus
B. streptococcus
C. bacillus
D. sarcina
E. *spirillum
What disease do Streptococci cause?
A. Epidemic cerebrospinal meningitidis
B. Enteric fever
C. Diphtheria
D. Flue
E. *Scarlet fever
What disease do Streptococci cause?
A. Typhoid fever
B. Mumps
C. Epidemic cerebrospinal meningitidis
D. Tuberculosis
E. *Rheumatic fever
What disease does Neisseria meningitidis cause?
A. Rheumatic fever
B. Scarlet fever
C. Cholera
D. Nodamura disease
E. *Epidemic cerebrospinal meningitidis
What does Kingdom Prokaryotae include:
A. protozoa.
B. fungi.
C. viruses
D. all answers are correct.
E. *bacteria.
What is a compound microscope?
A. A microscope that has one lenses.
B. A microscope whose lenses are concave.
C. A microscope whose lenses are convex.
D. No correct answer
E. *A microscope that has two sets of lenses: an ocular lens and an eyepiece.
What is a microorganism?
A. A microorganism is a small organism that takes in and breaks down food for
energy and nutrients, excretes unused food as waste, and is capable of
reproduction.
B. A microorganism is a small organism that causes diseases only in plants.
C. A microorganism is a small organism that causes diseases only in animals.
D. No correct answer
E. *A microorganism is a term that refers to a cell.
What is a pathogenic microorganism?
A. A microorganism that multiplies
B. A microorganism that grows in a host
C. A microorganism that is small
D. No correct answer
123.
124.
125.
126.
127.
128.
129.
130.
E. *A disease-causing microorganism
What is a plasmid?
A. Self-replicating segment of single stranded RNA
B. A bacterial chromosome
C. A nucleoid
D. An inclusion
E. *Self-replicating segment of double stranded DNA
What is a smear?
A. A smear is a preparation process in which a specimen is dyed.
B. A smear is a process in which a specimen is moved beneath a microscope.
C. A smear is a process used to identify a specimen.
D. No correct answer
E. *A smear is a preparation process in which a specimen is spread on a
slide.
What is Archaea?
A. Archaea is a classification for organisms that have two nuclei.
B. Archaea is a classification for organisms that use phagocytosis.
C. Archaea is a classification of an organism that identifies prokaryotes
that have
peptidoglycan cell walls.
D. Archaea is a classification of Protozoa.
E. *Archaea is a classification of an organism that identifies prokaryotes
that do not
have peptidoglycan cell walls.
What is NOT a characteristic of prokaryotic organisms?
A. the presence of a cell membrane
B. "naked" DNA molecule
C. the presence of cytoplasm
D. No correct answer
E. *the presence of a nuclear membrane
What is the function of an illuminator?
A. To control the temperature of the specimen
B. To keep the specimen moist
C. To keep the specimen dry
D. No correct answer
E. *An illuminator is the light source used to observe a specimen under a
microscope
What is the purpose of bacterial endospores?
A. Allow the bacterium to make hundreds of "seeds" to spread on the wind.
B. Help the bacterium to differentiate into faster growing stages of
bacteria.
C. Allow the bacterium to survive the absence of oxygen.
D. Allow the bacterium to survive the absence of CO2.
E. *Allow the bacterium to survive extended periods of heat or dryness.
What microbes do belong to coccal forms?
A. Escherichia coli
B. Mycobacterium tuberculosis
C. Yersinia pestis
D. No the correct answer
E. *Staphylococcus aureus
What microbes do belong to coccal forms?
A. Salmonella
B. Leptospira
C. Yersinia
D. Clostridia
E. *Streptococcus
131.
A.
B.
C.
D.
E.
132.
A.
B.
C.
D.
E.
133.
A.
B.
C.
D.
E.
134.
A.
B.
C.
D.
E.
135.
A.
B.
C.
D.
E.
136.
A.
B.
C.
D.
E.
137.
A.
B.
C.
D.
E.
138.
A.
B.
C.
D.
E.
139.
A.
B.
C.
What microbes do belong to coccal forms?
Bordetella
Treponema
Vibrio
Shigella
*Micrococcus
What microbes do belong to coccal forms?
Borrelia
Corynebacteria
Proteus
Spirilla
*Neisseria
What microbes do belong to spiral forms?
Neisseria
Borrelia
Corynebacteria
Proteus
*Spirilla
What microbes do belong to spiral forms?
Bordetella
Micrococcus
Sarcina
Shigella
*Treponema
What microbes do belong to spiral forms?
Salmonella
Yersinia
Clostridia
Streptococcus
*Leptospira
What microbes do belong to spiral forms?
Bordetella
Clostridia
Micrococcus
Shigella
*Vibrio
What shape is not peculiar for bacteria:
round or oval
rod or stick
spiral
No correct answer
*icosaedral
What was the first bacterium shown to cause human disease?
Mycobacterium
Diphtheria
Streptococcus
Staphylococcus
*Anthrax
What was the first successful solid medium for colony purification of
bacteria?
Agar
Gelatin
Meat
D. Apple
E. *Potato
140.
What was the first virus shown to cause disease?
A. Polio
B. Hepatitis
C. Potato blight
D. Influenza
E. *Tobacco mosaic virus
141.
When Louis Pasteur first tried his vaccine on a young boy, there was a
possibility that the vaccine itself could kill the child. This was
permissible under the
standards of the day because
A. the importance of the Science was worth the risk.
B. less value was placed on human life than today.
C. the child was from a poor family.
D. No correct answer
E. *the child had rabies, which was always fatal.
142.
Which of the following bacteria is cell wall deficient?
A. Treponema.
B. Slaphylococcus.
C. Klebsiella.
D. Salmonella
E. *Mycoplasma.
143.
Which of the following components are not found in the Gram-positive cell
wall?
A. peptidoglycan
B. teichoic acid
C. N-acetyl muramic acid
D. amino acids
E. *lipopolysaccharides
144.
Which of the following discoveries is NOT attributed to Sergei Winogradsky?
A. Colony enrichment on selective medium.
B. Bacteria oxidation of iron and sulfur to obtain energy.
C. CO2 fixation by non-photosynthetic microorganisms.
D. No correct answer
E. *Colony isolation on solid phase medium.
145.
Which of the following is NOT equivalent to 10 micrometers.
A. 0,0001 cm
B. 0,01 mm
C. 10,000 nm
D. 100,000 Angstroms
E. *10 nm
146.
Which of the following is NOT part of Koch's postulates?
A. the microorganism is never found in healthy animals.
B. the microorganisms is always found in diseased animals.
C. the microorganism must cause disease in healthy animals.
D. No correct answer
E. *the microorganism must secrete a toxin in culture.
147.
Which of the following is not true about bacterial flagella?
A. Most of their length consists of a hollow, rigid protein tube.
B. They are constructed largely of a single protein called flagellin.
C. They spin like wheels, either clockwise or counterclockwise.
D. They are much longer than the cell body.
E. *They use cytoplasmic ATP as their primary energy source.
148.
Which of the following is not true about capsules and slime layers?
A. They consist of secreted material lying outside of the bacterial cell
wall.
B. They can prevent desiccation of bacteria cells.
C. They help bacteria resist phagocytosis by macrophages.
D. No correct answer
E. *They are required for bacteria to grow normally in culture.
149.
Which of the following objectives would give you the best resolution of small
objects?
A. 10x air, N.A. 0.25
B. 40x air, N.A. 0.65
C. 100x oil, N.A. 1.25
D. No correct answer
E. *64x oil, N.A. 1.4
150.
Which of the following statements about Transmission Electron Microscopy is
not true.
A. The specimens are placed in a high vacuum for viewing.
B. The specimens must be sliced very thin, 20-100 nm in thickness.
C. The beam is focused by electromagnetic lenses.
D. All answers are correct
E. *The specimen must be stained with osmium or other heavy metal.
151.
Which of the following statements is most correct about the differential Gram
stain?
A. Crystal violet differentially stains Gram positive cells.
B. Gram's iodine differentially stains Gram positive cells.
C. Saffron red differentially stains Gram negative cells.
D. Etanol fixes Gram negative cells
E. *Etanol differentially destains Gram negative cells.
152.
Which of the following statements is most correct about the differential Gram
stain?
A. Crystal violet differentially stains Gram positive cells.
B. Gram's iodine differentially stains Gram positive cells.
C. Saffron red differentially stains Gram negative cells.
D. Saffron red differentially stains Gram positive cells.
E. *Acetone differentially destains Gram negative cells.
153.
Which of the following statements is most correct about Atomic Force
Microscopy (AFM)?
A. AFM can visualize protein bound to DNA molecules.
B. AFM can visualize unfixed specimens in water or buffer.
C. AFM moves a very sharp tip over the surface of the specimen to "feel" its
shape.
D. All are false
E. *All the statements are true.
154.
Which of the following structures is unique to prokaryotic organisms?
A. mitochondria
B. ribosomes
C. cell wall
D. cell membrane
E. *peptidoglycan cell wall
155.
Choose the unique structure of prokaryotic organisms?
A. mitochondria
B. ribosomes
C. cell wall
D. cell membrane
E. *peptidoglycan cell wall
156.
Who did advance the concept of Contagium vivum?
A. Pollender.
B. Louis Pasteur.
C. Lazzaro Spallanzani.
D. Edward Jenner
E. *Fracastorius.
157.
Who did coin the term vaccine?
A. Kitasato.
B. Ehrlich.
C. Louis Pasteur,
D. Robert Koch.
E. *Edward Jenner.
158.
Who did develop rabies vaccine for the first time in 1885?
A. Semple.
B. Edward Jenner
C. Paul Ehrlich.
D. Wasserman.
E. *Louis Pasteur.
159.
Who did discovere bacillus of tuberculosis?
A. Hansen.
B. Loeffler.
C. Bruce.
D. Pasteur
E. *Robert Koch.
160.
Who did introduce the method of vaccination to prevent smallpox?
A. Louis Pasteur.
B. Paul Ehrlich.
C. John Hunter.
D. Antony van Leeuwenhoek.
E. *Edward Jenner.
161.
Who first described microorganisms such as bacteria?
A. Louis Pasteur
B. Robert Koch
C. Fannie Hesse
D. Richard Petri
E. *Anton von Leeuwenhoek
162.
Who first developed the process of colony purification on solid media?
A. Louis Pasteur
B. Fannie Hesse
C. Anton von Leeuwenhoek
D. Richard Petri
E. *Robert Koch
163.
Who was this scientist? He was originally trained as a chemist, but his
studies on
fermentation led him to take interest in microorganisms. His discoveries
revolutionized
medical practice, although he never studied medicine.
A. Robert Koch.
B. Antony van Leeuwenhoek.
C. Edward Jenner.
D. Paul Ehrlich.
E. *Louis Pasteur.
164.
Why bacteria are prokaryotic organism?
A. A bacterium is a multicell organism that has a distinct nucleus.
B. A bacterium is a one-cell organism that has a distinct nucleus.
C. A bacterium is a multicell organism that does not have a distinct nucleus.
D. No correct answer
E. *A bacterium is a one-cell organism that does not have a distinct nucleus
165.
Why is a bacterium called a prokaryotic organism?
A. A bacterium is a one-cell organism that has a distinct nucleus.
B. A bacterium is a multicell organism that does not have a distinct nucleus.
C. A bacterium is a multicell organism that has a distinct nucleus.
D. No correct answer
E. *A bacterium is a one-cell organism that does not have a distinct nucleus.
166.
When flagella are distributed on both sides of the bacterial cell, the
arrangement
is known as:
A. peritrichous.
B. polytrichous
C. monotrichous.
D. lophotrichous.
E. *amphitrichous.
167.
A basic difference of bacilli and bacteria:
A. volutin granules formation
B. presence of pili
C. presence of flagella
D. capsule formation
E. *spores formation
168.
A cytoplasmic membrane is
A. a membrane that provides a barrier between the cell’s internal structures
B. a membrane that provides a selective barrier between the cell wall and the
cell’s
internal structures
C. No correct answers
D. a membrane that provides a selective barrier between the nucleus and the
cell’s
internal structures
E. *part of the cell envelope
169.
A pili is:
A. A form of storage granule within the cell.
B. Is a long thin protein rod that is used for adhesion.
C. Generally composed of sugars.
D. Always present on a cell that has the ability to form a capsule.
E. *Is a long thin protein rod that is used for bacterial exchange of genetic
material.
170.
Acid fast microbes are resistante to acid because they contain in cell wall:
A. acetylglucosamine
B. diaminopimelic acid
C. polyphosphates
D. lipopolysaccharides
E. *waxes, fatty acid
171.
After what sign of mycoplasma differ from bacteria :
A. by intracellular parasite
B. by tinctorial properties
C. by structure of cytoplasmatic membrane
D. by structure of the nucleosis
E. *by absence of cell wall
172.
All microbes belong to the spiral-shaped forms, except for:
A.
B.
C.
D.
E.
Vibrio cholera
Treponema pallidum
Leptospira interrogans
Borrelia recurrentis
*Mycobacterium tuberculosis
173.
All microbes belong to the spiral-shaped forms, except for:
A. Causative agent of leptospirosis
B. Causative agent of borreliosis
C. Causative agent of sodoku disease
D. Causative agent of cholera
E. *Causative agent of whooping cough
174.
All prokaryotes
A. produce endospores.
B. possess ribosomes identical to those of eukaryotes.
C. have cell walls.
D. have inner and outer membranes.
E. *lack mitochondria.
175.
An important property of the cell envelope of acid-fast bacteria is:
A. the presence of porins in the outer membrane
B. the lack of oxidative phosphorylation enzymes
C. the absence of a peptidoglycan layer
D. the accumulation of dipicolinic acid in the periplasmic space
E. *the abundance of mycolic acid in the cell wall
176.
Appearance of a hard chancre is characteristic of:
A. secondary syphilis.
B. latent syphilis.
C. tertiary syphilis.
D. complication of syphilis
E. *primary syphilis.
177.
Bacilli are differed from bacteria according to their ability to form:
A. capsules;
B. flagella;
C. volutin;
D. presence of ribosomes;
E. *spores
178.
Bacteria do not have some organelles, but always have:
A. Mitochondria
B. Cell membrane
C. Golgi apparatus
D. Chloroplasts
E. *Nucleoid
179.
Bacterial genus/genera of medical importance which produce endospores is/are:
A. Bacterium.
B. Corynebacterium
C. Mycobacterium
D. Micrococcus
E. *Bacillus.
180.
Bacterial genus/genera of medical importance which does not produce
endospores is/are:
A. Bacillus.
B. Clostridium.
C. all answers are correct
181.
182.
183.
184.
185.
186.
187.
188.
D. no correct answers
E. *Bacteroides
Bacteriologist can use Ziehl-Neelsen’s technique for diagnosis of what
disease?
A. croupous pneumonia
B. whooping-cough
C. flu
D. diphtheria
E. *tuberculosis
Borrelia burgdorferi does cause which disease?
A. Rocky Mountain Spotted Fever
B. Hanta
C. Rabies
D. Relapsing fever
E. *Lyme Disease
Borrelia is classified with the spirochetes because it
A. is a rod
B. is a pathogen
C. no correct answers
D. is aerobic
E. *possesses an axial filament
Capsules to the bacteria are needed for:
A. toxins production
B. antibodies production
C. spore production
D. survival in an external environment
E. *defence from fagocytosis
What is incorrect?
A. Spores are formed under adverse environmental conditions such as the
absence
of a
B. Spores are resistant to boiling
C. Spores are metabolically inactive and contain dipicolinic acid, a calcium
chelator
D. They may be killed in autoclave
E. *Spores are formed primarily by organisms of the genus Neisseria
Cell wall of bacteria can be destroyed with:
A. Sulfonamides
B. Alcohols
C. Streptomycin
D. Interferon
E. *Lysozyme
Cell wall of gram-negative bacteria contains all, except:
A. thin monolayer peptidiglycan
B. lipoproteins
C. outer membrane
D. lipopolysaccharides
E. polilayer peptidoglycan
Cell wall of gram-positive bacteria consist of:
A. thin monolayer peptidoglycan
B. lipopolysaccharides
C. outer membrane
D. lipoproteins
E. polilayer peptidoglycan
189.
Cell walls, when they exist, usually contain peptidoglycan in
A. eucaryotes only.
B. fungi
C. protozoa
D. both procaryotes and eucaryotes
E. *procaryotes only.
190.
Chlamydia trachomatis can attacks which structure(s)?
A. eye
B. urethra
C. fallopian tubes
D. lungs
E. *all are correct
191.
Choose among listed bacteria microorganism, which is not able to form a
capsule:
A. Streptococcus pneumoniae
B. Klebsiella pneumoniae
C. Bacillus anthracis
D. Yersinia pestis
E. *Rickettsia typhi
192.
Choose among listed what is correct regarding Gram positive cells
A. have a thick capsule that traps the crystal violet stain.
B. have a periplasmic space that traps the crystal violet.
C. have monolayers of peptidoglycan
D. have a second, outer membrane that helps retain the crystal violet stain.
E. *have multiple layers of peptidoglycan that help retain the crystal violet
stain.
193.
Choose among listed what is correct regarding Gram positive cells
A. have a thick capsule that traps the crystal violet stain.
B. have a periplasmic space that traps the crystal violet.
C. have monolayers of peptidoglycan
D. have a second, outer membrane that helps retain the crystal violet stain.
E. *have multiple layers of peptidoglycan that help retain the crystal violet
stain.
194.
Choose correct statement about bacterial L-forms:
A. they have no nucleoid
B. they have no ribosomes
C. they have no cell membrane
D. special type of bacterial colonies
E. *they have no cell wall
195.
Choose method of staining of bacterial spores, which are formed by C. tetani
and
C. botulinum:
A. Staining by Ziehl-Neelsen method
B. Staining by Gram method
C. Staining by crystal violet
D. Staining by mеthylene blue
E. *Staining by Anjesko method
196.
Choose method of staining of bacterial spores, which are formed by C.
perfringens:
A. Staining by mеthylene blue
B. Staining by Ziehl-Neelsen method
C. Staining by Gram method
D. Staining by crystal violet
E. *Staining by Anjesko’s method
197.
Choose method of staining of bacterial spores:
A.
B.
C.
D.
E.
Staining by Ziehl-Neelsen method
Staining by Gram method
Staining by crystal violet
Staining by mеthylene blue
Staining by Auesko’s method
198.
Clostridium and Bacillus are unique among most bacteria, because they ______
A. are acid-fast
B. do not have teichoic acids
C. are mesophiles
D. are Gram positive
E. *produce endospores
199.
Doctor-bacteriologist for examination of spores used such technique:
A. Peshkov’s
B. Gins_Burry’s
C. Ziehl-Neelsen’s
D. Gram’s
E. *Anjesky’s
200.
Each of the following statements concerning peptidoglycan is correct EXCEPT:
A. It has a backbone composed of alternating units of muramic acid and
acetylglucosamine
B. It can be degraded by lysozyme
C. No non correct answer
D. Cross-links between the tetrapeptides involve D-alanine
E. *It is thinner in gram-positive than in gramnegative cells
201.
Each of the following statements concerning peptidoglycan is correct EXCEPT:
A. It has a backbone composed of alternating units of muramic acid and
acetylglucosamine
B. Cross-links between the tetrapeptides involve D-alanine
C. It can be degraded by lysozyme
D. It has tetramers of aminoacids
E. *It is thinner in gram-positive than in gram-negative cells
202.
Each of the following statements concerning bacterial spores is correct
EXCEPT:
A. They are formed by gram-positive rods
B. They can be killed by being heated to 121 °C for 15 minutes
C. They contain much less water than bacterial cells
D. All answers are correct.
E. *Their survival ability is based on their enhanced metabolic activity
203.
Each of the following statements concerning bacterial spores is correct
EXCEPT:
A. They are formed by gram-positive rods
B. They can be killed by being heated to 121 °C for 15 minutes
C. They contain much less water than bacterial cells
D. They are formed by some gram-positive cocci
E. *Their survival ability is based on their enhanced metabolic activity
204.
Each of the following statements concerning bacterial spores is correct
EXCEPT:
A. They are formed by gram-positive rods
B. They can be killed by being heated to 120 °C? for 15 minutes
C. They contain much less water than bacterial cells
D. No correct answer
E. *Their survival ability is based on their enhanced metabolic activity
205.
Each of the following statements concerning the Gram stain is correct EXCEPT:
A. Mycoplasma pneumoniae isn't visible in the Gram stain because it doesn't
have a
cell wall
B. Salmonella typhi stains pink because it has a thin peptidoglycan layer
C. Escherichia coli stains pink because it has a thin peptidoglycan layer
D. Streptococcus pyogenes stains blue be cause it has a thick peptidoglycan
layer
E. *Mycobacterium tuberculosis stains blue because it has a thick lipid layer
206.
Each of the following statements concerning the surface structures of
bacteria is
correct EXCEPT:
A. Pili mediate the interaction of bacteria with mucosal epithelium
B. Polysaccharide capsules retard phagocytosis
C. Both gram-negative rods and cocci have lipopolysaccharide ("endotoxin") in
their cell wall
D. No correct answers
E. *Bacterial flagella are nonantigenic in humans because they closely
resemble
human flagella in chemical composition
207.
Endospore formation is a property of:
A. Escherichia coli
B. Treponema pallidum
C. Mycobacterium tuberculosis
D. Staphylococcus aureus
E. *Clostridium botulinum
208.
Endospore formation is a property of:
A. Escherichia coli
B. Treponema pallidum
C. Mycobacterium tuberculosis
D. Staphylococcus aureus
E. *Clostridium botulinum
209.
Endospores of bacillus are necessary for:
A. Defence from acid in stomach
B. Reproduction
C. Survival into human and animal’s organism
D. Defence from fagocytosis
E. *Survival in an external environment
210.
Fimbriae
A. cause bacteria move through fluids.
B. sense changes in nutrient concentration.
C. are pathways for the secretion of exoenzymes.
D. cause diseases
E. *attach bacteria to various surfaces.
211.
For the staining of spores we can use the following method:
A. Gram
B. Peshkov
C. Neisser
D. Romanovsky-Gimsae
E. *Anjesky
212.
How we can divide rod-shaped microbes?
A. bacilli, clostridia, vibrio
B. bacilli, clostridia, spirochetes
C. *bacilli, bacteria, clostridia
D. bacilli, bacteria, sarcina
E. bacteria, clostridia
213.
A.
B.
C.
D.
E.
214.
A.
B.
C.
D.
E.
215.
A.
B.
C.
D.
E.
216.
A.
B.
C.
D.
E.
217.
A.
B.
C.
D.
E.
218.
A.
B.
C.
D.
E.
219.
A.
B.
C.
D.
E.
220.
A.
B.
C.
D.
E.
221.
A.
B.
C.
Indicate disease caused by Borrelia burgdorferi:
Weil syndrome (icterohemorrhagic fever), canicola fever
Syphilis
Enteric fever
Endemic relapsing fever
*Lyme disease
Indicate disease caused by Borrelia recurrentis:
Lyme disease
Syphilis
Enteric fever
Weil syndrome (icterohemorrhagic fever), canicola fever
*Endemic relapsing fever
Indicate disease caused by Leptospira interrogans:
Endemic relapsing fever
Lyme disease
Syphilis
Enteric fever
*Weil syndrome (icterohemorrhagic fever), canicola fever
Indicate disease caused by Treponema pallidum:
Weil syndrome (icterohemorrhagic fever), canicola fever
Enteric fever
Endemic relapsing fever
Lyme disease
*Syphilis
Indicate microbe, which has sexual mode of transmission:
Borrelia recurrentis
Borrelia burgdorferi
Leptospira interrogans
Treponema vincentii
*Treponema pallidum
Lipopolysaccharide is a major constituent of cell wall in:
fungi.
protozoa.
no correct answers
gram-positive bacteria.
*gram-negative bacteria.
Louse-borne relapsing fever is caused by:
B. duttoni.
B. burgdorferi.
B. parkeri.
B. persica
*B. recurrentis.
Lyme disease is caused by ______:
Borrelia sogdiana
Borrelia hermsii
Borrelia caucasica
Borrelia recurrentis
*Borrelia burgdorferi
Lyme disease is caused by a
enteric bacteria
no correct answers
Chlamydia
D. Rickettsia
E. *spirochaeta
222.
Medical important spore forming bacteria belong to next genus:
A. Fusobacterium
B. Streptococcus
C. Escherichia
D. Proteus
E. *Clostridia
223.
Mesosomes of bacteria are analogous to:
A. lysosomes of eukaryotes.
B. Golgi apparatus of eukaryotes.
C. polyribosomes of eukaryotes
D. no correct answers.
E. *mitochondria of eukaryotes.
224.
Microbiologist used Anjesky’s technique for examination of spores. What color
will they have?
A. White
B. Black
C. Blue
D. Green
E. *Red
225.
Microorganisms capable of producing endospores include:
A. Clostridium species
B. Escherichia species
C. Staphylococcus species
D. Streptococcus species
E. *Bacillus species
226.
What microorganisms are capable to produce endospores?
A. Clostridium species
B. Escherichia species
C. Staphylococcus species
D. Streptococcus species
E. *Bacillus species
227.
Microorganisms capable of producing endospores include:
A. Clostridium tetanus
B. Clostridium botulinum
C. None of them
D. Bacillus species
E. *All of them
228.
Microorganisms, which produce spores and diameter of spores is less then
diameter of microbial cell, are called:
A. Sarcina
B. Bacteria
C. Vibrio
D. Clostridia
E. *Bacilli
229.
Microorganisms, which produce spores and diameter of spores is more then
diameter of microbial cell, are called:
A. Bacilli
B. Sarcina
C. Bacteria
D. Vibrio
230.
231.
232.
233.
234.
235.
236.
237.
238.
E. *Clostridia
Most Bacillus species are
A. commensals
B. saprophyticus
C. true pathogens
D. opportunistic pathogens
E. *nonpathogens
Mycoplasmas damage next structure of host cells.
A. nucleus
B. cell walls
C. mitochondria
D. ribosomes
E. *cell membranes
Ornithosis is a _____ infection associated with ______
A. rickettsial, parrots
B. rickettsial, flies
C. chlamydial, rats
D. chlamydial, mice
E. *chlamydial, birds
Peptidoglycan is major constituent of cell wall of:
A. gram-negative bacteria.
B. fungi.
C. protozoa.
D. no correct answers
E. gram-positive bacteria.
Plasmids are important to the genetics of many bacteria. This is because
A. they are inherited from one generation to the next.
B. they may carry genes that give their host a selective advantage.
C. they can render bacteria drug-resistant.
D. they may carry genes that code sugarlytic enzymes
E. *All answers are correct.
Plasmids are important to the genetics of many bacteria. This is because
A. they are inherited from one generation to the next.
B. they may carry genes that give their host a selective advantage.
C. no correct answers
D. they can render bacteria drug-resistant.
E. *All answers are correct.
Plasmids are important to the genetics of many bacteria. This is because
A. they are inherited from one generation to the next.
B. they may carry genes that give their host a selective advantage.
C. they can render bacteria drug-resistant.
D. they may carry genes that code sugarlytic enzymes
E. *All answers are correct.
Polyphosphate inclusion bodies
A. turn reddish brown when stained with iodine.
B. are composed of polymers of glucose.
C. no correct answer
D. protect bacteria from excessive drying.
E. *supply of nutritives and energy
Resistance of bacteria to acid depends from present in cytoplasm
A. acetylglucosamine
B. diaminopimelic acid
239.
240.
241.
242.
243.
244.
245.
246.
247.
C. poliphosphates
D. lipopolysaccharides
E. *oxyacid, fatty acid
Rickettsia and chlamydia are similar in being
A. produce exotoxin
B. the cause of eye infections
C. carried by arthropod vectors
D. free of a cell wall
E. *obligate intracellular bacteria
Rickettsia, an aerobic Gram-negative rod, is which of the following?
A. Obligate extracellular parasite
B. Facultative intracellular parasite
C. No correct answer
D. *Obligate intracellular parasite
Rod-shaped microbes which do not sporulate are called:
A. Clostridia
B. Bacilli
C. Vibrio
D. Sarcina
E. *Bacteria
Show main reason of sporulation in bacteria:
A. Decrease of temperature of the environment
B. Senescence of bacterial cells
C. Increase of temperature of the environment
D. Decrease the of partial pressure of oxygen
E. *Disappearance in the environment of sources of carbon and nitrogen
Show one property of spirochaetes:
A. Presence of teichoic acids in the cell wall
B. Golgi’s apparatus in a cytoplasm
C. They are peritrichates bacteria
D. Capsules and spores formation
E. *Axial filament and cytoplasm wound spirally around the filament
Show why spores are necessary spores for the bacteria:
A. survivalence in the animals and human body
B. protection from the acid content of a stomach
C. defense from phagocytosis
D. reproduction
E. *survivalence in the environment
Specify morphological group of Borrelia:
A. Spirillas
B. Streptobacteria
C. Vibrios
D. Thread-like bacteria
E. *Spirochaetes
Specify morphological group of Leptospira:
A. Thread-like bacteria
B. Vibrios
C. Streptobacteria
D. Spirillas
E. *Spirochaetes
Specify morphological group of Treponema:
A. Thread-like bacteria
B.
C.
D.
E.
248.
A.
B.
C.
D.
E.
249.
to:
A.
B.
C.
D.
E.
Vibrios
Spirillas
Streptobacteria
*Spirochaetes
Specify morphological types of causative agents of chancroid
Spirochaetes and Streptobacilli
Treponema and Borrelia
Strerptobacteria and Spirochaetes
Staphylococci and spirilla
*Strerptobacteria and streptobacilli
Specify what group of microorganisms the causative agents of
belong
Vibrios
Spirillas
Thread-like bacteria
Streptobacteria
*Spirochaetes
250.
Specify what group of microorganisms the causative agents of
belong to:
A. Thread-like bacteria
B. Vibrios
C. Streptobacteria
D. Spirillas
E. *Spirochaetes
251.
Specify what group of microorganisms the causative agents of
belong to:
A. Thread-like bacteria
B. Vibrios
C. Spirillas
D. Streptobacteria
E. *Spirochaetes
252.
Specify when do protoplasts appear:
A. action of penicillin against acid fast bacteria
B. action of penicillin against gram negative bacteria
C. action of penicillin against rickettsia
D. action of penicillin against spiral-shaped bacteria
E. *action of penicillin against gram positive bacteria
253.
Why are spores necessary to bacteria for:
A. Survival into human and animal’s organism
B. Defence from fagocytosis
C. Defence from acid in stomach
D. Reproduction
E. Survival in an external environment
254.
Spores are necessary to bacteria for:
and anthrax:
syphilis do
Lyme disease do
leptospirosis do
A. Defence from fagocytosis
B. Defence from acid in stomack
C. Survival into human and animal’s organism
D. Reproduction
E. *Survival in an external environment
255.
The 70S procaryotic ribosomes consist of
A. a 40S and a 30S subunit.
B. a 50S and a 20S subunit.
C. a 30S and 70S subunit.
D. two 40S subunits.
E. *a 50S and a 30S subunit.
256.
The bacteria of what family have a capsule constantly?
A. shigella
B. brucella
C. shigella
D. salmonella
E. *klebsiella
257.
The bacterial capsule is:
A. A form of storage granule within the cell.
B. a long thin protein rod that is used for bacterial exchange of genetic
material.
C. Always present on a cell that has the ability to form a capsule.
D. a long thin protein rod that is used for attachment.
E. *A coating on the exterior of many cells.
258.
The causative agent/s of gas gangrene is/are:
A. Clostridium perfringens.
B. novyi.
C. С. septicum.
D. histolyticum
E. *All answers are correct.
259.
The Chlamydiaceae life cycle has elemental bodies (EBs) that are ____ and
reticulate bodies (RBs) that are ____.
A. Metabolically active and noninfection; Metabolically inactive and
infectious
B. Metabolically inactive and noninfection; Metabolically active and
infectious
C. No correct answer
D. Metabolically active and infection; Metabolically inactive and
noninfectious
E. *Metabolically inactive and infection; Metabolically active and
noninfectious
260.
The cytoplasmic membrane of bacteria is important in each of the following
aspects EXCEPT:
A. exocytosis of proteins
B. oxidative phosphorylation and electron transport
C. active transport of nutrients
D. cell wall synthesis
E. *location of lipopolysaccharide
261.
The cytoplasmic membrane of bacteria is important in each of the following
aspects EXCEPT:
A. exocytosis of proteins
B. B. oxidative phosphorylation and electron transport
C. active transport of nutrients
D. cell wall synthesis
E. *location of lipopolysaccharide
262.
The LPS or lipopolysaccharide found in Gram-negative bacteria IS NOT
composed of:
A. lipid A
B. core polysaccharide
C. O-antigen
D. All are true
E. *peptidoglycan
263.
The LPS or lipopolysaccharide found in Gram-negative bacteria IS NOT
composed of:
A. lipid A
B.
C.
D.
E.
264.
A.
B.
C.
D.
E.
265.
A.
B.
C.
D.
E.
266.
A.
B.
C.
D.
E.
267.
A.
B.
C.
D.
E.
268.
A.
B.
C.
D.
E.
269.
A.
B.
C.
D.
E.
270.
A.
B.
C.
D.
E.
271.
A.
B.
C.
D.
E.
core polysaccharide
O-antigen
any of the answer
*peptidoglycan
The major function of the cell membrane is:
To keep water from entering the cell, which causes it to burst.
No correct answer
To bind lipids in the hydrophobic inner layer.
To trap energy so the cell can use it for the functions of life.
*To protect the internal components of a cell from the chaos outside of a
cell.
The most important role of the prokaryotic cell wall is to
protect the cell from osmotic pressures.
prevent ions from diffusing away from the cell.
block the effects of antibiotics like penicillin.
protect cell from oxygen
*maintain the shape of the cell.
The nature of volutin granules:
Carbohydrates
Lipoproteins
Unsaturated fatty acids
Proteins
*Metaphosphates
The significance of the plasma membrane is that
it prevents movement of molecules out of the organism
it is the site of protein synthesis
All answers are correct
no correct answers
*it selectively allows some molecules to pass into the organism
The structure of DNA was discovered by:
Darwin and Crick
Pasteur and Koch
Baltimore and Flemming
Watson and Griffith
*Watson and Crick
To the spiral-shaped bacteria ______ belong:
staphylococci
streptococci
bacilli
clostridia
*spirillas
To the spiral-shaped bacteria _____ belong:
streptococci
bacilli
clostridia
streptobacilli
*treponema
To the spiral-shaped bacteria …… belong:
clostridia
streptobacilli
staphylococci
streptococci
*borrelia
To the spiral-shaped bacteria …… belong:
A. staphylococci
B. streptococci
C. streptobacilli
D. clostridia
E. *leptospira
273.
Typical drumstick appearance of bacilli is observed in:
A. Clostridium perfringens.
B. histolyticum.
C. difficilae
D. tetani.
E. *С. botulinum.
274.
Upon staining by Ziehl-Neelsen’s method after treating with sulphuric acid
the
preparation is necessary ussing
A. Ziehl’s phenol fuchsine
B. Alcohol
C. Меthylene blue
D. iodine solution
E. *Washing with water
275.
Upon staining spores by Auesko’s method after treating with hydrochloric acid
and heating the preparation:
A. is treating with sulphuric acid
B. is staining with crystal violet
C. is staining with mеthylene blue
D. is washing with water and drying out
E. *is staining with Ziehl’s phenol fuchsine
276.
What are plasmides of bacteria?
A. circular DNA in a shell
B. fragments DNA in volutin granules
C. circular RNA in cytoplasm
D. circular DNA in nucleoid
E. *circular DNA in a cytoplasm
277.
What bacterium does cause gas gangrene?
A. Leptospira interrogans
B. Staphylococcus aureus
C. Escherichia coli
D. Clostridium tetani
E. *Clostridium perfringens
278.
What causative agent does cause relapsing fever?
A. Treponema pallidum
B. Leptospira interrogans
C. Spirillum minor
D. Vibrio cholerae
E. *Borrelia recurrentis
279.
What color do Mycobacteria have got when Ziehl-Neelsen technique is used for
their staining?
A. white
B. blue
C. violet
D. yellow
E. *red
272.
280.
What color has Mycobacterim tuberculosis after staining by Ziehl-Neelsen
technique
A. blue
B. brown
C. green
D. violet
E. *red
281.
What disease does Rickettsia prowazekii cause?
A. Scrub typhus (Asia, Oceania)
B. Q fever (Worldwide)
C. Relapsing fever
D. Rocky Mountain spotted fever (Western hemisphere)
E. *Epidemic typhus (Worldwide)
282.
What disease does Vibrio cholerae cause?
A. Typhoid fever
B. Paratyphoid A fever
C. Salmonellosis
D. Dysentery
E. *Cholera
283.
What is a plasmid?
A. Self-replicating segment of single stranded RNA
B. A bacterial chromosome
C. Bacterial inclusion
D. Part of the ribosome
E. *Self-replicating segment of double stranded DNA
284.
A plasmid is ________
A. *Self-replicating segment of double stranded DNA
B. Self-replicating segment of single stranded RNA
C. A bacterial chromosome
D. Bacterial inclusion
E. Bacterial spores
285.
What is necessary component of bacterial ultrastructure?
A. Nucleus
B. Mitochondria
C. Cell membrane
D. Golgi apparatus
E. *Mesosome
286.
What is the first symptom of Treponema pallidum?
A. Bulls eye rash
B. Butterfly rash
C. Vaginal discharge
D. No correct answer
E. *Chancre at infection site
287.
What is the name of causative agent of syphilis?
A. Escherichia coli
B. Borrelia recurrentis
C. Treponema macrodentium
D. Leptospira interrogans
E. *Treponema pallidum
288.
What is the name of rod-shaped spore-forming microbes, which localize in the
smear separately?
A. Monobacteria
B.
C.
D.
E.
Micrococci
Diplobacteria
Vibrio
*Monobacilli
289.
What is the name of rod-shaped spore-forming microbes, which form chains in
the smear?
A. Monobacilli
B. Monobacteria
C. Streptococci
D. Diplobacteria
E. *Streptobacilli
290.
What is the purpose of bacterial endospores?
A. Allow the bacterium to make hundreds of "seeds" to spread on the wind.
B. Help the bacterium to differentiate into faster growing stages of
bacteria.
C. All answers are correct.
D. Allow the bacterium to survive the absence of oxygen.
E. *Allow the bacterium to survive the bad condition (drying,lack of
nutrition).
291.
What is the value of a hanging-drop preparation?
A. For study sensitivity to antibiotics
B. For study cultural properties of bacteria
C. For examinate biochemical properties
D. For study morphology of bacteria
E. *For study motility of bacteria
292.
What is the value of a wet-mount preparation?
A. For study morphology of bacteria
B. For examinate biochemical properties
C. For study sensitivity to antibiotics
D. For study cultural properties of bacteria
E. *For study motility of bacteria
293.
What is unique about the Mycoplasma bacterial group?
A. they lack a cell membrane
B. they have organelles
C. they are prokaryotes
D. they have ribosome
E. *they lack cell walls
294.
What microbe have comma-shaped form?
A. Spirillas
B. Threadlike bacteria
C. Streptobacilli
D. Spirochaetes
E. *Vibrio
295.
What morphological structure is responsible for bacterial motility?
A. Capsule
B. LPS
C. Pilli
D. Fimbria
E. *Flagella
296.
What’s next step of staining by Ziehl-Neelsen method after using sulphuric
acid?
A. Staining by Ziehl phenol fuchsine
B. Using alcohol
C. Using iodine solution
D. Staining by mеthylene blue
E. *Washing with water
297.
When bacteria have cluster of flagella at one or both sides, the arrangement
is
known as:
A. monotrichous.
B. amphitrichous.
C. peritrichous.
D. bitrichous
E. *lophotrichous.
298.
When flagella are distributed all round the bacterial cell, the arrangement
is
known as:
A. monotrichous.
B. lophotrichous.
C. bitrichous
D. amphitrichous.
E. *peritrichous.
299.
When flagella are distributed all round the bacterial cell, the arrangement
is
known as:
A. monotrichous.
B. lophotrichous.
C. polytrichous
D. amphitrichous.
E. *peritrichous.
300.
When flagella are distributed on both sides of the bacterial cell, the
arrangement
is known as:
A. monotrichous.
B. peritrichous.
C. bitrichous
D. lophotrichous.
E. *amphitrichous.
301.
When we use Romanowsky-Giemsa technique for staining of Borrelia, they have
such color:
A. green
B. red
C. black
D. yellow orange
E. *violet
302.
When would you use a wet mount?
A. A wet mount is used to observe an inorganic specimen under a microscope.
B. A wet mount is the first step in preparing a specimen.
C. A wet mount is the last step in preparing a specimen
D. A wet mount is used to observe a dead specimen under a micro-scope.
E. *A wet mount is used to observe a live specimen under a microscope.
303.
Which from following microorganisms does contain sterols in the cytoplasmic
membrane?
A. Bacillus.
B. Clostridium.
C. Proteus.
D. Ricketsia
E. *Mycoplasma.
304.
Which of the following bacteria don’t contain cell wall?
A. Ricketsia
B. Slaphylococcus.
305.
306.
307.
308.
309.
A.
B.
C.
D.
E.
310.
311.
312.
313.
C. Treponema.
D. Klebsiella
E. Mycoplasma
Which of the following causes syphilis?
A. Serratia marcescens
B. Treponema vincentii
C. Helicobacter pylori
D. Proteus mirabilis
E. *Treponema pallidum
Which of the following does NOT apply to plasmids?
A. They are composed of DNA.
B. They multiply independently of the chromosome.
C. They may pass from cell to cell in recombinations.
D. They may be included into chromosome
E. *They are essential to growth of the cell.
Which of the following exotoxins is most toxic?
A. Tetanus toxin.
B. Diphtheria toxin.
C. Cholera toxin.
D. Shigella toxin
E. *Botulinum toxin.
Which of the following is associated with Ureaplasma?
A. Atypical pneumonia
B. Tracheobronchitis
C. Influenza-like illness
D. Upper respiratory tract infection
E. *Genitourinary tract infection
Which of the following is not true about bacterial flagella?
They use cytoplasmic ATP as their primary energy source.
All answers are correct.
Most of their length consists of a hollow, rigid protein tube.
They are constructed largely of a single protein called flagellin.
*They spin like wheels, either clockwise or counterclockwise.
Which of the following is not true about capsules and slime layers?
A. They help bacteria resist phagocytosis by macrophages.
B. They help bacteria survive inside organism
C. They consist of secreted material lying outside of the bacterial cell
wall.
D. They can prevent desiccation of bacteria cells.
E. *They are required for bacteria to grow normally in culture.
Which of the following is not true about capsules and slime layers?
A. They consist of secreted material lying outside of the bacterial cell
wall.
B. They help bacteria resist phagocytosis by macrophages.
C. All answers are correct.
D. They can prevent desiccation of bacteria cells.
E. They are required for bacteria to grow normally in culture.
Which of the following is true for visualizing spirochete?
A. They can be viewed without significant magnification (e.g. 10x)
B. They can be viewed with Gram stain
C. They are eucaryotes
D. They can be viewed with Giemsa stain
E. *They require special analysis such as dark field illumination or silver
stain
Which of the following materials is NOT a bacterial storage granule:
A.
B.
C.
D.
E.
Sulfur
Glycogen
Volutin
Phosphorous
*Ribosomes
314.
Which of the following members of the Clostridium family causes
pseudomembranous colitis?
A. Clostridium botulinum
B. Clostridium perfringens
C. Listeria monocytogenes.
D. Clostridium tetani
E. *Clostridium difficile
315.
Which one of the following organisms principally infects vascular endothelial
cells?
A. Haemophilus influenzae
B. Coxiella burnetii
C. All answers are correct
D. Salmonella typhi
E. *Rickettsia typhi
316.
Which one of the following types of organisms is NOT an obligate
intracellular
parasite and there fore can replicate on bacteriologic media?
A. Adenovirus
B. Rickettsia
C. All answers are correct
D. Chlamydia
E. *Mycoplasma
317.
Which property does mycoplasma differ from bacteria most of all?
A. by intracellular parasite
B. by tinctorial properties
C. by structure of cytoplasmatic membrane
D. by structure of the nucleoid
E. by absence of cell wall
318.
Which with these substances can damage bacteria cell wall:
A. interferone
B. alcohol
C. streptomycin
D. sulphonamides
E. *lysozyme
319.
A 25-year-old man complains of a urethral discharge. You perform a Gram stain
on a specimen of the discharge and see neutrophils but no bacteria. Of the
organisms
listed, the one MOST likely to cause the discharge is
A. Treponema pallidum
B. Candida albicans
C. Coxiella burnetii
D. All are true
E. *Chlamydia trachomatis
320.
A microbiologist found pili during electrone microscopy of gram-negative
bacteria. The pili are:
A. A form of storage granule within the cell.
B. long thin protein rod that is used for adhesion.
C. Generally composed of sugars.
D. Always present on a cell that has the ability to form a capsule.
E. *long thin protein rod that is used for bacterial exchange of genetic
material and
bacterial attachment to the host cell.
321.
A student has to compare presence of different chemicals in bacterial cell
wall.
The Gram-positive and Gram-negative cell walls generally differ in that the
Grampositives exclusively possess:
A. carbohydrates
B. peptides
C. lipid
D. none of the above
E. *teichoic acids
322.
A student has to point some important properties of bacteria. Each of the
following statements concerning the surface structures of bacteria is correct
EXCEPT:
A. Pili mediate the interaction of bacteria with mucosal epithelium
B. Polysaccharide capsules retard phagocytosis
C. Both gram-negative rods and cocci have lipopolysaccharide ("endotoxin") in
their cell wall
D. None of the above
E. *Bacterial flagella are nonantigenic in humans because they closely
resemble
human flagella in chemical composition
323.
A student stained Mycobacteria by Ziehl-Neelsen technique. What color has
Mycobacterim tuberculosis after staining by Ziehl-Neelsen technique
A. blue
B. violet
C. brown
D. green
E. *red
324.
A tetanus was diagnosed in patient D. Bacterioscopic examination of tested
material revealed the presence of spore-forming microbes. The diameter of the
spores
was more then diameter of cells. What microbes are there in the smear?
A. Vibrio
B. Bacilli
C. Helicobacter
D. Bacteria
E. *Clostridia
325.
After examination of tested material from carbuncle rod-shaped bacteria with
spores were revealed. These microbes belong to:
A. Diplococci
B. Monobacteria
C. Diplobacilli
D. Streptobacteria
E. *Streptobacilli
326.
Anthrax was diagnosed in patient S. What is the causative agent of this
disease?
A. Borrelia recurrentis
B. Treponema pallidum
C. Sarcina ventriculi
D. Staphylococcus aureus
E. *Bacillus anthracis
327.
Anthrax was diagnosed in patient V. What morphological type of bacteria do
these microbes belong to?
A. Streptococci
B. Streptobacteria
C. Bacteria
D. Staphylococci
E. *Streptobacilli
328.
Bacteriologisd found spore-producing bacteria in tested material from the
patient
with food poison. Endospore formation is a property of:
A. Escherichia coli
B. Staphylococcus aureus
C. Treponema pallidum
D. Mycobacterium tuberculosis
E. *Clostridium botulinum
329.
Bacteriologist after isolation of bacterial pure cultures found microbes with
spores. Microorganisms, which produce spores and diameter of spores is more
then
diameter of microbial cell, are called:
A. Bacilli
B. Sarcina
C. Bacteria
D. Vibrio
E. *Clostridia
330.
Bacteriologist compares procaryotis and eucaryotic cells. Bacteria do not
have
some organelles, but always have:
A. Mitochondria
B. Cell membrane
C. Golgi apparatus
D. Chloroplasts
E. *Nucleoid
331.
Bacteriologist examines bacterial spores in tested material. Upon staining
spores
by Anjesky method after treating with hydrochloric acid and heating the
preparation:
A. is staining with mеthylene blue
B. is washing with water and drying out
C. is treating with sulphuric acid
D. is staining with crystal violet
E. *is staining with Ziehl phenol fuchsine
332.
Bacteriologist has to examine some morphologic property of bacteria. What is
the value of a hanging-drop preparation?
A. For study sensitivity to antibiotics
B. For examinate biochemical properties
C. For study cultural properties of bacteria
D. For study morphology of bacteria
E. For study motility of bacteria
333.
Bacteriologist stains asid fast bacteria. What’s next step of staining by
ZiehlNeelsen method after sulphuric acid using sulphuric acid?
A. Staining by Ziehl phenol fuchsine
B. Using alcohol
C. Staining by mеthylene blue
D. Using iodine solution
E. Washing with water
334.
Capsule-producing bacteria are examined in the laboratory. Which of the
following is not true about capsules and slime layers?
A. They consist of secreted material lying outside of the bacterial cell
wall.
B. They can prevent desiccation of bacteria cells.
C. They help bacteria resist phagocytosis by macrophages.
D. They help bacteria survive inside organism
E. *They are required for bacteria to grow normally in culture.
335.
Causative agent of cholerae is comma-shaped. Show its genes name.
A. Spirochaetes
B. Spirillas
C. Threadlike bacteria
D. Streptobacilli
E. *Vibrio
336.
Chemical structure of bacteria are examining in the laboratory. Which from
following microorganisms does contain sterols in the cytoplasmic membrane?
A. Bacillus.
B. Clostridium.
C. Proteus.
D. Ricketsia
E. *Mycoplasma.
337.
Chlamydia trachomatis may be cultured on monolayers of human cells. If
infected cells were examined by staining and light microscopy, what would you
be most
likely to observe?
A. A membrane-bounded inclusion body, within the cytoplasm.
B. Rod shaped Gram-positive bacilli, free within the cytoplasm.
C. Gram-negative cocci, within the lumen of the endoplasmic reticulum.
D. Cyst forms inside lysosomes.
E. *A membrane-bounded inclusion body, within the nucleus.
338.
Chlamydia uses ATP for energy in order to construct DNA and proteins as well
as encoding their own ribosomes. This makes Chlamydia which of the following?
A. Extracellular pathogen
B. Obligate extracellular parasite
C. Facultative intracellular parasite
D. No correct answer
E. *Obligate intracellular parasite
339.
During examination of gram-negative cell wall of microbes a microbiologist
can
observe alll layers, except:
A. thin monolayer peptidiglycan
B. lipoproteins
C. outer membrane
D. lipopolysaccharides
E. polilayer of peptidoglycan
340.
Escherichia coli is isolated from the patients feces. When flagella are
distributed
all round the bacterial cell, the arrangement is known as:
A. monotrichous.
B. lophotrichous.
C. amphitrichous.
D. bitrichous
E. *peritrichous.
341.
For diagnosis of Klebsiella infection it is necessary to examine bacterial
capsule.
What method will be used:
A. Ziehl-Neelsen
B. Gram
C. Anjesky
D. Loeffler
E. *Gins-Burry
342.
For diagnosis of syphilis Romanowsky-Giemsa’s technique was used for staining
of tested material from hard chancre. What color will Treponema pallidum
have? For
diagnosis of tuberculosis a doctor used special technique for staining of
sputum. What
method will be used for examination of acid-fast microbes?
A. Gram
B. Anjesky
C. Loeffler
D. Burry
E. *Ziehl-Neelsen
343.
For examination of presence of bacterial capsule Burry-Gins method was used.
The capsules of bacteria after staining will be:
A. red
B. white
C. blue
D. violet
E. *colorless
344.
From a patient with pneumonia capsule-producing microbes were isolated. Show
the genes of microorganisms wich can have a capsule constantly?
A. shigella
B. salmonella
C. brucella
D. vibrio
E. *klebsiella
345.
From child’s sputum causative agent of pertussis was found. It does not
belong
spiral-shaped bacteria. Find it.
A. Causative agent of cholera
B. Causative agent of leptospirosis
C. Causative agent of borreliosis
D. Causative agent of sodoku disease
E. *Causative agent of whooping cough
346.
In bacterial cells, ribosomes are packed into the cytoplasmic matrix and also
loosely attached to the plasma membrane. What is the function of ribosomes?
A. Site of energy production
B. Site of genetic reproduction
C. Site of toxin production
D. Site of lipopolysaccharide production
E. *Site of protein synthesis
347.
In the child S. of 2 years old mycotic stomatitis was diagnosed. Specify the
causative agent of this disease.
A. Staphylococci
B. Streptococci
C. Sarcina
D. Treponema
E. *Candida
348.
In the laboratory bacterial tested material for the presence of spores must
be
examined. Choose method of staining of bacterial spores:
A. Staining by mеthylene blue
B. Staining by Gram method
C. Staining by Ziehl-Neelsen method
D. Staining by crystal violet
E. *Staining by Anjesky’s method
349.
In the smear from hard chancre spiral-shaped bacteria were found. Causative
agent of syphilis – …. belongs to the spiral-shaped bacteria.
A. streptococci
B.
C.
D.
E.
bacilli
clostridia
streptobacilli
*treponema
350.
In the smear from tested materia clostridia with terminal spores localization
were
found. Typical drumstick appearance of bacilli is observed in:
A. Clostridium perfringens.
B. *Clostridium tetani.
C. Clostridium histolyticum.
D. Clostridium difficilae
E. Clostridium botulinum.
351.
In the smear from the patient with gas gangrene spore-forming bacteria were
revealed. The diameter of the spores is more then diameter of cells. What
microbes are
there in the smear?
A. Bacilli
B. Vibrio
C. Sarcina
D. Bacteria
E. *Clostridia
352.
In the smear from the soil gram-positive rod-shaped bacteria were found. What
is
the name of rod-shaped spore-forming microbes, which localize in the smear
separately?
A. Monobacteria
B. Micrococci
C. Diplobacteria
D. Vibrio
E. *Monobacilli
353.
In the smear from the tested material in patient with rat-bite fever coiled
form of
bacteria exhibiting twists with one or more turns were found. They are
similar to corkscrew. What group do these bacteria belong to?
A. Spirochaetes
B. Vibrios
C. Streptobacteria
D. Streptobacilli
E. *Spirilla
354.
In the smear, which was stained by Anjesky method there are separate
sporeforming cells. What microbes are there in the smear?
A. Monobacteria
B. Diplobacilli
C. Diplobacteria
D. Streptobacilli
E. *Monobacilli
355.
In the water after microscopic research comma-shaped microorganisms were
revealed. What morphological group do these bacteria belong to?
A. Spirochaetes
B. Spirillas
C. Threadlike bacteria
D. Streptococci
E. *Vibrio
356.
It is necessary to compare bacteria and bacilli according to their
morphologic
properties. Bacilli are differed from bacteria according to their ability to
form:
A. capsules;
B. flagella;
C. volutin;
D. presence of ribosomes;
E. *spores
357.
It is necessary to compare chemical structure of some causative agents. Whose
cell walls do usually contain peptidoglycan?
A. eucaryotes only.
B. fungi
C. protozoa
D. both procaryotes and eucaryotes
E. *procaryotes only.
358.
It is necessary to describe of microbes according to the presence of cell
wall.
Which of the following bacteria don’t contain cell wall?
A. Klebsiella
B. Ricketsia
C. Slaphylococcus.
D. Treponema.
E. *Mycoplasma
359.
It is necessary to desribe biological function of cell membrane. The
cytoplasmic
membrane of bacteria is important in each of the following aspects EXCEPT:
A. exocytosis of proteins
B. cell wall synthesis
C. oxidative phosphorylation and electron transport
D. active transport of nutrients
E. *location of lipopolysaccharide
360.
It is necessary to examine bacterial motility. What morphological structure
is
responsible for bacterial motility?
A. Pilli
B. Fimbria
C. Capsule
D. LPS
E. *Flagella
361.
It is necessary to exwmine bacterial spores in tested material. For the
staining of
spores we can use the following method:
A. Gram
B. Romanovsky-Gimsa
C. Peshkov
D. Neisser
E. *Anjesky
362.
It is necessary to make express diagnosis of anaerobic gas infection in
patient
with purulent discharges from the wound and oedima of the tissues. Choose
method of
staining of bacterial spores, which are formed by C. perfringens:
A. Staining by mеthylene blue
B. Staining by Ziehl-Neelsen’s method
C. Staining by Gram method
D. Staining by crystal violet
E. *Staining by Anjesko’s method
363.
It is necessary to stain bacteria by Gram’s technique. What dye is not used
for
this staining:
A. Crystal violet (gencyanviolet)
B. Alcohol
C. Iodine solution
D. Fuchsine solution or safranin solution
E. *Methylene blue
364.
It is necessary to write down the name of causative agent of syphilis in
patient’s
medical card. What is its name?
A. Escherichia coli
B. Borrelia recurrentis
C. Treponema macrodentium
D. Leptospira interrogans
E. *Treponema pallidum
365.
Microbiologist isolated causative agent of anthrax. Show bacterial
genus/genera
of medical importance which produce endospores is/are:
A. Bacterium.
B. Corynebacterium
C. Mycobacterium
D. Micrococcus
E. *Bacillus.
366.
Microbiologist revealed chains of gram-positive bacteria in the smear. What
is
the name of rod-shaped spore-forming microbes, which form chains in the
smear?
A. Monobacilli
B. Monobacteria
C. Streptococci
D. Diplobacteria
E. *Streptobacilli
367.
Salmonella typhi was isolated from patient’s blood. Rod-shaped microbes which
do not sporulate are called:
A. Clostridia
B. Bacilli
C. Vibrio
D. Sarcina
E. *Bacteria
368.
Specify the obligatory structural component of microbial cell. The absence of
it
causes the death of the cell:
A. Nucleus
B. Mesosome
C. Episome
D. Ribosome
E. *Cell membrane
369.
Student examines Mycoplasma morphologic properties. What property does
mycoplasma differ from bacteria most of all?
A. by structure of the nucleoid
B. by intracellular parasite
C. by tinctorial properties
D. by structure of cytoplasmatic membrane
E. *by absence of cell wall
370.
Student found rod-shaped bacteria in the patient’s pus. How we can divide
rodshaped microbes?
A. bacilli, clostridia, vibrio
B. bacilli, clostridia, spirochetes
C. bacilli, bacteria, sarcina
D. bacteria, clostridia
E. *bacilli, bacteria, clostridia
371.
Tested material from a patient with suspicion on diphtheria was stained with
Neisser’s method. What color will have volutin granules?
A.
B.
C.
D.
E.
Dark brown
Bright red
thin monolayer peptidoglWhite
Yellow
*Dark blue
372.
The doctor examines cell wall of gram-positive bacteria under electron
microscope. Cell wall of gram-positive bacteria consist of:
A. lipopolysaccharides
B. outer membrane
C. periplasm space
D. all are true
E. *peptidoglycan multilayer
373.
The examination of the bacterial cell under an electronic microscope shows,
that
a cell membrane is multi-layers structure. It consists of:
A. Triple layer of lipids
B. Four layers of carbohydrates
C. Two layers of phosphoproteins
D. Triple layers of phosphoproteins
E. *Double layer of phospholipid molecules
374.
What bacterial organoid has such properties: occupies central part of
bacterium,
has irregular form, has no membrane separating it from the a cytoplasm,
combines with
the cell membrane and меsosome, consists of one double stranded DNA, which
has
ring-like form, and integrated with RNA, RNA-polymerase and protein:
A. Plasmid
B. Nucleus
C. Granules of volutin
D. Ribosomes
E. *Nucleoid
375.
What groups do spiral-shaped bacteria form. Choose correct answer:
A. Spirochaetes, borrelia, treponema
B. Spirochaetes, sarcina, borrelia
C. Streptobacilli, treponema, borrelia
D. Vibrio, treponema, borrelia, leptospira
E. *Vibrio, spirilla, spirochaetes
376.
What is a name of extrachromosomal DNA which is ring-like and provides the
synthesis of some substances, enzymes, resistance of bacteria to antibiotic?
A. Nucleus
B. Mesosome
C. Ribosome
D. Nucleoid
E. *Plasmid
377.
Ziehl-Neelsen’s method is utilized for staining of acid fast bacteria. Upon
staining by Ziehl-Neelsen’s method after treating with sulphuric acid the
preparation is
necessary ussing
A. Ziehl phenol fuchsine
B. Alcohol
C. Меthylene blue
D. iodine solution
E. *Washing with water
378.
A degree of heterotrophy may be different. The highest heterotrophy has
prokaryotic organisms that are able to live only inside the living cells.
There are:
A. staphylococci and streptococci
B.
C.
D.
E.
escherichia and salmonellas
mycobacteria and bordetella
vibrios and treponema
*rickettsia and chlamidia
379.
According to the types of bacterial respiration such groups of bacteria may
exist:
A. obligate aerobes
B. obligate anaerobes
C. facultative anaerobes
D. capneic
E. all answers are correct
380.
After finishing the experience in students’ laboratory it’s necessary to
disinfect
the workplace. What chemicals can be used?
A. ether
B. hydrochloric acid
C. formalin
D. chloroform
E. Chloraminum
381.
After inoculation of Escherichia coli on Ploskirev medium the growth of
bacteria
is inhibited. What chemical does predetermine this phenomenon?
A. oxalic acid
B. sodium sulphite
C. Bismuth salts
D. fuchsine
E. brilliant green
382.
After inoculation of Escherichia coli on Ploskirev medium the growth of these
bacteria is inhibited. What chemical does predetermine this phenomenon?
A. oxalic acid
B. sodium sulphite
C. Bismuth salts
D. fuchsin
E. brilliant green
383.
After inoculation of the patient’s faces on Endo medium there were grown to
types of colonies: one – red with metallic hue, other – colorless. To what
group of
nutrient media does Endo medium belong to?
A. Elective
B. Enriching media
C. Universal
D. Selective
E. Differential diagnostic
384.
After inoculation of the patient’s faces on Endo medium there were grown to
types of colonies: one – red with metallic hue, other – colorless. To what
group of
nutrient media does Endo medium belong to?
A. Elective
B. Enriching media
C. Universal
D. Selective
E. Differential diagnostic
385.
After practical classes it is necessary to kill bacterial cultures that were
used by
students. Choose optimum regime of sterilization in an autoclave:
A. 120 °С 20 minutes
B. 112 °С 15 minutes
C. 134 °С 10 minutes
D. 127 °С 30 minutes
E. *134 °С 40 minutes
386.
Choose the method of inoculation of bacteria, based on mechanical principle?
A. Ru method
B. Gram method
C. Inoculation by a needle
D. Morozov’s method
E. *Drigalsky’s method
387.
Constructive and power metabolisms are closely connected, sometimes their
ways coincide, and the same substrates are utilized for different
necessities. Such ways
are called:
A. metabolic
B. associative
C. dissociative
D. polybolic
E. *amphibolic
388.
Energy which is generated by a cell exists in the form of electrochemical
transmembrane gradient of hydrogen ions - ?µН+ or in ATP molecules. What is
the
mechanism of transformation of one type of energy into another?
A. electronic ATF-synthesizing complex
B. molecular ATF-synthesizing complex
C. all answers are correct
D. all answers are wrong
E. *proton ATF-synthesizing complex
389.
For abolition of bacteria in the air of operating room such methods may be
used:
A. gamma-rays
B. X-ray
C. gamma-rays
D. gas method
E. *ultraviolet light
390.
For biological control of sterilization quality microorganisms are used. What
important feature do they have?
A. gram-positive
B. gram-negative
C. form capsules
D. form pili
E. *form spores
391.
For checking the quality of filters in an experiment a fluid containing
testmicrobe was filtered. Later a filter was laid on the surface of nutrient
medium and put
into the incubator. How much time is it necessary to hold it on medium to
give a final
answer about its quality?
A. at an optimum temperature, 1 day
B. optimum temperature, 2 days
C. optimum temperature, 3 days
D. optimum temperature, 4 days
E. *at an optimum temperature, 5 day
392.
For examination of lipolytic properties of microbes lipids and lipid-like
substances – tweens – are supplemented into the medium. According to what
signs is it
possible to check the lipolytic activity of bacteria?
A. The colonies of bacteria lose their typical form
B. The colonies of bacterium are painted in the color of indicator
C. The colonies of bacteria become semilucent
D. If microbes lipolytic properties they do not grow on an medium
E. *Bacteria form iridescent halos round the colonies
393.
For inoculation of microbes according to Drigalsky’s method we can use:
A. Bacteriological loop
B. Bacteriological needle
C. Jar
D. All answers are correct
E. *Spatula
394.
From patient with gas anaerobic infection it is necessary to select the
culture of
causative agents. What medium can be chosen?
A. Leffler medium
B. Petrov medium
C. Makkoy-Chepin medium
D. Petranyani medium
E. *Kitt-Tarozzi medium
395.
How is it possible to divide microorganisms according to spore formation for
isolation of pure culture?
A. To cool the tested material before inoculation, as spores are
insusceptible to the
action of low temperatures
B. To inoculate tested material and cultivate it in anaerobic conditions
C. To inoculate tested material, which contains spore forming bacilli in
laboratory
animals
D. To separate microorganisms according to their spore formation is
impossible
E. *To heat the tested material before inoculation, as spores are heat stable
396.
How is it possible to increase the phase of exponential growth and
Mconcentration artificially?
A. additional aeration (for anaerobes)
B. it is impossible to do it
C. a right answer is not present
D. all answers are correct
E. *by the renewing of nutrient medium
397.
How is it possible to verify the motility of bacteria which form colony?
A. Staining according to Leffler technique
B. Staining according to Gram technique
C. By electron microscope
D. By «lying» drop technique
E. *By «hanging» drop technique
398.
How much time is it necessary to cultivate the bacteria on Bismuth-sulfite
agar
for final evaluation of growth?
A. 12 hours
B. 24 hours
C. 36 hours
D. 96 hour
E. 48 hours
399.
In a bacteriological laboratory it is necessary to nutrient media, which
contain
matters, changing at a temperature higher then 100 °С (urea, carbohydrates,
proteins).
What method of sterilization can you offer?
A. autoclave, pressed steam
B. boiling
C. tindalization
D. pasteurization
E. by fluid steam
400.
In a hospital chief of department decided to check the quality of instruments
sterilization in an autoclave by biological method. What bacteria can be used
for this
test?
A. Pathogenic
B. Capsule-forming
C. Acid fast
D. Thermophilic
E. Spore-forming
401.
In a laboratory for acceleration of sterilization of media with sugar by live
steam
this procedure was made in one day: in the morning, in the day-time and in
the evening
for 30 min. How it was necessary to sterilize these media correctly?
A. To sterilize 1 hour
B. To sterilize 15 minutes
C. To sterilize 45 minutes
D. To sterilize twice for a days
E. To sterilize three times with 24 hour interval
402.
In a medium with gelatin it is possible to verify the proteolytic properties
of
bacteria. What signs will show the positive results?
A. compression of gelatin column
B. discoloration of gelatin column
C. appearance of dark precipitate in gelatin column
D. brightening of gelatin column.
E. *liquefacience of gelatin column
403.
In a pharmacy it is necessary to sterilize the small bottles with
physiological
sodium chloride solution for injection. What method is it necessary to apply
for their
sterilization?
A. in an autoclave by fluid steam
B. in a heat oven
C. by X-ray
D. by ultraviolet light
E. *in an autoclave, 2 atmospheres
404.
In MPB it is necessary to check the peptolytic properties of bacteria. How is
it
possible to prove, if microbes produce the hydrogen sulphide?
A. to insert the indicator paper, saturated with the lead acetate; the
hydrogen
sulphide is changed its color, it becomes red
B. to insert the indicator paper, saturated with the lead acetate; the
hydrogen
sulphide is changed its color, it becomes dark blue
C. the color of medium becomes greyish
D. to insert the indicator paper, saturated with an oxalic acid; the hydrogen
sulphide
is changed by its color, it becomes black
E. *to insert the indicator paper, saturated with a lead acetate; the
hydrogen
sulphide is changed its color, it becomes black
405.
How is it possible to check the peptolytic properties of bacteria if microbes
produce the hydrogen sulphide? In MPB it is necessary.
A. to insert the indicator paper, saturated with the lead acetate; the
hydrogen
sulphide is changed its color, it becomes red
B. to insert the indicator paper, saturated with the lead acetate; the
hydrogen
sulphide is changed its color, it becomes dark blue
C. the color of medium becomes greyish
D. to insert the indicator paper, saturated with an oxalic acid; the hydrogen
sulphide
is changed by its color, it becomes black
E. *to insert the indicator paper, saturated with a lead acetate; the
hydrogen
sulphide is changed its color, it becomes black
406.
In MPB it is necessary to verify the peptolytic properties of bacteria. What
signs
will show the positive results – indole production?
A. to insert the indicator, saturated with a litmus; an indole is changed its
color, it
becomes red
B. to insert the indicator paper, saturated with an oxalic acid; an indole is
changed
its color, it becomes dark blue
C. to insert the indicator paper, saturated with a litmus; a indole is
changed its color,
it becomes dark blue
D. the color of medium becomes rose
E. *to insert the indicator paper, saturated with an oxalic acid; a indole is
changed
by its color, it becomes rose
407.
In Olkenitsky medium Escherichia coli was inoculated. What changes will be
observed in a medium, which will confirm utilization of glucose and lactose?
A. The changes of the color from a pinky to yellow in the column of agar
B. The changes of the color from a pinky to yellow of slant surface of agar
C. The changes of the color from a pinky to red of slant surface of agar
D. *The changes of the color from a pinky to yellow of the column of agar and
slant surface of agar
E. The changes of the color from a pinky to red of the column of agar
408.
In Olkenitsky medium Salmonella typhi, which decomposes glucose to acid was
inoculated. What changes will be observed in a medium, which certify
utilization of
glucose?
A. The changes of the color from a pinky to yellow the slant surface of agar
B. The changes of the colorfrom a pinky to red the slant surface of agar
C. The changes of the color from a pinky to yellow of the column of agar and
the
slant surface of agar
D. The changes of the color from a pinky to red of the column of agar
E. *The changes of the color from a pinky to yellow of the column of agar
409.
In what period after finishing the disinfections is it recommended to collect
the
control samples?
A. not before than in 10-15 min
B. not before than in 15-20 min
C. not before than in 20-30 min
D. not before than in 60-90 min
E. *not before than in 30-45 min
410.
In what phases of growth are microorganisms most physiologically and
functionally active?
A. initial and exponential
B. stationary and death
C. initial and stationary
D. exponential and death
E. *exponential and stationary
411.
It is necessary to verify the hemolytic properties of bacteria. What nutrient
media
will you recommend?
A. simple MPA
B. simple MPB
C. Endo medium
D. serum MPA
E. *blood MPA
412.
It is necessary to check the peptolytic properties of bacteria. What nutrient
medium will you recommend?
A. MPA
B. Endo medium
C. Ru medium
D. 1 % alkaline peptone water
E. *MPB
413.
It is necessary to check the quality of sterilization by compressed steam at
121
°С. What chemical indicator of sterilization effectiveness may be used?
A. benzonaphtolum
B. antipyrine
C. sulphur
D. urea
E. *benzoic acid
414.
It is necessary to check the quality of sterilization by compressed steam at
115
°С. What chemical indicator of sterilization quality can be utilized?
A. benzonaphtolum
B. sulphur
C. benzoic acid
D. urea
E. *antipyrine
415.
It is necessary to check the quality of sterilization by compressed steam at
132
°С. What chemical indicator of sterilization quality can be utilized?
A. benzonaphtolum
B. antipyrine
C. sulphur
D. benzoic acid
E. *urea
416.
It is necessary to choose the medium for cultivation of anaerobic bacteria:
A. Endo and Levin media
B. Meat-peptone agar, meat-peptone broth
C. Coagulated serum, мeat-peptone gelatin
D. Ascitic agar, serum agar
E. *Zeissler blood-sugar agar, Kitt-Tarozzi medium
417.
It is necessary to conduct pasteurization of juice. Choose its optimum mode:
A. 70 °С during 5-10 min
B. 70 °С during 30 min
C. 80 °С during 60 min
D. 80 °С during 30 min
E. *80 °С during 5-10 min
418.
It is necessary to create anaerobic conditions for cultivation of proper
bacteria by
biological method. What method could you propose?
A. Zeissler method
B. Shukevich method
C. Veynberg method
D. Pasteur method
E. *Fortner method
419.
It is necessary to examine the carriage of Corynebacterium diphtheriae by the
child. What nutrient medium is elective for Corynebacterium diphtheriae?
A. Levenshteyn-Yensen medium
B. Ploskirev medium
C. Ol'kenickiy medium
D. Endo medium
E. *Ru medium
420.
It is necessary to examine the carriage of Corynebacterium diphtheriae among
the children. What nutrient medium is elective for Corynebacterium
diphtheriae?
A. Levenshteyn-Yensen medium
B. Ploskirev medium
C. Ol'kenickiy medium
D. Endo medium
E. *Ru medium
421.
It is necessary to examine the carriage of vibrios by group of persons in the
infection focus of cholera. What medium is elective for Vibrio cholerae?
A. MPB
B. Ploskirev medium
C. Serum MPB
D. Ascitic agar
E. *1 % alkaline peptone water
422.
In the infection focus of cholera it is necessary to examine the carriage of
vibrios
by group of persons. What medium is elective for Vibrio cholerae?
A. MPB
B. Ploskirev medium
C. Serum MPB
D. Ascitic agar
E. *1 % alkaline peptone water
423.
It is necessary to make biochemical identification and examine saccharolytic
properties of bacteria. What media could you offer?
A. Veynberg medium
B. Zeissler medium
C. Kitt-Tarozzi medium
D. Milk
E. *Hiss media
424.
It is necessary to prepare a coagulated serum for isolation of culture of
Corynebacterium diphtheriae. How can you do serum coagulation?
A. 80-90 °С during 10 min
B. 70-80 °С during 30 min
C. 50-60 °С during 30 min
D. 90-100 °С during 30 min
E. *80-90 °С during 1 hour.
425.
It is necessary to study ability of microbes to utilize glucose, saccharose,
lactose,
production of hydrogen sulphide and utilization of urea. What nutrient media
will you
recommend?
A. Ru medium
B. Leffler medium
C. Endo medium
D. Ploskirev medium
E. *Olkenitsky medium
426.
Laboratory diagnosis of tetanus was made in the laboratory. What method of
sterilization is it necessary to use for killing the selected cultures?
A. Boiling
B. Tindalization
C. In the heat oven
D. Pasteurization
E. *Autoclaving
427.
Material from a patient with suspicion on typhoid fever was inoculated on
Endo
medium. What color do colony of lactosonegative Salmonella typhi have?
A. dark blue
B. red with metallic hue
C. brown
D. green
E. *colorless
428.
Material from a patient with suspicion on an intestinal infection was
inoculated
on the Ploskirev medium. What color do colonies of Escherichia coli have?
A. dark blue
B. colorless
C. brown
D. green
E. *red
429.
What color do colonies of Escherichia coli Endo medium?
A. blue
B. colorless
C. brown
D. green
E. *red with metallic hue
430.
Material from a patient with suspicion on shigellosis (dysentery) was
inoculated
on the Ploskirev medium. What color do colonies of lactose-negative Shigella
have?
A. dark blue
B. green
C. red with metallic hue
D. brown
E. *colorless
431.
Material from a sick woman must be inoculated on selective nutrient medium
for
diagnosis of vaginal candidiasis. Indicate this medium?
A. Mueller medium
B. 1 % alkaline peptone water
C. Ru medium
D. Blood agar
E. *Saburo medium
432.
Material from patient with typhoid fever must be inoculated on selective
nutrient
medium. Choose most optimum from them:
A. 1 % alkaline peptone water
B. Ru medium
C. Blood agar
D. Saburo medium
E. *Mueller medium
433.
Choose most optimum selective nutrient medium for cultivation material from
patient with typhoid fever:
A. 1 % alkaline peptone water
B. Ru medium
C. Blood agar
D. Saburo medium
E. *Mueller medium
434.
Microorganisms that are used as an energy source chemical reaction in the
medium belong to:
A. heterotrophs
B. autotrophs
C. lithotrophs
D. organotrophs
E. *chemotrophs
435.
Microorganisms that require the high concentrations of salts for their
cultivation
are called:
A. Acidophilic
B. Oxyphilic
C. Mesophilic
D. Osmiephilic
E. *Halophilic
436.
Microorganisms, which are able to cause diseases in human, belong to:
A. chmemoorganoautotrophs
B. chemolithoheterotrophs
C. chemolithoautotrophs
D. heterophotoorganotrophs
E. *chemoorganoheterotrophs
437.
On the Endo medium colonies Escherichia and Salmonella were isolated.
According to what signs is it possible to make differentiation between them?
A. sizes
B. character of the edges
C. character of surfaces
D. consistency
E. *color
438.
Some microorganisms lost their capacity for a synthesis of some necessary
growth factors, therefore they do not can grow on minimum nutrient media.
They are
called:
A. paratrophs
B. prototrophs
C. myxotrophs
D. oligotrophs
E. *auxotrophs
439.
Specify, to what pressure in an autoclave a temperature 100 °С corresponds:
A. 0,5 аtm.
B. 1 аtm.
C. 1,5 аtm.
D. 2 аtm.
E. *0 аtm.
440.
The pure culture of microorganisms grows and develops at presence of an
atmosphere oxygen (no less what 20 % oxygen). What group of microorganisms
according to their respiration does this culture belong to?
A. Obligate anaerobes
B. Facultative anaerobes
C. Microaerophilic bacteria
D. Capneic bacteria
E. Obligate aerobes
441.
The wild bacteria are able to synthesize all necessary for them substances
from a
limited number of organic compounds, for example, from glucose and ammonium
salts.
They are called:
A. auxotrophs
B. paratrophs
C. myxotrophs
D. oligotrophs
E. *prototrophs
442.
To Chlorine which are used for disinfection, belong all preparation, EXCEPT:
A. 0,2-1,0 % solutions of Chloramin B.
B. 5 % water solutions of calcium hypochloride.
C. 0,05-0,1 % solution of trichloroisocyanuric acid.
D. 0,1-0,2 % sulfochlorantine solution
E. *3-5 % solution of carbolic acid.
443.
To the universal nutrient mediums belong:
A. serum MPA and serum MPB.
B. Endo medium and Ploskirev medium
C. Blood MPA and TCBS medium
D. Yolk salt agar and Ascitic agar
E. *MPA and MPB
444.
What chemicals can be added into of nutrient media for cultivation of
anaerobes,
because they have oxygen-fixing effect?
A. Calcium gluconate
B. Potassium permanganate
C. Iron sulfate
D. Aluminium hydroxide
E. *Sodium hydrosulphite
445.
What color does acquire the semi-solid Hiss media with the indicators water
blue
and rosolic acid there is sugar utilization?
A. red
B. green
C. brown
D. a color does not change
E. *dark blue
446.
What color does acquire the semi-solid Hiss media with the indicators water
blue
and rosolic acid there is sugar utilization?
A. red
B. green
C. brown
D. a color does not change
E. *dark blue
447.
What color does non-inoculated Olkenitsky medium have?
A. bright red
B. yellow
C. greenish
D. violet
E. *pink
448.
What degree of disinfection does foresee elimination of spore-forming
microorganisms and protozoans?
A. A.
B. B.
C. C.
D. E.
E. *D.
449.
What finished products do appear in the liquid Hiss media after decomposition
of sugars?
A. alkali
B. the gas fixing
C. only acid
D. finished goods is not succeeded
E. *acid and gas
450.
What groups of methods for creation of anaerobic conditions for anaerobic
bacteria cultivation do you know ?
A. mechanical
B. physical
C. chemical
D. biological
E. *all answers are correct
451.
What indicator is utilized in a medium with an urea according to Christensen?
A. Andrede
B. Neutral red
C. Bromthymol blue
D. Thymol blue
E. *Phenol red
452.
What is it necessary to take into consideration during collection of tested
material for isolation of anaerobic causative agents?
A. Warning neutralizing effect of antibodies
B. Warning influence of disinfectants
C. Warning changes of tinctorial properties
D. Warning changes of serologic properties
E. *Warning toxic effect of oxygen
453.
What microorganisms can produce such enzymes, as fibrinolysin,
streptodornase, streptokinase, which are used for patients’ treatment?
A. Staphylococci
B. Neisseria
C. Branchamella
D. Gemella
E. *Streptococci
454.
What technique does belong to the mechanical method of anaerobic conditions
creation?
A. use of Gas generating box
B. to cover the surface of medium with the layer of vaseline oil
C. use Fortner’s method
D. all answers are correct.
E. *answers are incorrect
455.
Which of following technique does belong to the mechanical method of
anaerobic conditions creation?
A. use of Gas generating box
B. use the Drygalsky method
C. all answers are right.
D. all answers are incorrect
E. *use Fortner’s method
456.
What technique does belong to the physical method of anaerobic conditions
creation?
A. use of Gas generating box
B. to cover the surface of medium with the layer of vaseline oil
C. use the high column of nutrient medium
D. a right answer is not present
E. *regeneration of nutrient medium before inoculation
457.
What way does glycerin penetrate in a bacterial cell?
A. Translocation of radicals
B.
C.
D.
E.
Active transport
Diffusion
Facilitated diffusion
Ionic transport
458.
What signs will show the positive results in a medium with gelatin to verify
the
proteolytic properties of bacteria?
A. compression of gelatin column
B. discoloration of gelatin column
C. appearance of dark precipitate in gelatin column
D. brightening of gelatin column.
E. *liquefacience of gelatin column
459.
With the purpose of increase of boiling temperature and removal of water hard
during sterilization by boiling it is necessary to add:
A. 1 % potassium permanganate
B. 1 % magnesium sulfate
C. 1 % ether
D. 1 % alcohol
E. *1 % sodium bicarbonate
460.
A nurse has to sterilize surgical tools for an operation. What the most
effective
method can you offer?
A. fractional sterilization
B. sterilization by pressed steam
C. boiling
D. ionizing radiation
E. *by dry heat
461.
According to the colony relief there may have such form:
A. Flat
B. Convex
C. Dome-shaped
D. Pitted
E. *All answers are correct
462.
According to the source of energy there are such bacteria:
A. lithotrophs and organotrophs
B. chemotrophs and heterotrophs
C. autotrophs and chemotrophs
D. autotrophs and heterotrophs
E. *chemotrophs and phototrophs
463.
All nutrient media are elective, EXCEPT:
A. Mueller medium
B. Ru medium
C. Leffler medium
D. 1 % alkaline peptone water
E. *blood MPA
464.
All of bacteria belong to obligate aerobes, EXCEPT:
A. Mycobacterium tuberculosis
B. Yersinia pestis
C. Micrococci spp.
D. Vibrio cholerae
E. Clostridium tetani
465.
How you can decribe bacterial S-colonies?
A. gyrose surface
B. fibred consistency
C. edges are rosette-like
D. have capsules
E. *smooth, convex surface
466.
All of the listed microorganisms are facultative anaerobes, EXCEPT:
A. Escherichia spp.
B. Salmonella spp.
C. Shigella spp.
D. Staphylococci spp.
E. *Clostridium botulinum
467.
Anaerobic microorganisms may be isolated by:
A. Fortner method
B. Shukevich method
C. Pasteur method
D. Leffler method
E. *Veynberg method
468.
At what advantage of Koch method in comparing Pasteur one for isolation of
pure cultures?
A. Enables to examine antibiotics susceptibility
B. Enables to examine biochemical properties of microbes
C. Enables to examine antigen properties of microbes
D. Has no substantial advantage
E. *Enables to obtain the isolated colonies
469.
At what temperature agar-agar melts and becomes solid?
A. 90 °С and 70 °С
B. 80 °С and 60 °С
C. 60 °С and 30 °С
D. 50 °С and 20 °С
E. *100 °С and 40 °С
470.
Bacteria that form R-colonies have all features, EXCEPT:
A. have capsules
B. sometimes does not have flagella
C. low or absent virulence
D. easily phagocytable
E. *does not have capsules
471.
Bacteria that form S-colonies have all features, EXCEPT:
A. poorly phagocytable
B. biochemically more active
C. isolated in the acute stage of disease
D. have capsules
E. *easily phagocytable
472.
What nutrient medium is not elective?
A. Mueller medium
B. Ru medium
C. Leffler medium
D. 1 % alkaline peptone water
E. *blood MPA
473.
Bacteria, which form R-colonies, have all features, EXCEPT:
A. have capsules
B. sometimes does not have flagella
C. low or absent virulence
D. easily phagocytable
E. *does not have capsules
474.
A.
B.
C.
D.
E.
475.
A.
B.
C.
D.
E.
476.
A.
B.
C.
D.
E.
477.
A.
B.
C.
D.
E.
478.
A.
B.
C.
D.
E.
479.
A.
B.
C.
D.
E.
480.
A.
B.
C.
D.
E.
481.
A.
B.
C.
D.
E.
482.
A.
B.
C.
Bacteria, which form S-colonies, have all features, EXCEPT:
easily phagocytable
poorly phagocytable
biochemically more active
isolated in the acute stage of disease
*have capsules
Choose a correct answer according to the types of microbial decontamination:
sterilization
disinfections
antiseptics
all answer are false
*all answer are right
Choose among listed media, differential diagnostic ones.
MPA
Kitt-Tarozzi medium
MPB
glucose MPB
*Hiss media
Indicate vitamins necessary for bacterial growth:
Biotin
Thiamine
Riboflavin
all answers are false
*all answers are correct
What are differential media used for?
Examination of antibiotics susceptibility of bacteria
Accumulations of microbial biomass
Examination of microorganisms pathogenicity
Studies of antigen structure of microorganisms
*Examination of bacteria enzyme activity
Choose bacteria susceptible to desiccation.
Shigella
Corynebacterium diphtheriae
Vibrio cholerae
Salmonella typhi
*Gononococci
Choose capneic bacteria:
Shigella dysentery
Clostridium botulinum
Mycobacterium tuberculosis
Corynebacterium diphtheriae
*Brucella abortus
Choose faithful composition of yolk salt agar
MPA, ram’s , 9 % of sodium chloride
MPA, rabbit’s red cells, 9 % of sodium chloride
MPA, horse’s red cells , 9 % of sodium chloride
MPA, glucose, 9 % chloride of sodium
*MPA, yolk, 9 % of sodium chloride
Choose among the folowing microaerophils:
Clostridium tetani
Escherichia coli
Salmonella typhi
483.
484.
485.
486.
487.
488.
to:
D. Salmonella paratyphi A
E. *Lactobacillus acidophilus
Choose obligate aerobes among these microorganisms:
A. Neisseria gonorrhoeae, Streptococcus pneumoniae
B. Clostridium tetani, Clostridium botulini
C. Shigella dysenteriae, Salmonella typhi
D. Bacillus anthracis, Brucella melitensis
E. *Mycobacterium tuberculosis, Micrococcus spp.
Choose obligate anaerobes among these microbes:
A. Bacillus anthracis
B. Mycobacterium tuberculosis
C. Rickettsia spp.
D. Corynebacterium spp.
E. *Bacteroides spp.
Choose obligate anaerobes:
A. Escherichia coli
B. Salmonella typhi
C. Proteus vulgaris
D. Brucella melitensis
E. *Clostridium tetani, Clostridium botulinum
Choose the best destroying examined cultures of bacteria:
A. sterilization by boiling
B. sterilizations heat oven
C. sterilization by ethylene oxide
D. sterilization by ultraviolet light
E. *sterilizations in an autoclave
Choose the components of Ressel’s medium:
A. MPB, lactose (1 %), glucose (0,1 %) and indicator WR (water blue and
rosolic
acid)
B. MPB, lactose (1 %), glucose (0,1 %) and indicator fuchsine
C. semi-solid agar, lactose (1 %), maltose (0,1 %) and indicator WR (water
blue
and rosolic acid)
D. semi-solid agar, mannitol (1 %), glucose (0,1 %) and indicator WR (water
blue
and rosolic acid)
E. *semi-solid agar, lactose (1 %), glucose (0,1 %) and indicator of WR
(water blue
and rosolic acid)
Colonies of bacteria that grow on solid nutrient media can differentiate
according
A.
B.
C.
D.
E.
consistencies
density
colour
all answers are false
*all answers are correct
489.
According what property/ies bacterial colonies grown on solid nutrient media
can differentiate to:
A. size
B. density
C. colour
D. all answers are false
E. *all answers are correct
490.
Differential diagnostic media are used for:
A. Examination of antibiotics susceptibility of bacteria
B.
C.
D.
E.
Accumulations of microbial biomass
Examination of microorganisms pathogenicity
Studies of antigen structure of microorganisms
*Examination of bacteria enzyme activity
491.
During a Log-phase the division of bacteria takes place:
A. delay of division
B. more bacteria die, then appears
C. division does not take place
D. all answers are incorrect
E. *at constant rate
492.
During the evolution bacteria produced the ways of energy reception. There
are:
A. fermentation
B. respiration
C. photosynthesis
D. all answers are false
E. *all answers are correct
493.
All techniques belong to the chemical method of anaerobic conditions
creation,
ECXEPT:
A. use of Gas generating box
B. application of pyrogallol on the bottom of the jar
C. use the high column of nutrient medium
D. All answers are correct
E. *introduction of pieces of liver or muscles and vitamin K3 as supplement
in the
medium
494.
During the stationary phase of population growth the reason of bacterial
death is:
A. diminishing amount of nutrients
B. changes of medium рН
C. toxic action of appearing substances
D. all answers are false
E. *all answers are correct
495.
Efficiency of sterilization at heating is characterized the D index. What is
it?
A. time (minutes), which is necessary at certain temperature to get twofold
diminishing the bacterial population
B. time (minutes), which is necessary at certain temperature, to get fourfold
diminishing the bacterial population
C. time (minutes), which is necessary at certain temperature, to get fivefold
diminishing the bacterial population
D. time (minutes), which is necessary at certain temperature to destroy all
bacteria
E. *time (minutes) which is necessary at certain temperature to get the
tenfold
diminishing the bacterial population
496.
Endo medium consist of:
A. MPB, 1 % lactose, an indicator fuchsin
B. MPA, 1 % lactose, an indicator eosin
C. MPB, 1 % lactose, an indicator eosin
D. MPA, 5 % lactose, indicator bromthymol blue
E. *MPA, 1 % lactose, an indicator fuchsin
497.
Enzymes that are produced outside from bacteria are called:
A. endoenzymes
B. heteroenzymes
C. isoenzymes
D. all answers are correct
E. *exoenzymes
498.
Ethylene oxide may be used for sterilization of all equipment, EXCEPT:
A. equipment for anesthesia
B. prostheses
C. endoscope instruments
D. catgut
E. *nutrient media
499.
For chemical sterilization by gases all of them are used, EXCEPT:
A. formaldehyde
B. chloroform
C. beta-propiolactone
D. ethylene oxide
E. *methan
500.
For examination of lipolytic properties of microbes lipids and lipid-like
substances – tweens – are supplemented into the medium. According to what
signs is it
possible to check the lipolytic activity of bacteria?
A. The colonies of bacteria lose their typical form
B. The colonies of bacterium are painted in the colour of indicator
C. The colonies of bacteria become semilucent
D. If microbes lipolytic properties they do not grow on an medium
E. *Bacteria form iridescent halos round the colonies
501.
For examination of saccharolytic properties of bacteria we can use such
medium:
A. Veinberg medium
B. Zeissler medium
C. Kitt-Tarozzi medium
D. Milk
E. *Hiss media
502.
For inoculation of microbes by Streak technique we can use:
A. Spatula
B. Bacteriological needle
C. All answers are false
D. All answers are correct
E. *Bacteriological loop
503.
For mechanical separation of bacteria is not used such method:
A. Pasteur serial dilution
B. Koch method
C. Drigalsky method
D. Streak method
E. *Leffler method
504.
For obtaining the isolated colonies of aerobic bacteria tested material is
inoculated ______:
A. in liquid nutrient media
B. in laboratory animals
C. in chicken embryo
D. in cell cultures
E. *on solid nutrient media
505.
For sterilization of glassware utilize a heat oven is used. Choose the best
regime
of sterilization:
A. 180 ?C, 15-30 minutes
B. 180 ?C, 120 minutes
C. 160 ?C, 45-60 minutes
D. 160 ?C, 15-30 minutes
E. *180 ?C, 45-60 minutes
506.
For verifying the peptolytic properties of bacteria what substances are
examined
in nutrient medium?
A. carbon acid and water
B. glucose and lactose
C. carbon acid and nitrogen
D. mannitol and methanol
E. *indole and hydrogen sulphide
507.
Free oxygen is toxic for bacteria, because it:
A. kills a bacterium
B. halts spore formation
C. inhibit toxins production
D. inhibit enzymes production
E. *detains their growth
508.
Heat oven may be used for sterilization of all objects, EXCEPT:
A. tubes
B. Petry plates
C. metallic instruments
D. heat-resistant powders
E. *rubber corks
509.
How is it possible to carry out control of sterility of nutrient media after
their
preparing?
A. By keeping in incubator at 37 °С for a few hours.
B. By keeping in incubator at a temperature 42 °С for a few hours
C. By keeping at 18-20 °С. For a few hours
D. Right answer is not present
E. *By keeping in incubator for a few days at 37 °С.
510.
How is it possible to separate microorganisms according to their motility?
A. To inoculate tested material on the upper part of slant agar
B. To do inoculation of tested material by a prick in the slant agar
C. To separate microorganisms is impossible
D. To add to the tested material some substance which cause coagulation
flagella
protein
E. *To inoculate tested material in the drop of fluid in the lower part of
slant agar
511.
How is it possible to verify motility of bacteria that form a colony?
A. By Grey’s technique
B. Staining according to Zdrodovsky’s technique
C. By scanning microscope
D. By radioisotope method
E. *By wet-mount technique
512.
How many classes of bacterial enzymes do you know?
A. 3
B. 4
C. 5
D. 2
E. *6
513.
How much time is it necessary to cultivate the bacteria on Bismuth-sulfite
agar
for final evaluation of growth?
A. 12 hours
B. 24 hours
C. 36 hours
D. 96 hour
E. *48 hours
514.
What are the basic components of Ploskirev medium.
A. MPB, saccharose, salts of bile acids, diamond green, neutral red.
B. MPA, saccharose, salts of bile acids, diamond green, neutral red.
C. MPB, lactose, salts of bile acids, diamond green, neutral red.
D. MPA, glucose, salts of bile acids, diamond green, neutral red.
E. *MPA, lactose, salts of bile acids, diamond green, neutral red.
515.
In an autoclave at 112 °С nutrient media were sterilized. What test-microbe
is it
possible to utilize for sterilization control?
A. Bacillus cereus
B. Bacillus subtilis
C. Clostridium perfringens
D. Clostridium difficile
E. *Clostridium sporogenes
516.
What nutrient media were sterilized in an autoclave at 121 °С. What
testmicrobe is it possible to utilize for sterilization control?
A. Bacillus cereus
B. Bacillus subtilis
C. Clostridium perfringens
D. Clostridium difficile
E. *Bacillus steatothermophilus
517.
In burner’s flame it is possible to sterilize everything, EXCEPT:
A. forceps
B. bacteriological loops
C. slide glasses
D. cover slip
E. *tampons for material collection
518.
In depending on substrate that is donor of electrons there are such bacteria:
A. lithotrophs and chemotrophs
B. chemotrophs and heterotrophs
C. autotrophs and chemotrophs
D. chemotrophs and phototrophs
E. lithotrophs and organotrophs
519.
According to the source of carbon there are such groups of bacteria:
A. lithotrophs and organotrophs
B. chemotrophs and phototrophs
C. chemotrophs and heterotrophs
D. autotrophs and chemotrophs
E. autotrophs and heterotrophs
520.
In heat oven at 160 C Petry palates were sterilized. What test-microbe is it
possible to use for sterilization control?
A. Bacillus cereus
B. Clostridium perfringens
C. Clostridium difficile
D. Proteus vulgaris
E. *Bacillus subtilis
521.
What Indicate the composition of liquid Hiss media
A. 1 % peptone water, different carbohydrates (glucose, maltose, lactose,
saccharose, Mannitol and other), an indicator bromthymol dark blue
B. 1 % peptone water, different carbohydrates (glucose, maltose, lactose,
saccharose, Mannitol and other), an indicator eosin
C. 1 % peptone water, different carbohydrates (glucose, maltose, lactose,
saccharose, Mannitol and other), an indicator is methylene blue
D. 1 % peptone water, different carbohydrates (glucose, maltose, lactose,
saccharose, Mannitol and other), an indicator neutral red
E. *1 % peptone water, different carbohydrates (glucose, maltose, lactose,
saccharose, Mannitol and other), indicator of Andrede
522.
Which of following is/are nutrient medium/a?
A. Blood agar
B. Serum agar
C. Ascitic agar
D. All answers are false
E. *All answers are correct
523.
In MPB it is necessary to verify the peptolytic properties of bacteria. What
signs
will show the positive results – indole production?
A. to insert the indicator, saturated with a litmus; an indole is changed its
colour, it
becomes red
B. to insert the indicator paper, saturated with an oxalic acid; an indole is
changed
its colour, it becomes dark blue
C. to insert the indicator paper, saturated with a litmus; a indole is
changed its
colour, it becomes dark blue
D. the colour of medium becomes rose
E. *to insert the indicator paper, saturated with an oxalic acid; a indole is
changed
by its colour, it becomes rose
524.
What is the composition of Olkenitsky medium?
A. MPA, lactose, complex salt of ammonium-iron sulfate (Moor’s salt), sodium
thiosulphate, urea, an indicator is phenol red
B. MPA, saccharose, complex salt of ammonium-iron sulfate (Moor’s salt),
sodium
thiosulphate, urea, an indicator is phenol red
C. MPA, glucose, complex salt of ammonium-iron sulfate (Moor’s salt), sodium
thiosulphate, urea, an indicator is phenol red
D. MPA, lactose, saccharose, glucose, complex salt of ammonium-iron sulfate
(Moor’s salt), sodium thiosulphate, an indicator is phenol red
E. *MPA, lactose, saccharose, glucose, complex salt of ammonium-iron sulfate
(Moor’s salt), sodium thiosulphate, urea, an indicator is phenol red.
525.
In Olkenitsky medium Escherichia coli was inoculated. What changes will be
observed in a medium, which will confirm utilization of glucose and lactose?
A. The changes of the colour from a pinky to yellow in the column of agar
B. The changes of the colour from a pinky to yellow of slant surface of agar
C. The changes of the colour from a pinky to red of slant surface of agar
D. The changes of the colour from a pinky to red of the column of agar
E. *The changes of the colour from a pinky to yellow of the column of agar
and
slant surface of agar
526.
In Olkenitsky medium microorganism which products an urea was inoculated.
What changes here will be observed in a medium?
A. appearance of red precipitate in the column of agar
B. appearance of greenish precipitate in the column of agar
C. to fix the urease production in this medium is impossible
D. appearance of brightly-violet precipitate in the column of agar
E. *medium will become red
527.
In Olkenitsky medium Salmonella typhi, which forms the hydrogen supplied was
inoculated. What changes will be in a medium in this case?
A. To fix the hydrogen sulphide production in this medium is impossible
B. appearance of greenish precipitate in the column of agar
C. appearance of bright red precipitate in the column of agar
528.
529.
530.
531.
532.
533.
534.
535.
D. all medium will be black
E. *appearance of black precipitate in the column of agar
Indicate aerotolerant bacteria:
A. Streptococcus viridans
B. Staphylococcus aureus
C. Staphylococcus epidermidis
D. Staphylococcus saprophyticus
E. *Streptococcus pyogenes
Indicate among these microbes obligate anaerobic bacteria:
A. Staphylococcus spp., Streptococcus spp.
B. Escherichia coli, Salmonella typhi
C. Bacillus anthracis, Brucella abortus
D. Mycobacterium tuberculosis, Corynebacterium diphtheriae
E. *Clostridium perfringens, Bacteroides spp.
Indicate composition of semi-solid Hiss media
A. MPA, different carbohydrates (glucose, maltose, lactose, saccharose,
mannitol,
and other), indicator Andrede
B. MPA, different carbohydrates (glucose, maltose, lactose, saccharose,
mannitol,
and other), an indicator diamond green
C. MPA, different carbohydrates (glucose, maltose, lactose, saccharose,
mannitol,
and other), indicator – complex salt of ammonium-iron sulfate
D. MPA, different carbohydrates (glucose, maltose, lactose, saccharose,
mannitol,
and other), an indicator phenol red
E. *MPA, different carbohydrates (glucose, maltose, lactose, saccharose,
mannitol,
and other), indicators water blue with rosolic acid)
Indicate halophilic bacterium:
A. Streptococcus pyogenes
B. Staphylococcus aureus
C. Treponema pallidum
D. Shigella flexneri
E. *Vibrio parahaemolyticus
Indicate the basic components of Ploskirev medium.
A. MPB, saccharose, salts of bile acids, diamond green, neutral red.
B. MPA, saccharose, salts of bile acids, diamond green, neutral red.
C. MPB, lactose, salts of bile acids, diamond green, neutral red.
D. MPA, glucose, salts of bile acids, diamond green, neutral red.
E. *MPA, lactose, salts of bile acids, diamond green, neutral red.
Indicate the capneic bacteria:
A. Staphylococcus aureus
B. Streptococcus pyogenes
C. Lactobacillus spp.
D. Clostridium perfringens
E. Brucella abortus
Indicate the composition of blood agar.
A. MPA, defibrinated or fresh blood of animals, an indicator bromthymol blue
B. MPA, defibrinated or fresh blood of animals, an indicator fuchsin
C. MPA, defibrinated or fresh blood of animals, an indicator neutral red
D. MPA, defibrinated or fresh blood of animals, an indicator eosin
E. *MPA, defibrinated or fresh blood of animals
Indicate the composition of Kitt-Tarozzi medium for cultivation of anaerobes:
A. MPA, pieces of liver or meat
B. MPB, yolk, medium is covered with vaseline oil
536.
537.
538.
539.
540.
C. Hottinger broth, pieces of brain tissues
D. Serum broth, pieces of liver or meat, , medium is covered with vaseline
oil
E. *Hottinger broth, pieces of liver or meat, glucose, medium is covered with
vaseline oil
What are the main components of liquid Hiss media?
A. 1 % peptone water, different carbohydrates (glucose, maltose, lactose,
saccharose, Mannitol and other), an indicator bromthymol dark blue
B. 1 % peptone water, different carbohydrates (glucose, maltose, lactose,
saccharose, Mannitol and other), an indicator eosin
C. 1 % peptone water, different carbohydrates (glucose, maltose, lactose,
saccharose, Mannitol and other), an indicator is methylene blue
D. 1 % peptone water, different carbohydrates (glucose, maltose, lactose,
saccharose, Mannitol and other), an indicator neutral red
E. *1 % peptone water, different carbohydrates (glucose, maltose, lactose,
saccharose, Mannitol and other), indicator of Andrede
Indicate the composition of Olkenitsky medium?
A. MPA, lactose, complex salt of ammonium-iron sulfate (Moor’s salt), sodium
thiosulphate, urea, an indicator is phenol red
B. MPA, saccharose, complex salt of ammonium-iron sulfate (Moor’s salt),
sodium
thiosulphate, urea, an indicator is phenol red
C. MPA, glucose, complex salt of ammonium-iron sulfate (Moor’s salt), sodium
thiosulphate, urea, an indicator is phenol red
D. MPA, lactose, saccharose, glucose, complex salt of ammonium-iron sulfate
(Moor’s salt), sodium thiosulphate, an indicator is phenol red
E. *MPA, lactose, saccharose, glucose, complex salt of ammonium-iron sulfate
(Moor’s salt), sodium thiosulphate, urea, and an indicator is phenol red.
Indicate the correct sequence of the stages of isolation of pure culture of
aerobes.
A. Macroscopic and microscopic examination of tested material, inoculation on
the
slant agar for the obtaining the isolated colonies,obtaining the pure
culture,
identification, conclusion
B. Macroscopic and microscopic examination of tested material, inoculation in
MPB for the obtaining the isolated colonies, inoculation on the slant agar
for
obtaining the pure culture , identification, conclusion
C. Macroscopic and microscopic examination of tested material, inoculation on
a
solid nutrient medium for the obtaining the isolated colonies,
identification,
conclusion
D. Macroscopic and microscopic examination of tested material, inoculation on
a
solid nutrient medium for obtaining the isolated colonies ,inoculation on the
slant agar for obtaining a pure culture, identification
E. *Microscopic and microscopic examination of tested material, inoculation
on a
solid nutrient medium for obtaining the isolated colonies, inoculation on the
slant agar for obtaining the pure culture, identification, conclusion
Indicate the growth signs of bacteria in a liquid nutrient medium:
A. formation of diffuse turbidity
B. formation of pellicle
C. formation of sediment
D. all answers are false
E. *all answers are correct
Indicate the necessary components of Levin medium:
A. MPB, lactose, indicators methylene blue, eosin .
B. MPA, saccharose, indicators methylene blue, eosin.
C. MPB, saccharose, indicators methylene dark, eosin.
D. MPA, lactose, indicators methylene blue, fuchsine.
E. *MPA, lactose, indicators methylene blue, eosin.
541.
Indicate the necessary components of Levin medium:
A. MPB, lactose, indicators methylene blue, eosin .
B. MPA, saccharose, indicators methylene blue, eosin.
C. MPB, saccharose, indicators methylene dark, eosin.
D. MPA, lactose, indicators methylene blue, fuchsin.
E. *MPA, lactose, indicators methylene blue, eosin.
542.
Indicate vitamins necessary for bacterial growth:
A. Biotin
B. Thiamine
C. Riboflavin
D. all answers are false
E. *all answers are correct
543.
Is it necessary to examine the structure of colonies. What method can you
offer?
A. In an electron microscope
B. By a "hanging" drop technique
C. By a phase-contrast microscope
D. By the immersion system of microscope
E. In the passing light at the small increase of microscope
544.
It is necessary to check the quality of antibacterial filters. What
microorganism is
it possible to choose as a test-object?
A. Salmonella schottmuelleri
B. Staphylococcus aureus
C. Bordetella pertussis
D. Mycobacterium kansassii
E. *Pseudomonas aeruginosa
545.
It is necessary to check the quality of sterilization by compressed steam at
110
°С sterilization. What chemical indicator of sterilization quality can be
utilized?
A. antipyrine
B. sulphur
C. benzoic acid
D. urea
E. *benzonaphtolum
546.
It is necessary to conduct pasteurization of milk. Choose its optimum mode:
A. 70 °С during 60 min
B. 70 °С during 90 min
C. 60 °С during 30 min
D. 60 °С during 60 min
E. *70 °С during 30 min
547.
It is necessary to sterilize an object that destroys at 100 °С . Choose the
comfortable regime of tyndalization:
A. sterilization by water bath at 58-60 °С during a hour 2-3 days one after
the other
B. single sterilization by water bath at 98-100 °С during 10 min
C. sterilization by water bath at 58-60 °С during a hour 3-4 days one after
the other
D. sterilization by water bath at 80-90 °С during a hour 2-3 days one after
the other
E. *sterilization by water bath at 58-60 °С during a hour 5-6 day one after
the other
548.
It is necessary to sterilize material that contains the spore-forming
bacteria by
live steam. Choose the regime of sterilization:
A. single sterilization during 10 min.
B. single sterilization during 20 min.
C. fractional sterilization – three times one after the other during one day,
30 min.
D. fractional sterilization – 2 days one after the other, 30 min.
E. *fractional sterilization – 3 days one after the other, 30 min.
549.
It is necessary to sterilize the dressing material. Choose optimum regime of
sterilization in an autoclave:
A. 110 °С during 20 minutes
B. 127 °С during 20 minutes
C. 134 °С during 40 minutes
D. 100 °С during 20 minutes
E. *120 °С during 20 minutes
550.
It is necessary to study bacterial peptolytic properties. Indicate the proper
medium:
A. Meat-peptone agar
B. Sugar MPA
C. Coagulated serum
D. gelatin
E. *Meat-peptone broth
551.
Which of following is possible to sterilize in burner’s flame:
A. liquid nutrient media
B. physiological solution
C. solid nutrient media
D. catheters
E. *forceps
552.
Material from a patient with suspicion on dysenterywas inoculated on Endo
medium. What colour does colony of lactose-negative Shigella have?
A. dark blue
B. green
C. red with metallic hue
D. brown
E. *colourless
553.
What is the mechanism of antibacterial effect of 37-40 % formaldehyde
solution?
A. destruction of cellular lipids.
B. destruction of cellular endotoxin
C. destruction of cellular polysaccharides.
D. Cell dehydration
E. *Interaction with protein amino groups and their denaturation
554.
Mechanism of high temperatures antibacterial action:
A. damage of ribosomes
B. destruction of tertiary structure of albumens
C. destruction of cytoplasm membranes
D. all answers are false
E. *all answers are correct
555.
Most of non-spore-forming perish at a temperature:
A. 20-30 °С in 2-4 ч
B. 40-50 °С in 1 ч
C. 10-20 °С in 5-8 ч
D. 20-30 °С in 30-60 minutes
E. *58-60 °С 30-60 minutes
556.
Nitrogen-fixing bacteria belong to what group according to their type of
respiration?
A. Obligate aerobes
B. Obligate anaerobes
C. Facultative anaerobes
D. Capneic
E. *Microaerophils
557.
Non-inoculated Endo medium has colour:
A. bright red
B. yellow
C. greenish
D. colourless
E. *light rose
558.
On the mucous membranes of human there are corynebacteria. What selective
medium for their cultivation?
A. Mueller medium
B. 1 % alkaline peptone water
C. Ru medium
D. Saburo medium
E. *Agar with furazolidonum and tween
559.
One of the asepsis measures is disinfection. What is it – a concept
"disinfection"?
A. deleting bacterial and fungal spores of the objects of environment
B. elimination of pathogenic microorganisms in a wound
C. diminishing the degree of microbial contamination of skin and mucous
membranes
D. elimination viruses in the environment
E. *decontamination of pathogenic microorganisms
560.
Peculiarities of S-colonies of bacteria:
A. gyrose surface
B. fibred consistency
C. edges are rosette-like
D. have capsules
E. *smooth, convex surface
561.
Respiration of bacteria is a process:
A. biosynthesis of protein molecules
B. biosynthesis of carbohydrates
C. biosynthesis of lipids
D. biosynthesis of microelements
E. *biooxidation with formation of ATP
562.
Respiration of bacteria is accompanied:
A. by the loss of energy
B. by accumulation of CO2
C. all answers are correct
D. all answers are false
E. *by formation of energy
563.
Rubber and polymeric wares can be sterilized immersing them in:
A. solution of dexon on 25 min at 18 °С
B. solution of dexon on 15 min at 18 °С
C. solution of dexon on 45 min at 56 °С
D. solution of dexon on 25 min at 56 °С
E. *solution of dexon on 45 min at 18 °С
564.
Salts of heavy metals will cause in a bacterial cell:
A. destruction of lipids
B. destroying the polysaccharides
C. DNA destroying
D. Protein synthesis disturbances
565.
566.
567.
568.
569.
570.
571.
572.
573.
E. *coagulation of protein
Special nutrient medium is/are ___:
A. Blood agar
B. Serum agar
C. Ascitic agar
D. All answers are false
E. *All answers are correct
Specify, to what pressures in an autoclave a temperature 112 °С corresponds:
A. 0 аtm.
B. 1 аtm.
C. 1,5 аtm.
D. 2 аtm.
E. *0,5 аtm.
Specify, to what pressures in an autoclave a temperature 121 °С corresponds:
A. 0 аtm.
B. 0,5 аtm.
C. 1,5 аtm.
D. 2 аtm.
E. *1 аtm.
Specify, to what pressures in an autoclave a temperature 127 °С corresponds:
A. 0 аtm.
B. 0,5 аtm.
C. 1 аtm.
D. *1,5 аtm.
E. 2 аtm.
Specify, to what pressures in an autoclave a temperature 131 °С corresponds:
A. 0 аtm.
B. 0,5 аtm.
C. 1 аtm.
D. 1,5 аtm.
E. *2 аtm.
Sterilization may be conducted by the following methods:
A. action of moist steam
B. filtration
C. radiation
D. pasteurization
E. *all answers are correct
Such enzymes prevail in a bacterial cell:
A. oxydoreductases
B. constitutive
C. hydrolases
D. adaptive
E. ligases
Such groups of bacteria are existed according to their respiration:
A. capneic
B. microaerophils
C. anaerobes
D. aerobes
E. all answers are correct
What is the composition of Kitt-Tarozzi medium for cultivation of anaerobes?
A. MPA, pieces of liver or meat
B. MPB, yolk, medium is covered with vaseline oil
C. Hottinger broth, pieces of brain tissues
D. Serum broth, pieces of liver or meat, , medium is covered with vaseline
oil
E. *Hottinger broth, pieces of liver or meat, glucose, medium is covered with
vaseline oil
574.
Such nutrient media belong to universal ones:
A. Mannitol salt agar Levin medium
B. Serum MPA and 1% alkaline peptone water
C. Endo medium and blood agar
D. Ploskirev medium and serum broth
E. *MPB and MPA
575.
Surface-tension-reducing agents (detergents) can cause:
A. violation of spores’ formation.
B. violation of flagella structure
C. violation of capsule formation.
D. Violation of nucleoid function
E. *violation of CPM structure and cell wall structure
576.
Synthesis of what cell molecules needs the most of energy?
A. nucleic acids
B. lipids
C. polysaccharides
D. amino acid
E. *proteins
577.
Taking into consideration the features of fermenting type of anaerobe
bacteria
metabolism they require:
A. More rich for nutrients medium
B. More rich for vitamins nutrient medium
C. Obligatory addition of bacterial growth factors in nutrient medium
D. All answers are false
E. *All answers are correct
578.
The optimum temperature for mesophilic bacteria is ____:
A. 50-60 ?C
B. 10-20 ?C
C. 20-30 ?C
D. 40-50 ?C
E. *30-37 ?C
579.
The optimum temperature for psychrophilic bacteria is ____:
A. 50-60 ?C
B. 20-30 ?C
C. 30-37 ?C
D. 40-50 ?C
E. *10-20 ?C
580.
The optimum temperature for thermophilic bacteria is ____:
A. 10-20 ?C
B. 20-30 ?C
C. 30-37 ?C
D. 40-50 ?C
E. *50-60 ?C
581.
The mechanism of action of iodine consists of:
A. destruction of lipids
B. destroying the polysaccharides
C. DNA depolarization
D. violation of proteins synthesis
E. *oxidization of protein active groups and their denaturizing
582.
The second stage of isolation of pure culture of aerobes is:
A. Macroscopic examination of material and its inoculation in liquid nutrient
medium
B. Macro- and microscopic verification of culture purity
C. Serological identification
D. Biochemical identification
E. *Macro- and microscopic study of colonies and their inoculation them on
slant
agar
583.
The structure of colonies can be examined by such method:
A. In an electronic microscope
B. By a "hanging" drop technique
C. In a phase-contrast microscope
D. By the immersion system of microscope
E. *In the passing light, small objective of microscope
584.
There are such components of Endo medium:
A. MPB, 1 % lactose, an indicator fuchsine
B. MPA, 1 % lactose, an indicator eosin
C. MPB, 1 % lactose, an indicator eosin
D. MPA, 5 % lactose, indicator bromthymol blue
E. *MPA, 1 % lactose, an indicator fuchsine
585.
There are such demands to the nutrient media, EXCEPT:
A. presence of nutrients
B. sterility
C. certain viscidity
D. transparency
E. certain colour
586.
Which of following demands is not nessesary to the nutrient media?
A. presence of nutrients
B. sterility
C. certain viscidity
D. transparency
E. certain colour
587.
There are such mechanisms of penetration different substances into the cell,
EXCEPT:
A. facilitated diffusion
B. passive diffusion
C. active transport
D. ionic transport
E. transformations of chemical groups
588.
To dyes, which can inhibit bacterial growth , belong:
A. diamond green.
B. rivanol.
C. acriflavin.
D. all answer are false.
E. *all answer are right.
589.
Wares from polymeric materials may be sterilized by such method:
A. mechanical
B. biological
C. pasteurization
D. in a heat oven
E. *chemical
590.
Wares from rubber and polymeric materials may be sterilized by immersing
them in:
A. 3 % hydrogen peroxide, 6 hours at 18 °С
B. 6 % hydrogen peroxide, 12 hours at 18 °С
C. 6 % hydrogen peroxide on 1 hour at 18 °С
D. 6 % hydrogen peroxide on 2 hour at 18 °С
E. *6 % hydrogen peroxide on 6 hours at 18 °С
591.
What is tyndalization?
A. single sterilization by water bath at 98-100 °С during 30 minutes
B. sterilization by water bath at 98-100 °С during a hour 5-6 day one after
the
other
C. sterilization by water bath 58-60 °С during a hour 2-3 days one after the
other
D. sterilization by water bath at 98-100 °С during a hour 5-6 days one after
the
other
E. *sterilization by water bath at 58-60 °С during a hour 5-6 day one after
the
other
592.
What microorganisms can be used for quality control of sterilization by
filtration?
A. Escherichia coli
B. Salmonella typhi
C. Clostridium perfringens
D. Corynebacterium xerosis
E. *Serratia mаrcescens
593.
What actions are necessary to do during І stage of isolation of pure cultures
of
aerobes?
A. To study the features of the tested material
B. To study the morphological features of possible causative agent
C. To choose necessary nutrient medium for inoculation
D. To inoculate tested material for obtaining the isolated colonies
E. *All answers are correct
594.
What antibiotic is it necessary to add into nutrient medium for inhibition of
fungi
growth?
A. Penicillin
B. Streptomycin
C. Ciprofloxacin
D. Mitomicin C
E. *Nistatin
595.
What bacteria are very susceptible to drying?
A. Staphylococci
B. Mycobacterium tuberculosis
C. Shigella
D. Salmonella
E. *Meningococci
596.
What bacteria do belong to obligate anaerobes?
A. Salmonella schottmuelleri
B. Staphylococcus aureus
C. Proteus rettgeri
D. Francisella tularensis
E. *Clostridium septicum, Bacteroides fragilis
597.
What bacterial colony?
A. Medium which is inoculated with bacteria, where different microorganisms
grow
B. Growth of microbes of one species on different nutrient medium
598.
599.
600.
601.
602.
603.
604.
605.
C. Visible growth of bacteria in a liquid nutrient medium
D. Visible growth of bacteria on the slant meat-peptone agar
E. Macroscopically visible growth of microbes (posterity of one bacterial
cell) on a
solid nutrient medium
When microbes utilize sugars Hiss media turn to ____ colour.
A. green
B. violet
C. brown
D. a colour does not change
E. *red
What colour does lactose-negative colony of bacteria have on Levin medium?
A. dark blue
B. red with metallic hue
C. brown
D. rose
E. *colourless
What colour does lactose-positive colony of bacteria have on Levin medium?
A. colourless
B. red with metallic hue
C. brown
D. green
E. *dark blue
What colour does a simple medium – MPA– have?
A. red
B. greenish
C. violet
D. dark blue
E. *yellow
What colour does a simple medium – MPB– have?
A. red
B. dark blue
C. greenish
D. violet
E. *yellow
What colour does noninoculated Levin medium have?
A. bright red
B. poorly-rose
C. yellow
D. greenish
E. *violet
What colour does non-inoculated Olkenitsky medium have?
A. bright red
B. pink
C. *yellow
D. greenish
E. violet
What colour does noninoculated Ressel’s medium have?
A. bright red
B. greenish
C. violet
D. pinky
E. *yellow
606.
What colour is acquired by Hiss media, if microbes utilize sugars in it?
A. green
B. violet
C. brown
D. a colour does not change
E. *yellow
607.
What components may be the source of nitrogen in nutrient media?
A. meat
B. fish
C. meat and bone flour
D. casein
E. *all answers are correct
608.
What are the basic components of Ploskirev medium.
A. MPB, saccharose, salts of bile acids, diamond green, neutral red.
B. MPA, saccharose, salts of bile acids, diamond green, neutral red.
C. MPB, lactose, salts of bile acids, diamond green, neutral red.
D. MPA, glucose, salts of bile acids, diamond green, neutral red.
E. *MPA, lactose, salts of bile acids, diamond green, neutral red.
609.
What enzyme of aerobic bacteria does provide neutralization of hydrogen
peroxide toxic effect?
A. lipase
B. protease
C. hydrolase
D. urease
E. *catalase
610.
What enzymes do control the substrate splitting with joining the water
molecules?
A. Oxydoreductases
B. Isomerases
C. Ligases
D. Transferases
E. Hydrolases
611.
What enzymes do provide oxydation-reduction reactions?
A. Hydrolases
B. Isomerases
C. Transferases
D. Lyases
E. Oxydoreductases
612.
What are the growth signs of bacteria in a liquid nutrient medium:
A. formation of diffuse turbidity
B. formation of pellicle
C. formation of sediment
D. all answers are false
E. *all answers are correct
613.
What enzymes do provide the reactions of groups of atoms transfer?
A. Ligases
B. Lyases
C. Oxydoreductases
D. Isomerases
E. Transferases
614.
What enzymes localized in cytoplasm membrane do provide the facilitated
diffusion?
A.
B.
C.
D.
E.
transferases
proteases
hydrolases
ligases
*permeases
615.
What factors do not influence on the bacterial reproduction?
A. age of bacteria
B. composition of nutrient medium
C. medium рН
D. temperatures, aeration
E. *year season
616.
What finished products does appear in the semi-solid Hiss media after sugar
utilization?
A. acid
B. alkali
C. gas
D. fixing the finished products is not succeeded
E. *acid and gas
617.
What from the listed words is a name of enzyme?
A. Saccharose
B. Glucose
C. Maltose
D. Galactose
E. Saccharase
618.
What group of nutrient media does Ressel medium belong to?
A. Universal
B. Liquid
C. Selective
D. Elective
E. *Differential-diagnostic
619.
What groups of methods for creation of anaerobic conditions for anaerobic
bacteria cultivation do you know?
A. mechanical
B. physical
C. chemical
D. biological
E. *all answers are correct
620.
What are the necessary components of Levin medium:
A. MPB, lactose, indicators methylene blue, eosin .
B. MPA, saccharose, indicators methylene blue, eosin.
C. MPB, saccharose, indicators methylene dark, eosin.
D. MPA, lactose, indicators methylene blue, fuchsin.
E. *MPA, lactose, indicators methylene blue, eosin.
621.
What is a biological catalyst?
A. nucleic acids
B. metabolites
C. Plasmids
D. Transposons
E. enzymes
622.
What is a chemical origin of agar-agar?
A. Lipids
B. Proteins
C. Carbohydrates
D. Amino acids
E. Polysaccharides
623.
Which of following vitamins are necessary for bacterial growth:
A. Biotin
B. Thiamine
C. Riboflavin
D. all answers are false
E. *all answers are correct
624.
What is a chemical origin of gelatin?
A. Polysaccharides
B. Lipids
C. Proteins
D. Carbohydrates
E. Amino acids
625.
Choose among these media differential ones:
A. MPA
B. Kitt-Tarozzi medium
C. MPB
D. Sugar meat peptone broth
E. *Hiss media
626.
What is a name of enzyme localized inside a bacterial cell:
A. adaptive
B. exoenzymes
C. constitutive
D. all answers are right
E. *endoenzymes
627.
What is a reason molecular oxygen toxic action according to bacteria?
A. formation of singlet oxygen
B. formation of hydrogen peroxide
C. formation of ozone
D. all answers are incorrect
E. all answers are right
628.
What is advantage of Koch, Drigalsky and streak techniques above the Pasteur
method?
A. Enable to study biochemical properties of bacteria
B. Enable to study antigen properties of bacteria
C. Enable to obtain a pure culture
D. Right answer is not present
E. *Enable to obtain the isolated colonies
629.
What is an essence of Koch technique for isolation of pure cultures?
A. *Use of solid nutrient media (gelatin) for obtaining the isolated colonies
B. Use of liquid nutrient media for obtaining the isolated colonies
C. Use of blood agar for examination of hemolysines types
D. Use of solid nutrient media for examination of antibiotics susceptibility
E. Use liquid nutrient media for examination of antibiotics susceptibility
630.
What is an essence of Pasteur technique for isolation of pure cultures?
A. Dilution of tested material in gelatin
B. Dilution of tested material in MPA
C. Dilution of tested material in the AGV nutrient medium
D. Dilution of tested material in a solid nutrient medium
E. *Dilution of tested material in a liquid nutrient medium
What is an essence of ІV stage of isolation of pure culture of anaerobic
bacteria?
A. Inoculation of colonies on the proper nutrient medium
B. Examination of tinctorial properties of bacteria
C. Examination of serologic features of bacteria which form colonies by
precipitation test
D. Right answer is absent
E. *Checking the culture purity and its identification
632.
What is biochemical identification of bacteria?
A. Verification of bacteria according to their ability to produce
dehydrogenases
B. Verification of bacteria according to their ability to produce hemolysines
C. Verification of bacteria according to their tinctorial properties
D. Verification of bacteria according to their reduce properties
E. *Verification of bacteria according to their biochemical properties
633.
What is chemical composition of prokaryotic cell?
A. Water - 50-60 %, dry weight - 40-60 %.
B. Water - 60-90 %, dry weight - 10-30 %.
C. Water - 40-50 %, dry weight - 50-60 %.
D. Water - 20-40 %, dry weight - 60-80 %.
E. Water - 70-90 %, dry weight - 10-30 %.
634.
What is a chemical origin of gelatin?
A. Polysaccharides
B. Lipids
C. Carbohydrates
D. Amino acids
E. Proteins
635.
What is chemostate?
A. synonym of term of «jar»
B. vehicle which allows to study chemical properties of bacteria
C. vehicle, in which it is possible to make sterilization of media
D. vehicle which allows to study antigenic properties of bacteria
E. *vehicle which allows to make continuous cultivation of microorganisms in
laboratory conditions
636.
What is disinfection?
A. Complex measures for complete, partial or selective elimination of
potential
pathogens for human in the air for warning transmission of causative agents
from the source of infection to the receptive organism
B. Complex of measures for complete, partial or selective elimination of
potential
pathogens for human in his body for warning transmission of causative agents
from the source of infection to the receptive organism
C. Complex of measures which prevent spread of potential pathogens from human
body in an medium
D. Complex of measures for complete, partial or selective elimination of
potential
pathogens for human in the water for warning transmission of causative agents
from the source of infection to the receptive organism
E. *Complex of measures for complete, partial or selective elimination of
potentially pathogenic for human causative agents on the different objects of
medium for warning transmission of causative agents from the source of
infection to the receptive organism
637.
What is it necessary to do during the second stage of isolation of pure
culture of
aerobes?
A. Study the features of colonies
B. Make a smear from colonies
631.
C. Inoculation onto slant agar
D. All answers are not correct
E. *All answers are correct
638.
What is raw material for agar-agar obtaining?
A. Skin
B. Meat
C. Arthral cartilage
D. Bones
E. Algae
639.
What is the essence of ІІІ stage of isolation of pure cultures of aerobes?
A. Examination the isolated colonies
B. Examination bacterial growth in the column of agar
C. Bacterial phage typing
D. Examination of ability for bacteriocine production
E. *Identifications of the unknown bacteria
640.
What is the main demand to any nutrient medium used for the primary selection
of microorganisms?
A. To be solid
B. To be liquid
C. To contain vitamins
D. To contain growth factors
E. *To be sterile
641.
What is the main idea of ІІІ stage of isolation of pure culture of anaerobic
bacteria?
A. Examination of features of pure culture and its identification
B. Examination of features of colonies, preparing the smear from them,
inoculation
of colonies on slant MPA for obtaining the pure culture
C. Examination of features of colonies, preparing the smear from them,
inoculation
of colonies in MPB for obtaining the pure culture
D. Examination of features of colonies, inoculation of colonies in KittTarozzi
medium for obtaining the pure culture
E. *Examination of features of colonies, preparing the smear from them ,
inoculation of colonies into Kitt-Tarozzi medium for obtaining the pure
culture
642.
What is the minimum area of washings for control of disinfection quality?
A. not less than, 50 см2.
B. not less than, 100 см2.
C. not less than, 150 см2.
D. not less than, 250 см2.
E. *not less than, 200 см2.
643.
What a main aim of the first stage of isolation of pure cultures of aerobes?
A. To study tested material
B. To choose nutrient medium for inoculation
C. To study the morphological features of possible causative agent
D. To study the antigen features of causative agent
E. *To obtain the isolated colonies
644.
What mechanism of action of ultraviolet light against bacteria?
A. does not have any effect according to the cell
B. affect RNA molecules
C. affect the ribosomes
D. affect the structure of cell wall
E. *affect DNA molecules and as a result forms thymine dimers
645.
What mechanism of ultrasound action against bacteria?
A.
B.
C.
D.
E.
destroys cell DNA
destroys cell RNA
destroys cell mesosome
an ultrasound does not influence on bacteria
*forms a cavitation cavity, filled in the pair of liquid, with high pressure
inside;
cytoplasm structures are destroyed
646.
What is mechanism of bacterial reproduction?
A. by a mitosis
B. by spore formation
C. by sexual way
D. by a disjunctive way
E. by a simple transversal division
647.
What medium is it possible to use for checking the utilization of a few
sugars?
A. Ru medium
B. Leffler medium
C. *Hiss media
D. Ploskirev medium
E. Endo medium
648.
What methods of isolation of pure cultures, based on mechanical principle do
you know ?
A. Pasteur method
B. Koch method
C. Streak method
D. All answers are incorrect
E. *All answers are correct
649.
What methods of isolation of pure cultures, based on mechanical principle do
you know?
A. Leffler method
B. Grey method
C. Buchin method
D. Nemchinov method
E. *Streak method
650.
What microbes belong facultative anaerobes to?
A. Clostridium tetani, Clostridium perfringens
B. Mycobacterium tuberculosis, Vibrio cholerae
C. Brucella abortus
D. Lactobacillus acidophilus
E. *Corynebacterium diphtheriae, Salmonella typhi
651.
What part of dry weight of the cell does belong to protein?
A. 15 %.
B. 35 %.
C. 75 %.
D. 95 %
E. 55 %.
652.
What phase is characterized by high speed of bacterial division and
increasing
their number?
A. Stationary
B. Initial
C. Death
D. Expotential
E. Exponential
653.
What phase of microbial growth in periodic culture does not exist?
A.
B.
C.
D.
E.
Initial
Exponential
Stationary
Decline
Accumulations
654.
What properties of colonies are studied for cultural identification?
A. Shape, colour
B. Character of edges
C. Character of surface of colonies
D. All answers are false
E. *All answers are correct
655.
What signs of hemolytic activity of bacteria are on blood MPA?
A. appearance of areas of darkening on a mat red background
B. the changes of medium colour
C. appearance of bright red colonies of microbes, if they cause haemolysis
D. appearance of colourless colonies
E. *appearance of colourless areas surround the colonies on mat red
background if
the have hemolytic activity
656.
What stage of meat water preparing is the most important?
A. Making meat small
B. Filtration
C. Boiling
D. Sterilization
E. Extraction
657.
What substance must be added into nutrient medium for examination of
bacterial
proteolytic properties?
A. Agar-agar
B. Agarose
C. All answers are correct
D. All answers are false
E. *Gelatin
658.
What substances can repress the action of inhibitors of bacterial growth and
toxin formation?
A. some aminoacids
B. tweens
C. active carbon
D. all answers are false
E. *all answers are correct
659.
What is optimum temperature for mesophilic bacteria?
A. 10-15 ?C
B. 20-30 ??C
C. 40-50 ? C
D. 50-60 ? C
E. *30-37 ? С
660.
What is optimum temperature for psychrophilic bacteria?
A. 20-30 ? C
B. 30-37 ? С
C. 40-50 ? C
D. 50-60 ? C
E. *10-25 ? C
661.
What is optimum for thermophilic bacteria?
A. 10-20 ? C
B.
C.
D.
E.
20-30 ? C
30-37 ? С
40-50 ? C
*50-60 ? C
662.
What substitutes of meat are used for making the nutrient media?
A. placenta
B. blood clot
C. yeast
D. all answers are false
E. *all answers are correct
663.
What substrate does provide a necessary density of nutrient medium?
A. Protein
B. Lipids
C. Carbohydrates
D. No correct answer
E. *Agar-agar
664.
Which of following colour does lactose-positive colony of bacteria have on
Ploskirev medium?
A. dark blue
B. rose
C. brown
D. green
E. *colourless
665.
Which of following colour does lactose-negative colony of bacteria form on
Ploskirev medium?
A. dark blue
B. rose
C. brown
D. green
E. *colourless
666.
What technique does belong to the biological method of anaerobic conditions
creation?
A. Cultivation of anaerobes in the organism of laboratory animals
B. Cultivation of anaerobes in the chicken embryos
C. All answers are correct
D. All answers are false
E. *Joint cultivation of anaerobic bacteria and Serratia marcescens, which
utilizes
an oxygen intensively
667.
What technique does belong to the chemical method of anaerobic conditions
creation?
A. use of Gas generating box
B. application of pyrogallol on the bottom of the jar
C. use the high column of nutrient medium
D. All answers are false
E. *All answers are correct
668.
Choose among following a colour of MPB?
A. red
B. dark blue
C. greenish
D. violet
E. *yellow
669.
Choose among following a colour of noninoculated Levin medium.
A.
B.
C.
D.
E.
bright red
poorly-rose
yellow
greenish
*violet
670.
What type of sediment may be formed by microbes in liquid nutrient media?
A. Crumble-like
B. homogeneous
C. viscid
D. mucous
E. *all answers are correct
671.
Which of following demands is not nessesary to the nutrient media?
A. presence of nutrients
B. sterility
C. certain viscidity
D. transparency
E. certain colour
672.
What types of bacterial fermentation do you know?
A. Lactic-acid fermentation
B. alcoholic fermentation
C. butiric-acid fermentation
D. all answers are false
E. *all answers are correct
673.
What vitamins are necessary for development of bacteria?
A. pantothenic acid
B. Cholin
C. nicotine acid
D. all answers of incorrect
E. *all answers are correct
674.
Where does point of replication localize, when the division of bacteria
begins?
A. on DNA of cell
B. on мРNA
C. on a cytoplasm membrane
D. on a cellular wall
E. *on DNA, located in the place of connection of mesosome and cytoplasm
membrane
675.
Who was discoverer of phenomenon of anaerobiosis?
A. Mechnikov
B. S. Vinogradsky
C. R. Koch
D. E. Ru
E. L. Pasteur
676.
What is the optimum temperature for mesophilic bacteria?
A. 10-15 ?C
B. 20-30 ?C
C. 40-50 ?C
D. 50-60 ?C
E. *30-37 ?С
677.
A nurse has to sterilize surgical tools for an operation. What the most
effective
method can you offer?
A. fractional sterilization
B. sterilization by pressed steam
C. boiling
D. ionizing radiation
E. by dry heat
678.
According to the colony relief there may have such form:
A. Flat
B. Convex
C. Dome-shaped
D. Pitted
E. *All answers are correct
679.
According to the source of energy there are such bacteria:
A. lithotrophs and organotrophs
B. chemotrophs and heterotrophs
C. autotrophs and chemotrophs
D. autotrophs and heterotrophs
E. chemotrophs and phototrophs
680.
All nutrient media are elective, EXCEPT:
A. Mueller medium
B. Ru medium
C. Leffler medium
D. 1 % alkaline peptone water
E. blood MPA
681.
All of bacteria belong to obligate aerobes, EXCEPT:
A. Mycobacterium tuberculosis
B. Yersinia pestis
C. Micrococci spp.
D. Vibrio cholerae
E. Clostridium tetani
682.
How you can decribe bacterial S-colonies?
A. gyrose surface
B. fibred consistency
C. edges are rosette-like
D. have capsules
E. *smooth, convex surface
683.
All of the listed microorganisms are facultative anaerobes, EXCEPT:
A. Escherichia spp.
B. Salmonella spp.
C. Shigella spp.
D. Staphylococci spp.
E. *Clostridium botulinum
684.
Anaerobic microorganisms may be isolated by:
A. Fortner method
B. Shukevich method
C. Pasteur method
D. Leffler method
E. *Veynberg method
685.
At what advantage of Koch method in comparing Pasteur one for isolation of
pure cultures?
A. Enables to examine antibiotics susceptibility
B. Enables to examine biochemical properties of microbes
C. Enables to examine antigen properties of microbes
D. Has no substantial advantage
E. *Enables to obtain the isolated colonies
686.
A.
B.
C.
D.
E.
687.
A.
B.
C.
D.
E.
688.
A.
B.
C.
D.
E.
689.
A.
B.
C.
D.
E.
690.
A.
B.
C.
D.
E.
691.
A.
B.
C.
D.
E.
692.
A.
B.
C.
D.
E.
693.
A.
B.
C.
D.
E.
694.
A.
B.
C.
At what temperature agar-agar melts and becomes solid?
90 °С and 70 °С
80 °С and 60 °С
60 °С and 30 °С
50 °С and 20 °С
*100 °С and 40 °С
Bacteria that form R-colonies have all features, EXCEPT:
have capsules
sometimes does not have flagella
low or absent virulence
easily phagocytable
*does not have capsules
Bacteria that form S-colonies have all features, EXCEPT:
poorly phagocytable
biochemically more active
isolated in the acute stage of disease
have capsules
*easily phagocytable
What nutrient medium is not elective?
Mueller medium
Ru medium
Leffler medium
1 % alkaline peptone water
blood MPA
Bacteria, which form R-colonies, have all features, EXCEPT:
have capsules
sometimes does not have flagella
low or absent virulence
easily phagocytable
*does not have capsules
Bacteria, which form S-colonies, have all features, EXCEPT:
*easily phagocytable
poorly phagocytable
biochemically more active
isolated in the acute stage of disease
have capsules
Choose a correct answer according to the types of microbial decontamination:
sterilization
disinfections
antiseptics
all answer are false
*all answer are right
Choose among listed media, differential diagnostic ones.
MPA
Kitt-Tarozzi medium
MPB
glucose MPB
*Hiss media
Indicate vitamins necessary for bacterial growth:
Biotin
Thiamine
Riboflavin
695.
696.
697.
698.
699.
700.
701.
702.
703.
D. all answers are false
E. *all answers are correct
What are differential media used for?
A. Examination of antibiotics susceptibility of bacteria
B. Accumulations of microbial biomass
C. Examination of microorganisms pathogenicity
D. Studies of antigen structure of microorganisms
E. *Examination of bacteria enzyme activity
Choose bacteria susceptible to desiccation.
A. Vibrio cholerae
B. Shigella
C. Corynebacterium diphtheriae
D. Salmonella typhi
E. *Gononococci
Choose capneic bacteria:
A. Shigella dysentery
B. Clostridium botulinum
C. Mycobacterium tuberculosis
D. Corynebacterium diphtheriae
E. *Brucella abortus
Choose faithful composition of yolk salt agar
A. MPA, ram’s , 9 % of sodium chloride
B. MPA, rabbit’s red cells, 9 % of sodium chloride
C. MPA, horse’s red cells , 9 % of sodium chloride
D. MPA, glucose, 9 % chloride of sodium
E. *MPA, yolk, 9 % of sodium chloride
Choose among the folowing microaerophils:
A. Clostridium tetani
B. Escherichia coli
C. Salmonella typhi
D. Salmonella paratyphi A
E. *Lactobacillus acidophilus
Choose obligate aerobes among these microorganisms:
A. Neisseria gonorrhoeae, Streptococcus pneumoniae
B. Clostridium tetani, Clostridium botulini
C. Shigella dysenteriae, Salmonella typhi
D. Bacillus anthracis, Brucella melitensis
E. *Mycobacterium tuberculosis, Micrococcus spp.
Choose obligate anaerobes among these microbes:
A. Bacillus anthracis
B. Mycobacterium tuberculosis
C. Rickettsia spp.
D. Corynebacterium spp.
E. *Bacteroides spp.
Choose obligate anaerobes:
A. Escherichia coli
B. Salmonella typhi
C. Proteus vulgaris
D. Brucella melitensis
E. *Clostridium tetani, Clostridium botulinum
Choose the components of Ressel’s medium:
704.
to:
A. MPB, lactose (1 %), glucose (0,1 %) and indicator WR (water blue and
rosolic
acid)
B. MPB, lactose (1 %), glucose (0,1 %) and indicator fuchsine
C. semi-solid agar, lactose (1 %), maltose (0,1 %) and indicator WR (water
blue
and rosolic acid)
D. semi-solid agar, mannitol (1 %), glucose (0,1 %) and indicator WR (water
blue
and rosolic acid)
E. *semi-solid agar, lactose (1 %), glucose (0,1 %) and indicator of WR
(water blue
and rosolic acid)
Colonies of bacteria that grow on solid nutrient media can differentiate
according
A.
B.
C.
D.
E.
consistencies
density
colour
all answers are false
*all answers are correct
705.
According what property/ies bacterial colonies grown on solid nutrient media
can differentiate to:
A. size
B. density
C. colour
D. all answers are false
E. *all answers are correct
706.
Differential diagnostic media are used for:
A. Examination of antibiotics susceptibility of bacteria
B. Accumulations of microbial biomass
C. Examination of microorganisms pathogenicity
D. Studies of antigen structure of microorganisms
E. *Examination of bacteria enzyme activity
707.
During a Log-phase the division of bacteria takes place:
A. delay of division
B. more bacteria die, then appears
C. division does not take place
D. all answers are incorrect
E. *at constant rate
708.
During the evolution bacteria produced the ways of energy reception. There
are:
A. fermentation
B. respiration
C. photosynthesis
D. all answers are false
E. *all answers are correct
709.
All techniques belong to the chemical method of anaerobic conditions
creation,
ECXEPT:
A. use of Gas generating box
B. application of pyrogallol on the bottom of the jar
C. use the high column of nutrient medium
D. All answers are correct
E. *introduction of pieces of liver or muscles and vitamin K3 as supplement
in the
medium
710.
During the stationary phase of population growth the reason of bacterial
death is:
A. diminishing amount of nutrients
B. changes of medium рН
C. toxic action of appearing substances
D. all answers are false
E. *all answers are correct
711.
Efficiency of sterilization at heating is characterized the D index. What is
it?
A. time (minutes), which is necessary at certain temperature to get twofold
diminishing the bacterial population
B. time (minutes), which is necessary at certain temperature, to get fourfold
diminishing the bacterial population
C. time (minutes), which is necessary at certain temperature, to get fivefold
diminishing the bacterial population
D. time (minutes), which is necessary at certain temperature to destroy all
bacteria
E. *time (minutes) which is necessary at certain temperature to get the
tenfold
diminishing the bacterial population
712.
Endo medium consist of:
A. MPB, 1 % lactose, an indicator fuchsin
B. MPA, 1 % lactose, an indicator eosin
C. MPB, 1 % lactose, an indicator eosin
D. MPA, 5 % lactose, indicator bromthymol blue
E. *MPA, 1 % lactose, an indicator fuchsin
713.
Enzymes that are produced outside from bacteria are called:
A. endoenzymes
B. heteroenzymes
C. isoenzymes
D. all answers are correct
E. exoenzymes
714.
Ethylene oxide may be used for sterilization of all equipment, EXCEPT:
A. equipment for anesthesia
B. prostheses
C. endoscope instruments
D. catgut
E. *nutrient media
715.
For chemical sterilization by gases all of them are used, EXCEPT:
A. formaldehyde
B. chloroform
C. beta-propiolactone
D. ethylene oxide
E. *methane
716.
For examination of lipolytic properties of microbes lipids and lipid-like
substances – tweens – are supplemented into the medium. According to what
signs is it
possible to check the lipolytic activity of bacteria?
A. The colonies of bacteria lose their typical form
B. The colonies of bacterium are painted in the colour of indicator
C. The colonies of bacteria become semilucent
D. If microbes lipolytic properties they do not grow on an medium
E. *Bacteria form iridescent halos round the colonies
717.
For examination of saccharolytic properties of bacteria we can use such
medium:
A. Veinberg medium
B. Zeissler medium
C. Kitt-Tarozzi medium
D. Milk
E. *Hiss media
718.
For inoculation of microbes by Streak technique we can use:
A.
B.
C.
D.
E.
Spatula
Bacteriological needle
All answers are false
All answers are correct
*Bacteriological loop
719.
For mechanical separation of bacteria is not used such method:
A. Pasteur serial dilution
B. Koch method
C. Drigalsky method
D. Streak method
E. *Leffler method
720.
For obtaining the isolated colonies of aerobic bacteria tested material is
inoculated ______:
A. in liquid nutrient media
B. in laboratory animals
C. in chicken embryo
D. in cell cultures
E. *on solid nutrient media
721.
For sterilization of glassware utilize a heat oven is used. Choose the best
regime
of sterilization:
A. 180 ?C, 15-30 minutes
B. 160 ?C, 45-60 minutes
C. 160 ?C, 15-30 minutes
D. 180 ?C, 120 minutes
E. *180 ?C, 45-60 minutes
722.
For verifying the peptolytic properties of bacteria what substances are
examined
in nutrient medium?
A. carbon acid and water
B. glucose and lactose
C. carbon acid and nitrogen
D. mannitol and methanol
E. *indole and hydrogen sulphide
723.
Free oxygen is toxic for bacteria, because it:
A. kills a bacterium
B. halts spore formation
C. inhibit toxins production
D. inhibit enzymes production
E. *detains their growth
724.
Heat oven may be used for sterilization of all objects, EXCEPT:
A. tubes
B. Petry plates
C. metallic instruments
D. heat-resistant powders
E. *rubber corks
725.
How is it possible to carry out control of sterility of nutrient media after
their
preparing?
A. By keeping in incubator at 37 °С for a few hours.
B. By keeping in incubator at a temperature 42 °С for a few hours
C. By keeping at 18-20 °С. For a few hours
D. Right answer is not present
E. *By keeping in incubator for a few days at 37 °С.
726.
How is it possible to separate microorganisms according to their motility?
A.
B.
C.
D.
To inoculate tested material on the upper part of slant agar
To do inoculation of tested material by a prick in the slant agar
To separate microorganisms is impossible
To add to the tested material some substance which cause coagulation flagella
protein
E. *To inoculate tested material in the drop of fluid in the lower part of
slant agar
727.
How is it possible to verify motility of bacteria that form a colony?
A. By Grey’s technique
B. Staining according to Zdrodovsky’s technique
C. By scanning microscope
D. By radioisotope method
E. *By wet-mount technique
728.
How many classes of bacterial enzymes do you know?
A. 3
B. 4
C. 5
D. 2
E. *6
729.
How much time is it necessary to cultivate the bacteria on Bismuth-sulfite
agar
for final evaluation of growth?
A. 12 hours
B. 24 hours
C. 36 hours
D. 96 hour
E. *48 hours
730.
What are the basic components of Ploskirev medium.
A. MPB, saccharose, salts of bile acids, diamond green, neutral red.
B. MPA, saccharose, salts of bile acids, diamond green, neutral red.
C. MPB, lactose, salts of bile acids, diamond green, neutral red.
D. MPA, glucose, salts of bile acids, diamond green, neutral red.
E. *MPA, lactose, salts of bile acids, diamond green, neutral red.
731.
In an autoclave at 112 °С nutrient media were sterilized. What test-microbe
is it
possible to utilize for sterilization control?
A. Bacillus cereus
B. Bacillus subtilis
C. Clostridium perfringens
D. Clostridium difficile
E. *Clostridium sporogenes
732.
In an autoclave at 121 °С nutrient media were sterilized. What test-microbe
is it
possible to utilize for sterilization control?
A. Bacillus cereus
B. Bacillus subtilis
C. Clostridium perfringens
D. Clostridium difficile
E. *Bacillus steatothermophilus
733.
In burner’s flame it is possible to sterilize everything, EXCEPT:
A. forceps
B. bacteriological loops
C. slide glasses
D. cover slip
E. *tampons for material collection
734.
In depending on substrate that is donor of electrons there are such bacteria:
A.
B.
C.
D.
E.
lithotrophs and chemotrophs
chemotrophs and heterotrophs
autotrophs and chemotrophs
chemotrophs and phototrophs
*lithotrophs and organotrophs
735.
According to the source of carbon there are such groups of bacteria:
A. lithotrophs and organotrophs
B. chemotrophs and phototrophs
C. chemotrophs and heterotrophs
D. autotrophs and chemotrophs
E. *autotrophs and heterotrophs
736.
In heat oven at 160 C Petry palates were sterilized. What test-microbe is it
possible to use for sterilization control?
A. Bacillus cereus
B. Clostridium perfringens
C. Clostridium difficile
D. Proteus vulgaris
E. *Bacillus subtilis
737.
Indicate the composition of liquid Hiss media
A. 1 % peptone water, different carbohydrates (glucose, maltose, lactose,
saccharose, Mannitol and other), an indicator bromthymol dark blue
B. 1 % peptone water, different carbohydrates (glucose, maltose, lactose,
saccharose, Mannitol and other), an indicator eosin
C. 1 % peptone water, different carbohydrates (glucose, maltose, lactose,
saccharose, Mannitol and other), an indicator is methylene blue
D. 1 % peptone water, different carbohydrates (glucose, maltose, lactose,
saccharose, Mannitol and other), an indicator neutral red
E. *1 % peptone water, different carbohydrates (glucose, maltose, lactose,
saccharose, mannitol and other), indicator of Andrede
738.
Which of following is/are nutrient medium/a?
A. Blood agar
B. Serum agar
C. Ascitic agar
D. All answers are false
E. *All answers are correct
739.
In MPB it is necessary to verify the peptolytic properties of bacteria. What
signs
will show the positive results – indole production?
A. to insert the indicator, saturated with a litmus; an indole is changed its
colour, it
becomes red
B. to insert the indicator paper, saturated with an oxalic acid; an indole is
changed
its colour, it becomes dark blue
C. to insert the indicator paper, saturated with a litmus; a indole is
changed its
colour, it becomes dark blue
D. the colour of medium becomes rose
E. *to insert the indicator paper, saturated with an oxalic acid; a indole is
changed
by its colour, it becomes rose
740.
What is the composition of Olkenitsky medium?
A. MPA, lactose, complex salt of ammonium-iron sulfate (Moor’s salt), sodium
thiosulphate, urea, an indicator is phenol red
B. MPA, saccharose, complex salt of ammonium-iron sulfate (Moor’s salt),
sodium
thiosulphate, urea, an indicator is phenol red
C. MPA, glucose, complex salt of ammonium-iron sulfate (Moor’s salt), sodium
thiosulphate, urea, an indicator is phenol red
D. MPA, lactose, saccharose, glucose, complex salt of ammonium-iron sulfate
(Moor’s salt), sodium thiosulphate, an indicator is phenol red
E. *MPA, lactose, saccharose, glucose, complex salt of ammonium-iron sulfate
(Moor’s salt), sodium thiosulphate, urea, an indicator is phenol red.
741.
In Olkenitsky medium Escherichia coli was inoculated. What changes will be
observed in a medium, which will confirm utilization of glucose and lactose?
A. The changes of the colour from a pinky to yellow in the column of agar
B. The changes of the colour from a pinky to yellow of slant surface of agar
C. The changes of the colour from a pinky to red of slant surface of agar
D. The changes of the colour from a pinky to red of the column of agar
E. *The changes of the colour from a pinky to yellow of the column of agar
and
slant surface of agar
742.
In Olkenitsky medium microorganism which products an urea was inoculated.
What changes here will be observed in a medium?
A. appearance of red precipitate in the column of agar
B. appearance of greenish precipitate in the column of agar
C. to fix the urease production in this medium is impossible
D. appearance of brightly-violet precipitate in the column of agar
E. *medium will become red
743.
In Olkenitsky medium Salmonella typhi, which forms the hydrogen supplied was
inoculated. What changes will be in a medium in this case?
A. To fix the hydrogen sulphide production in this medium is impossible
B. appearance of greenish precipitate in the column of agar
C. appearance of bright red precipitate in the column of agar
D. all medium will be black
E. *appearance of black precipitate in the column of agar
744.
Indicate aerotolerant bacteria:
A. Streptococcus viridans
B. Staphylococcus aureus
C. Staphylococcus epidermidis
D. Staphylococcus saprophyticus
E. *Streptococcus pyogenes
745.
Indicate among these microbes obligate anaerobic bacteria:
A. Staphylococcus spp., Streptococcus spp.
B. Escherichia coli, Salmonella typhi
C. Bacillus anthracis, Brucella abortus
D. Mycobacterium tuberculosis, Corynebacterium diphtheriae
E. *Clostridium perfringens, Bacteroides spp.
746.
Indicate composition of semi-solid Hiss media
A. MPA, different carbohydrates (glucose, maltose, lactose, saccharose,
mannitol,
and other), indicator Andrede
B. MPA, different carbohydrates (glucose, maltose, lactose, saccharose,
mannitol,
and other), an indicator diamond green
C. MPA, different carbohydrates (glucose, maltose, lactose, saccharose,
mannitol,
and other), indicator – complex salt of ammonium-iron sulfate
D. MPA, different carbohydrates (glucose, maltose, lactose, saccharose,
mannitol,
and other), an indicator phenol red
E. *MPA, different carbohydrates (glucose, maltose, lactose, saccharose,
mannitol,
and other), indicators water blue with rosolic acid)
747.
Indicate halophilic bacterium:
A.
B.
C.
D.
E.
748.
A.
B.
C.
D.
E.
749.
A.
B.
C.
D.
E.
750.
A.
B.
C.
D.
E.
751.
A.
B.
C.
D.
E.
752.
A.
B.
C.
D.
E.
753.
A.
B.
C.
D.
E.
Streptococcus pyogenes
Staphylococcus aureus
Treponema pallidum
Shigella flexneri
*Vibrio parahaemolyticus
Indicate the basic components of Ploskirev medium.
MPB, saccharose, salts of bile acids, diamond green, neutral red.
MPA, saccharose, salts of bile acids, diamond green, neutral red.
MPB, lactose, salts of bile acids, diamond green, neutral red.
MPA, glucose, salts of bile acids, diamond green, neutral red.
*MPA, lactose, salts of bile acids, diamond green, neutral red.
Indicate the capneic bacteria:
Staphylococcus aureus
Streptococcus pyogenes
Lactobacillus spp.
Clostridium perfringens
*Brucella abortus
Indicate the composition of blood agar.
MPA, defibrinated or fresh blood of animals, an indicator bromthymol blue
MPA, defibrinated or fresh blood of animals, an indicator fuchsin
MPA, defibrinated or fresh blood of animals, an indicator neutral red
MPA, defibrinated or fresh blood of animals, an indicator eosin
*MPA, defibrinated or fresh blood of animals
Indicate the composition of Kitt-Tarozzi medium for cultivation of anaerobes:
MPA, pieces of liver or meat
MPB, yolk, medium is covered with vaseline oil
Hottinger broth, pieces of brain tissues
Serum broth, pieces of liver or meat, , medium is covered with vaseline oil
*Hottinger broth, pieces of liver or meat, glucose, medium is covered with
vaseline oil
What are the main components of liquid Hiss media?
1 % peptone water, different carbohydrates (glucose, maltose, lactose,
saccharose, Mannitol and other), an indicator bromthymol dark blue
1 % peptone water, different carbohydrates (glucose, maltose, lactose,
saccharose, Mannitol and other), an indicator eosin
1 % peptone water, different carbohydrates (glucose, maltose, lactose,
saccharose, Mannitol and other), an indicator is methylene blue
1 % peptone water, different carbohydrates (glucose, maltose, lactose,
saccharose, Mannitol and other), an indicator neutral red
*1 % peptone water, different carbohydrates (glucose, maltose, lactose,
saccharose, mannitol and other), indicator of Andrede
Indicate the composition of Olkenitsky medium?
MPA, lactose, complex salt of ammonium-iron sulfate (Moor’s salt), sodium
thiosulphate, urea, an indicator is phenol red
MPA, saccharose, complex salt of ammonium-iron sulfate (Moor’s salt), sodium
thiosulphate, urea, an indicator is phenol red
MPA, glucose, complex salt of ammonium-iron sulfate (Moor’s salt), sodium
thiosulphate, urea, an indicator is phenol red
MPA, lactose, saccharose, glucose, complex salt of ammonium-iron sulfate
(Moor’s salt), sodium thiosulphate, an indicator is phenol red
*MPA, lactose, saccharose, glucose, complex salt of ammonium-iron sulfate
(Moor’s salt), sodium thiosulphate, urea, and an indicator is phenol red.
754.
Indicate the correct sequence of the stages of isolation of pure culture of
aerobes.
A. Macroscopic and microscopic examination of tested material ? inoculation
on
the slant agar for the obtaining the isolated colonies ? obtaining the pure
culture
? identification ? conclusion
B. Macroscopic and microscopic examination of tested material ? inoculation
in
MPB for the obtaining the isolated colonies ? inoculation on the slant agar
for
obtaining the pure culture ? identification ? conclusion
C. Macroscopic and microscopic examination of tested material ? inoculation
on a
solid nutrient medium for the obtaining the isolated colonies ?
identification ?
conclusion
D. Macroscopic and microscopic examination of tested material ? inoculation
on a
solid nutrient medium for obtaining the isolated colonies ? inoculation on
the
slant agar for obtaining a pure culture ? identification
E. *Microscopic and microscopic examination of tested material ? inoculation
on a
solid nutrient medium for obtaining the isolated colonies ? inoculation on
the
slant agar for obtaining the pure culture ? identification ? conclusion
755.
Indicate the growth signs of bacteria in a liquid nutrient medium:
A. formation of diffuse turbidity
B. formation of pellicle
C. formation of sediment
D. all answers are false
E. *all answers are correct
756.
Indicate the necessary components of Levin medium:
A. MPB, lactose, indicators methylene blue, eosin .
B. MPA, saccharose, indicators methylene blue, eosin.
C. MPB, saccharose, indicators methylene dark, eosin.
D. MPA, lactose, indicators methylene blue, fuchsine.
E. *MPA, lactose, indicators methylene blue, eosin.
757.
Indicate the necessary components of Levin medium:
A. MPB, lactose, indicators methylene blue, eosin .
B. MPA, saccharose, indicators methylene blue, eosin.
C. MPB, saccharose, indicators methylene dark, eosin.
D. MPA, lactose, indicators methylene blue, fuchsin.
E. *MPA, lactose, indicators methylene blue, eosin.
758.
Indicate vitamins necessary for bacterial growth:
A. Biotin
B. Thiamine
C. Riboflavin
D. all answers are false
E. *all answers are correct
759.
Is it necessary to examine the structure of colonies. What method can you
offer?
A. In an electron microscope
B. By a "hanging" drop technique
C. By a phase-contrast microscope
D. By the immersion system of microscope
E. *In the passing light at the small increase of microscope
760.
It is necessary to check the quality of antibacterial filters. What
microorganism is
it possible to choose as a test-object?
A. Salmonella schottmuelleri
B. Staphylococcus aureus
C. Bordetella pertussis
D. Mycobacterium kansassii
E. *Pseudomonas aeruginosa
761.
It is necessary to check the quality of sterilization by compressed steam at
110
°С sterilization. What chemical indicator of sterilization quality can be
utilized?
A. antipyrine
B. sulphur
C. benzoic acid
D. urea
E. *benzonaphtolum
762.
It is necessary to conduct pasteurization of milk. Choose its optimum mode:
A. 70 °С during 60 min
B. 70 °С during 90 min
C. 60 °С during 30 min
D. 60 °С during 60 min
E. *70 °С during 30 min
763.
It is necessary to sterilize an object that destroys at 100 °С . Choose the
comfortable regime of tyndalization:
A. sterilization by water bath at 58-60 °С during a hour 2-3 days one after
the other
B. single sterilization by water bath at 98-100 °С during 10 min
C. sterilization by water bath at 58-60 °С during a hour 3-4 days one after
the other
D. sterilization by water bath at 80-90 °С during a hour 2-3 days one after
the other
E. *sterilization by water bath at 58-60 °С during a hour 5-6 day one after
the other
764.
It is necessary to sterilize material that contains the spore-forming
bacteria by
live steam. Choose the regime of sterilization:
A. single sterilization during 10 min.
B. single sterilization during 20 min.
C. fractional sterilization – three times one after the other during one day,
30 min.
D. fractional sterilization – 2 days one after the other, 30 min.
E. *fractional sterilization – 3 days one after the other, 30 min.
765.
It is necessary to sterilize the dressing material. Choose optimum regime of
sterilization in an autoclave:
A. 110 °С during 20 minutes
B. 127 °С during 20 minutes
C. 134 °С during 40 minutes
D. 100 °С during 20 minutes
E. *120 °С during 20 minutes
766.
It is necessary to study bacterial peptolytic properties. Indicate the proper
medium:
A. Meat-peptone agar
B. Sugar MPA
C. Coagulated serum
D. gelatin
E. *Meat-peptone broth
767.
Which of following is possible to sterilize in burner’s flame:
A. liquid nutrient media
B. physiological solution
C. solid nutrient media
D. catheters
E. *forceps
768.
Material from a patient with suspicion on dysenterywas inoculated on Endo
medium. What colour does colony of lactose-negative Shigella have?
A. dark blue
B. green
C. red with metallic hue
D. brown
E. *colourless
769.
What is the mechanism of antibacterial effect of 37-40 % formaldehyde
solution?
A. destruction of cellular lipids.
B. destruction of cellular endotoxin
C. destruction of cellular polysaccharides.
D. Cell dehydration
E. *Interaction with protein amino groups and their denaturation
770.
Mechanism of high temperatures antibacterial action:
A. damage of ribosomes
B. destruction of tertiary structure of albumens
C. destruction of cytoplasm membranes
D. all answers are false
E. *all answers are correct
771.
Most of non-spore-forming perish at a temperature:
A. 20-30 °С in 2-4 ч
B. 40-50 °С in 1 ч
C. 10-20 °С in 5-8 ч
D. 20-30 °С in 30-60 minutes
E. *58-60 °С 30-60 minutes
772.
Nitrogen-fixing bacteria belong to what group according to their type of
respiration?
A. Obligate aerobes
B. Obligate anaerobes
C. Facultative anaerobes
D. Capneic
E. *Microaerophils
773.
Non-inoculated Endo medium has colour:
A. bright red
B. yellow
C. greenish
D. colourless
E. *light rose
774.
On the mucous membranes of human there are corynebacteria. What selective
medium for their cultivation?
A. Mueller medium
B. 1 % alkaline peptone water
C. Ru medium
D. Saburo medium
E. *Agar with furazolidonum and tween
775.
One of the asepsis measures is disinfection. What is it – a concept
"disinfection"?
A. deleting bacterial and fungal spores of the objects of environment
B. elimination of pathogenic microorganisms in a wound
C. diminishing the degree of microbial contamination of skin and mucous
membranes
D. elimination viruses in the environment
E. *decontamination of pathogenic microorganisms
776.
Peculiarities of S-colonies of bacteria:
A. gyrose surface
B. fibred consistency
777.
778.
779.
780.
781.
782.
783.
784.
785.
C. edges are rosette-like
D. have capsules
E. *smooth, convex surface
Respiration of bacteria is a process:
A. biosynthesis of protein molecules
B. biosynthesis of carbohydrates
C. biosynthesis of lipids
D. biosynthesis of microelements
E. *biooxidation with formation of ATP
Respiration of bacteria is accompanied:
A. by the loss of energy
B. by accumulation of CO2
C. all answers are correct
D. all answers are false
E. *by formation of energy
Rubber and polymeric wares can be sterilized immersing them in:
A. solution of dexon on 25 min at 18 °С
B. solution of dexon on 15 min at 18 °С
C. solution of dexon on 45 min at 56 °С
D. solution of dexon on 25 min at 56 °С
E. *solution of dexon on 45 min at 18 °С
Salts of heavy metals will cause in a bacterial cell:
A. destruction of lipids
B. destroying the polysaccharides
C. DNA destroying
D. Protein synthesis disturbances
E. *coagulation of protein
Special nutrient medium is/are ___:
A. Blood agar
B. Serum agar
C. Ascitic agar
D. All answers are false
E. *All answers are correct
Specify, to what pressures in an autoclave a temperature 112 °С corresponds:
A. 0 аtm.
B. 1 аtm.
C. 1,5 аtm.
D. 2 аtm.
E. *0,5 аtm.
Specify, to what pressures in an autoclave a temperature 121 °С corresponds:
A. 0 аtm.
B. 0,5 аtm.
C. 1,5 аtm.
D. 2 аtm.
E. *1 аtm.
Specify, to what pressures in an autoclave a temperature 127 °С corresponds:
A. 0 аtm.
B. 0,5 аtm.
C. 1 аtm.
D. 2 аtm.
E. *1,5 аtm.
Specify, to what pressures in an autoclave a temperature 131 °С corresponds:
0 аtm.
0,5 аtm.
1 аtm.
1,5 аtm.
*2 аtm.
786.
Sterilization may be conducted by the following methods:
A. action of moist steam
B. filtration
C. radiation
D. pasteurization
E. *all answers are correct
787.
Such enzymes prevail in a bacterial cell:
A. oxydoreductases
B. hydrolases
C. adaptive
D. ligases
E. *constitutive
788.
Such groups of bacteria are existed according to their respiration:
A. capneic
B. microaerophils
C. anaerobes
D. aerobes
E. *all answers are correct
789.
What is the composition of Kitt-Tarozzi medium for cultivation of anaerobes?
A. MPA, pieces of liver or meat
B. MPB, yolk, medium is covered with vaseline oil
C. Hottinger broth, pieces of brain tissues
D. Serum broth, pieces of liver or meat, , medium is covered with vaseline
oil
E. *Hottinger broth, pieces of liver or meat, glucose, medium is covered with
vaseline oil
790.
Such nutrient media belong to universal ones:
A. Mannitol salt agar Levin medium
B. Serum MPA and 1% alkaline peptone water
C. Endo medium and blood agar
D. Ploskirev medium and serum broth
E. *MPB and MPA
791.
Surface-tension-reducing agents (detergents) can cause:
A. violation of spores’ formation.
B. violation of flagella structure
C. violation of capsule formation.
D. Violation of nucleoid function
E. *violation of CPM structure and cell wall structure
792.
Synthesis of what cell molecules needs the most of energy?
A. nucleic acids
B. lipids
C. polysaccharides
D. amino acid
E. *proteins
793.
Taking into consideration the features of fermenting type of anaerobe
bacteria
metabolism they require:
A. More rich for nutrients medium
B. More rich for vitamins nutrient medium
A.
B.
C.
D.
E.
794.
795.
796.
797.
798.
799.
800.
801.
C. Obligatory addition of bacterial growth factors in nutrient medium
D. All answers are false
E. *All answers are correct
The optimum temperature for mesophilic bacteria is ____:
A. 10-15 0C
B. 20-30 0C
C. 40-50 0C
D. 50-60 0C
E. *30-37 0С
The optimum temperature for psychrophilic bacteria is ____:
A. *10-25 0C
B. 20-30 0C
C. 30-37 0С
D. 40-50 0C
E. 50-60 0C
The optimum temperature for thermophilic bacteria is ____:
A. 10-20 0C
B. 20-30 0C
C. 30-37 0С
D. 40-50 0C
E. *50-60 0C
The mechanism of action of iodine consists of:
A. destruction of lipids
B. destroying the polysaccharides
C. DNA depolarization
D. violation of proteins synthesis
E. *oxidization of protein active groups and their denaturizing
The second stage of isolation of pure culture of aerobes is:
A. Macroscopic examination of material and its inoculation in liquid nutrient
medium
B. Macro- and microscopic verification of culture purity
C. Serological identification
D. Biochemical identification
E. *Macro- and microscopic study of colonies and their inoculation them on
slant
agar
The structure of colonies can be examined by such method:
A. *In the passing light, small objective of microscope
B. In an electronic microscope
C. By a "hanging" drop technique
D. In a phase-contrast microscope
E. By the immersion system of microscope
There are such components of Endo medium:
A. MPB, 1 % lactose, an indicator fuchsine
B. MPA, 1 % lactose, an indicator eosin
C. MPB, 1 % lactose, an indicator eosin
D. MPA, 5 % lactose, indicator bromthymol blue
E. *MPA, 1 % lactose, an indicator fuchsine
There are such demands to the nutrient media, EXCEPT:
A. presence of nutrients
B. sterility
C. certain viscidity
D. transparency
E. certain colour
802.
Which of following demands is not nessesary to the nutrient media?
A. presence of nutrients
B. sterility
C. certain viscidity
D. transparency
E. certain colour
803.
There are such mechanisms of penetration different substances into the cell,
EXCEPT:
A. facilitated diffusion
B. passive diffusion
C. active transport
D. ionic transport
E. transformations of chemical groups
804.
To dyes, which can inhibit bacterial growth , belong:
A. diamond green.
B. rivanol.
C. acriflavin.
D. all answer are false.
E. *all answer are right.
805.
Wares from polymeric materials may be sterilized by such method:
A. mechanical
B. biological
C. pasteurization
D. in a heat oven
E. *chemical
806.
Wares from rubber and polymeric materials may be sterilized by immersing
them in:
A. 3 % hydrogen peroxide, 6 hours at 18 °С
B. 6 % hydrogen peroxide, 12 hours at 18 °С
C. 6 % hydrogen peroxide on 1 hour at 18 °С
D. 6 % hydrogen peroxide on 2 hour at 18 °С
E. *6 % hydrogen peroxide on 6 hours at 18 °С
807.
What is tyndalization?
A. single sterilization by water bath at 98-100 °С during 30 minutes
B. sterilization by water bath at 98-100 °С during a hour 5-6 day one after
the
other
C. sterilization by water bath 58-60 °С during a hour 2-3 days one after the
other
D. sterilization by water bath at 98-100 °С during a hour 5-6 days one after
the
other
E. *sterilization by water bath at 58-60 °С during a hour 5-6 day one after
the
other
808.
What microorganisms can be used for quality control of sterilization by
filtration?
A. Escherichia coli
B. Salmonella typhi
C. Clostridium perfringens
D. Corynebacterium xerosis
E. *Serratia mаrcescens
809.
What actions are necessary to do during І stage of isolation of pure cultures
of
aerobes?
A. To study the features of the tested material
B.
C.
D.
E.
To study the morphological features of possible causative agent
To choose necessary nutrient medium for inoculation
To inoculate tested material for obtaining the isolated colonies
*All answers are correct
810.
What antibiotic is it necessary to add into nutrient medium for inhibition of
fungi
growth?
A. Penicillin
B. Streptomycin
C. Ciprofloxacin
D. Mitomicin C
E. *Nistatin
811.
What bacteria are very susceptible to drying?
A. Staphylococci
B. Mycobacterium tuberculosis
C. Shigella
D. Salmonella
E. *Meningococci
812.
What bacteria do belong to obligate anaerobes?
A. Salmonella schottmuelleri
B. Staphylococcus aureus
C. Proteus rettgeri
D. Francisella tularensis
E. *Clostridium septicum, Bacteroides fragilis
813.
What bacterial colony?
A. Medium which is inoculated with bacteria, where different microorganisms
grow
B. Growth of microbes of one species on different nutrient medium
C. Visible growth of bacteria in a liquid nutrient medium
D. Visible growth of bacteria on the slant meat-peptone agar
E. Macroscopically visible growth of microbes (posterity of one bacterial
cell) on a
solid nutrient medium
814.
When microbes utilize sugars Hiss media turn to ____ colour.
A. Violet
B. Brown
C. green
D. a colour does not change
E. *red
815.
What colour does lactose-negative colony of bacteria have on Levin medium?
A. dark blue
B. red with metallic hue
C. brown
D. rose
E. *colourless
816.
What colour does lactose-positive colony of bacteria have on Levin medium?
A. colourless
B. red with metallic hue
C. brown
D. green
E. *dark blue
817.
What colour does a simple medium – MPA– have?
A. red
B. greenish
C. violet
D. dark blue
E. *yellow
818.
What colour does a simple medium – MPB– have?
A. red
B. dark blue
C. greenish
D. violet
E. *yellow
819.
What colour does noninoculated Levin medium have?
A. bright red
B. poorly-rose
C. yellow
D. greenish
E. *violet
820.
What colour does non-inoculated Olkenitsky medium have?
A. bright red
B. pink
C. *yellow
D. greenish
E. violet
821.
What colour does noninoculated Ressel’s medium have?
A. bright red
B. greenish
C. violet
D. pinky
E. *yellow
822.
What components may be the source of nitrogen in nutrient media?
A. meat
B. fish
C. meat and bone flour
D. casein
E. *all answers are correct
823.
What are the basic components of Ploskirev medium.
A. MPB, saccharose, salts of bile acids, diamond green, neutral red.
B. MPA, saccharose, salts of bile acids, diamond green, neutral red.
C. MPB, lactose, salts of bile acids, diamond green, neutral red.
D. MPA, glucose, salts of bile acids, diamond green, neutral red.
E. *MPA, lactose, salts of bile acids, diamond green, neutral red.
824.
What enzyme of aerobic bacteria does provide neutralization of hydrogen
peroxide toxic effect?
A. lipase
B. protease
C. hydrolase
D. urease
E. *catalase
825.
What enzymes do control the substrate splitting with joining the water
molecules?
A. Oxydoreductases
B. Isomerases
C. Ligases
D. Transferases
E. *Hydrolases
826.
What enzymes do provide oxydation-reduction reactions?
A. Hydrolases
B. Isomerases
C. Transferases
D. Lyases
E. *Oxydoreductases
827.
What are the growth signs of bacteria in a liquid nutrient medium:
A. formation of diffuse turbidity
B. formation of pellicle
C. formation of sediment
D. all answers are false
E. *all answers are correct
828.
What enzymes do provide the reactions of groups of atoms transfer?
A. Ligases
B. Lyases
C. Oxydoreductases
D. Isomerases
E. *Transferases
829.
What enzymes localized in cytoplasm membrane do provide the facilitated
diffusion?
A. transferases
B. proteases
C. hydrolases
D. ligases
E. *permeases
830.
What factors do not influence on the bacterial reproduction?
A. age of bacteria
B. composition of nutrient medium
C. medium рН
D. temperatures, aeration
E. *year season
831.
What finished products does appear in the semi-solid Hiss media after sugar
utilization?
A. acid
B. alkali
C. gas
D. fixing the finished products is not succeeded
E. *acid and gas
832.
What from the listed words is a name of enzyme?
A. Saccharose
B. Glucose
C. Maltose
D. Galactose
E. *Saccharase
833.
What group of nutrient media does Ressel medium belong to?
A. Universal
B. Liquid
C. Selective
D. Elective
E. *Differential-diagnostic
834.
What groups of methods for creation of anaerobic conditions for anaerobic
bacteria cultivation do you know ?
A.
B.
C.
D.
E.
835.
A.
B.
C.
D.
E.
836.
A.
B.
C.
D.
E.
837.
A.
B.
C.
D.
E.
838.
A.
B.
C.
D.
E.
839.
A.
B.
C.
D.
E.
840.
A.
B.
C.
D.
E.
841.
A.
B.
C.
D.
E.
842.
A.
B.
C.
D.
mechanical
physical
chemical
biological
*all answers are correct
What are the necessary components of Levin medium:
MPB, lactose, indicators methylene blue, eosin .
MPA, saccharose, indicators methylene blue, eosin.
MPB, saccharose, indicators methylene dark, eosin.
MPA, lactose, indicators methylene blue, fuchsin.
*MPA, lactose, indicators methylene blue, eosin.
What is a biological catalyst?
nucleic acids
metabolites
Plasmids
Transposons
*enzymes
What is a chemical origin of agar-agar?
Lipids
Proteins
Carbohydrates
Amino acids
*Polysaccharides
Which of following vitamins are necessary for bacterial growth:
Biotin
Thiamine
Riboflavin
all answers are false
*all answers are correct
What is a chemical origin of gelatin?
Polysaccharides
Lipids
Proteins
Carbohydrates
*Amino acids
Choose among these media differential ones:
MPA
Kitt-Tarozzi medium
MPB
Sugar meat peptone broth
*Hiss media
What is a name of enzyme localized inside a bacterial cell:
adaptive
exoenzymes
constitutive
all answers are right
*endoenzymes
What is a reason molecular oxygen toxic action according to bacteria?
formation of singlet oxygen
formation of hydrogen peroxide
formation of ozone
all answers are incorrect
E. *all answers are right
843.
What is advantage of Koch, Drigalsky and streak techniques above the Pasteur
method?
A. Enable to study biochemical properties of bacteria
B. Enable to study antigen properties of bacteria
C. Enable to obtain a pure culture
D. *Enable to obtain the isolated colonies
E. Right answer is not present
844.
What is an essence of Koch technique for isolation of pure cultures?
A. Use of liquid nutrient media for obtaining the isolated colonies
B. Use of blood agar for examination of hemolysines types
C. Use of solid nutrient media for examination of antibiotics susceptibility
D. Use liquid nutrient media for examination of antibiotics susceptibility
E. *Use of solid nutrient media (gelatin) for obtaining the isolated colonies
845.
What is an essence of Pasteur technique for isolation of pure cultures?
A. Dilution of tested material in gelatin
B. Dilution of tested material in MPA
C. Dilution of tested material in the AGV nutrient medium
D. Dilution of tested material in a solid nutrient medium
E. *Dilution of tested material in a liquid nutrient medium
846.
What is an essence of ІV stage of isolation of pure culture of anaerobic
bacteria?
A. Inoculation of colonies on the proper nutrient medium
B. Examination of tinctorial properties of bacteria
C. Examination of serologic features of bacteria which form colonies by
precipitation test
D. Right answer is absent
E. *Checking the culture purity and its identification
847.
What is biochemical identification of bacteria?
A. Verification of bacteria according to their ability to produce
dehydrogenases
B. Verification of bacteria according to their ability to produce hemolysines
C. Verification of bacteria according to their tinctorial properties
D. Verification of bacteria according to their reduce properties
E. *Verification of bacteria according to their biochemical properties
848.
What is chemical composition of prokaryotic cell?
A. Water - 50-60 %, dry weight - 40-60 %.
B. Water - 60-90 %, dry weight - 10-30 %.
C. Water - 40-50 %, dry weight - 50-60 %.
D. Water - 20-40 %, dry weight - 60-80 %.
E. *Water - 70-90 %, dry weight - 10-30 %.
849.
What is a chemical origin of gelatin?
A. Polysaccharides
B. Lipids
C. Carbohydrates
D. Amino acids
E. *Proteins
850.
What is chemostate?
A. synonym of term of «jar»
B. vehicle which allows to study chemical properties of bacteria
C. vehicle, in which it is possible to make sterilization of media
D. vehicle which allows to study antigenic properties of bacteria
E. *vehicle which allows to make continuous cultivation of microorganisms in
laboratory conditions
851.
What is disinfection?
A. Complex measures for complete, partial or selective elimination of
potential
pathogens for human in the air for warning transmission of causative agents
from the source of infection to the receptive organism
B. Complex of measures for complete, partial or selective elimination of
potential
pathogens for human in his body for warning transmission of causative agents
from the source of infection to the receptive organism
C. Complex of measures which prevent spread of potential pathogens from human
body in an medium
D. Complex of measures for complete, partial or selective elimination of
potential
pathogens for human in the water for warning transmission of causative agents
from the source of infection to the receptive organism
E. *Complex of measures for complete, partial or selective elimination of
potentially pathogenic for human causative agents on the different objects of
medium for warning transmission of causative agents from the source of
infection to the receptive organism
852.
What is it necessary to do during the second stage of isolation of pure
culture of
aerobes?
A. Study the features of colonies
B. Make a smear from colonies
C. Inoculation onto slant agar
D. All answers are not correct
E. *All answers are correct
853.
What is raw material for agar-agar obtaining?
A. Skin
B. Meat
C. Arthral cartilage
D. Bones
E. *Algae
854.
What is the essence of ІІІ stage of isolation of pure cultures of aerobes?
A. Examination the isolated colonies
B. Examination bacterial growth in the column of agar
C. Bacterial phage typing
D. Examination of ability for bacteriocine production
E. *Identifications of the unknown bacteria
855.
What is the main demand to any nutrient medium used for the primary selection
of microorganisms?
A. To be solid
B. To be liquid
C. To contain vitamins
D. To contain growth factors
E. *To be sterile
856.
What is the main idea of ІІІ stage of isolation of pure culture of anaerobic
bacteria?
A. Examination of features of pure culture and its identification
B. Examination of features of colonies, preparing the smear from them,
inoculation
of colonies on slant MPA for obtaining the pure culture
C. Examination of features of colonies, preparing the smear from them,
inoculation
of colonies in MPB for obtaining the pure culture
D. Examination of features of colonies, inoculation of colonies in KittTarozzi
medium for obtaining the pure culture
E. *Examination of features of colonies, preparing the smear from them ,
inoculation of colonies into Kitt-Tarozzi medium for obtaining the pure
culture
857.
What is the minimum area of washings for control of disinfection quality?
A. not less than, 50 см2.
B. not less than, 100 см2.
C. not less than, 150 см2.
D. not less than, 250 см2.
E. *not less than, 200 см2.
858.
What a main aim of the first stage of isolation of pure cultures of aerobes?
A. To study tested material
B. To choose nutrient medium for inoculation
C. To study the morphological features of possible causative agent
D. To study the antigen features of causative agent
E. *To obtain the isolated colonies
859.
What mechanism of action of ultraviolet light against bacteria?
A. does not have any effect according to the cell
B. affect RNA molecules
C. affect the ribosomes
D. affect the structure of cell wall
E. *affect DNA molecules and as a result forms thymine dimers
860.
What mechanism of ultrasound action against bacteria?
A. destroys cell DNA
B. destroys cell RNA
C. destroys cell mesosome
D. an ultrasound does not influence on bacteria
E. *forms a cavitation cavity, filled in the pair of liquid, with high
pressure inside;
cytoplasm structures are destroyed
861.
What is mechanism of bacterial reproduction?
A. by a mitosis
B. by spore formation
C. by sexual way
D. by a disjunctive way
E. *by a simple transversal division
862.
What medium is it possible to use for checking the utilization of a few
sugars?
A. Ru medium
B. Leffler medium
C. *Hiss media
D. Ploskirev medium
E. Endo medium
863.
What methods of isolation of pure cultures, based on mechanical principle do
you know ?
A. Pasteur method
B. Koch method
C. Streak method
D. All answers are incorrect
E. *All answers are correct
864.
What methods of isolation of pure cultures, based on mechanical principle do
you know?
A. Leffler method
B. Grey method
C. Buchin method
D. Nemchinov method
E. *Streak method
865.
What microbes belong facultative anaerobes to?
A. Clostridium tetani, Clostridium perfringens
B. Mycobacterium tuberculosis, Vibrio cholerae
C. Brucella abortus
D. Lactobacillus acidophilus
E. *Corynebacterium diphtheriae, Salmonella typhi
866.
What part of dry weight of the cell does belong to protein?
A. 15 %.
B. 35 %.
C. 75 %.
D. 95 %
E. 55 %.
867.
What phase is characterized by high speed of bacterial division and
increasing
their number?
A. Stationary
B. Initial
C. Death
D. Expotential
E. *Exponential
868.
What phase of microbial growth in periodic culture does not exist?
A. Initial
B. Exponential
C. Stationary
D. Decline
E. *Accumulations
869.
What properties of colonies are studied for cultural identification?
A. Shape, colour
B. Character of edges
C. Character of surface of colonies
D. All answers are false
E. *All answers are correct
870.
What signs of hemolytic activity of bacteria are on blood MPA?
A. appearance of areas of darkening on a mat red background
B. the changes of medium colour
C. appearance of bright red colonies of microbes, if they cause haemolysis
D. appearance of colourless colonies
E. *appearance of colourless areas surround the colonies on mat red
background if
the have hemolytic activity
871.
What stage of meat water preparing is the most important?
A. Making meat small
B. Filtration
C. Boiling
D. Sterilization
E. *Extraction
872.
What substance must be added into nutrient medium for examination of
bacterial
proteolytic properties?
A. Agar-agar
B. Agarose
C. All answers are correct
D. All answers are false
E. *Gelatin
873.
What substances can repress the action of inhibitors of bacterial growth and
toxin formation?
A. some aminoacids
B. tweens
C. active carbon
D. all answers are false
E. *all answers are correct
874.
What is optimum temperature for mesophilic bacteria?
A. 10-15 ?C
B. 40-50 ? C
C. 50-60 ? C
D. 20-30 ? C
E. *30-37 ? С
875.
What is optimum temperature for psychrophilic bacteria?
A. 20-30 ? C
B. 30-37 ? С
C. 40-50 ? C
D. 50-60 ? C
E. *10-25 ? C
876.
What is optimum for thermophilic bacteria?
A. 10-20 ? C
B. 20-30 ? C
C. 30-37 ? С
D. 40-50 ? C
E. *50-60 ? C
877.
What substitutes of meat are used for making the nutrient media?
A. placenta
B. blood clot
C. yeast
D. all answers are false
E. *all answers are correct
878.
What substrate does provide a necessary density of nutrient medium?
A. Protein
B. Lipids
C. Carbohydrates
D. No correct answer
E. *Agar-agar
879.
Which of following colour does lactose-positive colony of bacteria have on
Ploskirev medium?
A. dark blue
B. rose
C. brown
D. green
E. *colourless
880.
Which of following colour does lactose-negative colony of bacteria form on
Ploskirev medium?
A. dark blue
B. rose
C. brown
D. green
E. *colourless
881.
What technique does belong to the biological method of anaerobic conditions
creation?
A. Cultivation of anaerobes in the organism of laboratory animals
B. Cultivation of anaerobes in the chicken embryos
C. All answers are correct
D. All answers are false
E. *Joint cultivation of anaerobic bacteria and Serratia marcescens, which
utilizes
an oxygen intensively
882.
What technique does belong to the chemical method of anaerobic conditions
creation?
A. use of Gas generating box
B. application of pyrogallol on the bottom of the jar
C. use the high column of nutrient medium
D. All answers are false
E. *All answers are correct
883.
Choose among following a colour of MPB?
A. red
B. dark blue
C. greenish
D. violet
E. *yellow
884.
Choose among following a colour of noninoculated Levin medium.
A. bright red
B. poorly-rose
C. yellow
D. greenish
E. *violet
885.
What type of sediment may be formed by microbes in liquid nutrient media?
A. Crumble-like
B. homogeneous
C. viscid
D. mucous
E. *all answers are correct
886.
Which of following demands is not nessesary to the nutrient media?
A. presence of nutrients
B. sterility
C. certain viscidity
D. transparency
E. certain colour
887.
What types of bacterial fermentation do you know?
A. Lactic-acid fermentation
B. alcoholic fermentation
C. butiric-acid fermentation
D. all answers are false
E. *all answers are correct
888.
What vitamins are necessary for development of bacteria?
A. pantothenic acid
B. Cholin
C. nicotine acid
D. all answers of incorrect
E. *all answers are correct
889.
Where does point of replication localize, when the division of bacteria
begins?
A.
B.
C.
D.
E.
on DNA of cell
on мРNA
on a cytoplasm membrane
on a cellular wall
*on DNA, located in the place of connection of mesosome and cytoplasm
membrane
890.
Who was discoverer of phenomenon of anaerobiosis?
A. Mechnikov
B. S. Vinogradsky
C. R. Koch
D. E. Ru
E. L. Pasteur
891.
What is the optimum temperature for mesophilic bacteria?
A. 10-15 1 C
B. 20-30 2C
C. 40-50 4C
D. 50-60 5C
E. *30-37 3С
892.
A degree of heterotrophy may be different. The highest heterotrophy has
prokaryotic organisms that are able to live only inside the living cells.
There are:
A. staphylococci and streptococci
B. escherichia and salmonella
C. mycobacteria and bordetella
D. vibrios and treponema
E. *rickettsia and chlamidia
893.
A nurse has to sterilize surgical instruments for an operation. What most
effective method can you offer?
A. fractional sterilization
B. sterilization by pressed steam
C. boiling
D. ionizing radiation
E. *by dry heat
894.
After finishing the experience in students’ laboratory it’s necessary to
disinfect
the workplace. What chemicals can be used?
A. ether
B. hydrochloric acid
C. formalin
D. chloroform
E. *Chloraminum
895.
After inoculation of Escherichia coli on Ploskirev medium the growth of
bacteria
is inhibited. What chemical does predetermine this phenomenon?
A. oxalic acid
B. sodium sulphite
C. Bismuth salts
D. Fuchsine
E. *brilliant green
896.
Which of following nutrient media is recomended to study ability of microbes
to
utilize glucose, saccharose, lactose, product of hydrogen sulphide and
utilize of urea.?
A. Ru medium
B. Leffler medium
C. Endo medium
D. Ploskirevmedium
E. *Olkenitsky medium
897.
After inoculation of the patient’s feces on Endo medium there were grown 2
types of colonies. Ones is red with metallic hue, others – colourless. To
what group of
nutrient media does Endo medium belong to?
A. Elective
B. Enriching media
C. Universal
D. Selective
E. *Differential
898.
Lower jaw osteomyelitis has developed. What examination is it necessary for
identification of causative agents and to choose effective drug for
treatment?
A. Complex serological examination
B. To examine the presence of proper antibodies
C. Microscopic examination of the pus
D. To examine bacterial antigens
E. *To isolate bacterial pure culture and examine its susceptibility to
antibiotics
899.
Chemical modification of benzylpenicillin (penicillin G) has resulted in
several
beneficial changes in the clinical use of this drug. Which one of the
following is NOT
one of those beneficial changes?
A. Lowered frequency of anaphylaxis
B. Increased activity against gram-negative rods
C. Reduced cleavage by penicillinase
D. No correct answer
E. *Increased resistance to stomach acid
900.
Colonies of bacteria that grow on solid nutrient media can differentiate
according
to:
A. consistencies
B. density
C. colour
D. all answers are wrong
E. *all answers are correct
901.
An outbreak of gastrointestinal disease is caused by an Escherichia coli
strain
which is sensitive to ampicillin, tetracycline, and chloramphenicol. A stool
sample from
one patient yields E. coli with the same serotype resistant to the three
antibiotics. This
phenomenon is the most closely associated with the next process:
A. Transduction
B. DNA transformation
C. Transposition
D. No correct answer
E. *Conjugation
902.
To examine of peptolytic properties what products are checked in a nutrient
medium:
A. Carbonic acids and waters
B. Glucose and lactose
C. Carbonic acid and nitrogen
D. Mannitol and methanol
E. *Indole and hydrogen sulfide
903.
During the evolution bacteria produced the ways of energy reception. There
are:
A. fermentation
B. respiration
C. photosynthesis
D. all answers are wrong
E. *all answers are correct
904.
To obtaine a pure culture of anaerobic microorganisms the isolated colony is
inoculated on/in ___:
A. Hisss medium
B. Zeissler medium
C. Endo medium
D. Ploskirev medium
E. *Kitt-Tarozzi medium
905.
Which of following indicator may be added into nutrient medium for
cultivation
of anaerobes to testify expedience of its use for inoculation of tested
material?
A. Fuchsine
B. Brilliant green
C. Methylene green
D. Fenolrot
E. *Rezazurin
906.
Energy which is generated by a cell exists in the form of electrochemical
transmembrane gradient of hydrogen ions - ?µН+ or in ATP molecules. What is
the
mechanism of transformation of one type of energy into another?
A. electronic ATF-synthesizing complex
B. molecular ATF-synthesizing complex
C. all answers are correct
D. all answers are wrong
E. *proton ATF-synthesizing complex
907.
Enzymes that are produced outside from bacteria are called _________:
A. endoenzymes
B. heteroenzymes
C. isoenzymes
D. all answers are correct
E. exoenzymes
908.
Escherichia coli was inoculated in Ressel medium. What changes will be
observed in nutrient medium to verify utilization of glucose and lactose?
A. Changes of the colour from a pinky to dark blue in the column of agar
B. Changes of the colour from pinky to dark blue of the slant surface of agar
C. Changes of the colour from a pinky to red of the slant surface of agar
D. Changes of the colour from a pinky to red of the column of agar
E. *Changes of the colour from a pinky to dark blue in the column of agar and
slant
surface of agar
909.
For biological control of sterilization quality microorganisms are used. What
important feature do they have?
A. gram-positive microbes
B. gram-negative microbes
C. form capsules
D. form pili
E. *form spores
910.
For checking the quality of filters in an experiment a fluid containing
testmicrobe was filtered. Later a filter was laid on the surface of nutrient
medium and put
into the incubator. How much time is it necessary to hold it on medium to
give a final
answer about its quality?
A. 1 day at an optimum temperature
B. optimum temperature, 2 days
C. optimum temperature, 3 days
D. optimum temperature, 4 days
E. *at an optimum temperature, 5 day
911.
For examination of lipolytic properties of microbes lipids and lipid-like
substances – tweens – are supplemented into the medium. According to what
signs is it
possible to check the lipolytic activity of bacteria?
A. The colonies of bacteria lose their typical form
B. The colonies of bacterium are painted in the colour of indicator
C. The colonies of bacteria become semilucent
D. If microbes lipolytic properties they do not grow on an medium
E. *Bacteria form iridescent halos round the colonies
912.
For examination of lipolytic properties of microbes lipids and lipid-like
substances – tweens – are supplemented into the medium. According to what
signs is it
possible to check the lipolytic activity of bacteria?
A. The colonies of bacteria lose their typical form
B. The colonies of bacterium are painted in the colour of indicator
C. The colonies of bacteria become semilucent
D. If microbes lipolytic properties they do not grow on an medium
E. *Bacteria form iridescent halos round the colonies
913.
For inoculation of microbes according to Drigalsky method we can use:
A. Bacteriological loop
B. Bacteriological needle
C. Jar
D. All answers are correct
E. *Spatula
914.
For sterilization of glassware utilize a heat oven is used. Choose the best
regime
of sterilization:
A. 180 ?C, 15-30 minutes
B. 180 ?C, 120 minutes
C. 160 ?C, 45-60 minutes
D. 160 ?C, 15-30 minutes
E. *180 ?C, 45-60 minutes
915.
For verifying the peptolytic properties of bacteria what substances are
examined
in nutrient medium?
A. carbon acid and water
B. glucose and lactose
C. carbon acid and nitrogen
D. mannitol and methanol
E. indole and hydrogen sulphide
916.
Choose among listed the optimum selective nutrient medium to inoculate sample
from patient with typhoid fever.
A. 1 % alkaline peptone water
B. Ru medium
C. Blood agar
D. Saburo medium
E. *Mueller medium
917.
From patient with gas anaerobic infection it is necessary to select the
culture of
causative agents. What medium can be chosen?
A. Leffler medium
B. Petrov medium
C. Makkoy-Chepin medium
D. Petranyani medium
E. *Kitt-Tarozzi medium
918.
From patient’s blood the culture Salmonella typhi was isolated. What cultural
properties characterize this microbe?
A. Formation of red colonies with metallic hue on Endo medium
B. Formation of colourless colonies on Bismuth-sulfite agar
C. Formation of haemolytic zone in a blood agar
D. Formation of tender pellicle on alkaline peptone water
E. *Formation of colourless or pinky colonies on Endo and Ploskirev nutrient
media
919.
How much time is it necessary to cultivate the bacteria on Bismuth-sulfite
agar
for final evaluation of growth?
A. 12 hours
B. 24 hours
C. 36 hours
D. 96 hour
E. *48 hours
920.
In a bacteriological laboratory it is necessary to nutrient media, which
contain
matters, changing at a temperature higher then 100 °С (urea, carbohydrates,
proteins).
What method of sterilization can you offer?
A. autoclave, pressed steam
B. boiling
C. tindalization
D. pasteurization
E. *by steam
921.
What selective nutrient medium is used for diagnosis of vaginal candidiasis?
A. Mueller medium
B. 1 % alkaline peptone water
C. Ru medium
D. Blood agar
E. *Saburo medium
922.
In a medium with gelatin it is possible to verify the proteolytic properties
of
bacteria. What signs will show the positive results?
A. compression of gelatin column
B. discolouration of gelatin column
C. appearance of dark precipitate in gelatin column
D. brightening of gelatin column.
E. *liquefacience of gelatin column
923.
In heat oven at 160 C Petry dishes were sterilized. What test-microbe is it
possible to use for sterilization control?
A. Bacillus cereus
B. Clostridium perfringens
C. Clostridium difficile
D. Proteus vulgaris
E. *Bacillus subtilis
924.
In MPB it is necessary to check the peptolytic properties of bacteria. How is
it
possible to prove, if microbes produces the hydrogen sulphide?
A. to insert the indicator paper, saturated with the lead acetate; the
hydrogen
sulphide is changed its colour, it becomes red
B. to insert the indicator paper, saturated with the lead acetate; the
hydrogen
sulphide is changed its colour, it becomes dark blue
C. the colour of medium becomes greyish
D. to insert the indicator paper, saturated with an oxalic acid; the hydrogen
sulphide
is changed by its colour, it becomes black
E. *to insert the indicator paper, saturated with a lead acetate; the
hydrogen
sulphide is changed its colour, it becomes black
925.
In Olkenitsky medium Escherichia coli was inoculated. What changes will be
observed in a medium for confirmation of utilization of glucose and lactose
by this
microbe?
A. The changes of the colour from a pinky to yellow in the column of agar
B. The changes of the colour from a pinky to yellow of slant surface of agar
C. The changes of the colour from a pinky to red of slant surface of agar
D. The changes of the colour from a pinky to red of the column of agar
E. *The changes of the colour from a pinky to yellow of the column of agar
and
slant surface of agar
926.
In Olkenitsky medium microorganism which products an urea was inoculated.
What changes will be observed in a medium?
A. appearance of red precipitate in the column of agar
B. appearance of greenish precipitate in the column of agar
C. to fix the urease production in this medium is impossible
D. appearance of brightly-violet precipitate in the column of agar
E. *hole medium will become red
927.
In Olkenitsky medium Salmonella typhi, which decomposes glucose to acid was
inoculated. What changes will be observed in a medium, which certify
utilization of
glucose?
A. The changes of the colour from a pinky to yellow the slant surface of agar
B. The changes of the colour from a pinky to red the slant surface of agar
C. The changes of the colour from a pinky to yellow of the column of agar and
the
slant surface of agar
D. The changes of the colour from a pinky to red of the column of agar
E. *The changes of the colour from a pinky to yellow of the column of agar
928.
What changes of the Olkenitsky medium will be observed to certify utilization
of
glucose by enterobacteria?
A. The changes of the colour from a pinky to yellow the slant surface of agar
B. The changes of the colour from a pinky to red the slant surface of agar
C. The changes of the colour from a pinky to yellow of the column of agar and
the
slant surface of agar
D. The changes of the colour from a pinky to red of the column of agar
E. *The changes of the colour from a pinky to yellow of the column of agar
929.
In what nutrient media for cultivation of bacteria 7,5-10 % chloride of
sodium
may be added?
A. Endo medium
B. Buchin medium
C. Blood-sugar agar
D. MPB
E. *Mannitol-salt agar
930.
Is it necessary to examine the structure of colonies. What method can you
offer?
A. In an electron microscope
B. By a "hanging" drop technique
C. By a phase-contrast microscope
D. By the immersion system of microscope
E. *In the passing light at the small increase of microscope
931.
It is necessary to verify the hemolytic properties of bacteria. What nutrient
media will you recommend?
A. simple MPA
B. simple MPB
C. Endo medium
D. serum MPA
E. *blood MPA
932.
It is necessary to check the peptolytic properties of bacteria. What nutrient
medium will you recommend?
A. MPA
B. Endo medium
C. Ru medium
D. 1 % alkaline peptone water
E. *MPB
933.
It is necessary to check the quality of antibacterial filters. What
microorganism is
it possible to choose as a test-object?
A. Salmonella schottmuelleri
B. Staphylococcus aureus
C. Bordetella pertussis
D. Mycobacterium kansassii
E. *Pseudomonas aeruginosa
934.
It is necessary to check the quality of sterilization by compressed steam at
121
°С. What chemical indicator of sterilization effectiveness may be used?
A. benzonaphtolum
B. antipyrine
C. sulphur
D. urea
E. *benzoic acid
935.
It is necessary to check the quality of sterilization by compressed steam at
110
°С sterilization. What chemical indicator of sterilization quality can be
utilized?
A. antipyrine
B. sulphur
C. benzoic acid
D. urea
E. *benzonaphtolum
936.
It is necessary to check the quality of sterilization by compressed steam at
115
°С. What chemical indicator of sterilization quality can be utilized?
A. benzonaphtolum
B. sulphur
C. benzoic acid
D. urea
E. *antipyrine
937.
It is necessary to check the quality of sterilization by compressed steam at
132
°С. What chemical indicator of sterilization quality can be utilized?
A. benzonaphtolum
B. antipyrine
C. sulphur
D. benzoic acid
E. *urea
938.
It is necessary to choose the medium for cultivation of anaerobic bacteria:
A. Endo and Levin media
B. Meat-peptone agar, meat-peptone broth
C. Coagulated serum, мeat-peptone gelatin
D. Ascitic agar, serum agar
E. *Zeissler blood-sugar agar, Kitt-Tarozzi medium
939.
It is necessary to create anaerobic conditions for cultivation of proper
bacteria by
biological method. What method could you propose?
A. Zeissler method
B. Shukevitch method
C. Veinberg method
D. Pasteur method
E. *Fortner method
940.
It is necessary to make biochemical identification and examine saccharolytic
properties of bacteria. What media could you offer?
A. Veinberg medium
B. Zeissler medium
C. Kitt-Tarozzi medium
D. Milk
E. *Hiss media
941.
It is necessary to prepare a coagulated serum for isolation of culture of
Corynebacterium diphtheriae. How can you do serum coagulation?
A. 80-90 °С during 10 min
B. 70-80 °С during 30 min
C. 50-60 °С during 30 min
D. 90-100 °С during 30 min
E. *80-90 °С during 1 hour.
942.
It is necessary to sterilize an object that destroys at 100 °С . Choose the
comfortable regime of tyndalization:
A. sterilization by water bath at 58-60 °С during a hour 2-3 days one after
the other
B. single sterilization by water bath at 98-100 °С during 10 min
C. sterilization by water bath at 58-60 °С during a hour 3-4 days one after
the other
D. sterilization by water bath at 80-90 °С during a hour 2-3 days one after
the other
E. *sterilization by water bath at 58-60 °С during a hour 5-6 day one after
the
other
943.
It is necessary to study ability of microbes to utilize glucose, saccharose,
lactose,
production of hydrogen sulphide and utilization of urea. What nutrient media
will you
recommend?
A. Ru medium
B. Leffler medium
C. Endo medium
D. Ploskirevmedium
E. *Olkenitsky medium
944.
Laboratory diagnosis of tetanus was made in the laboratory. What method of
sterilization is it necessary to use for killing the selected cultures?
A. Boiling
B. Tindalizatio
C. In the heat oven
D. Pasteurization
E. *Autoclaving
945.
Sample from a patient with typhoid fever was inoculated on Endo medium. What
colour do colony of lactose-negative Salmonella typhi have?
A. dark blue
B. red with metallic hue
C. brown
D. green
E. *colourless
946.
Material from a patient with suspicion on typhoid fever was inoculated on
Endo
medium. What colour do colony of lactosonegative Salmonella typhi have?
A. dark blue
B. red with metallic hue
C. brown
D. green
E. *colourless
947.
Material from a patient with dysentery was inoculated on Endo medium. What
colour do colony of lactose-negative Shigella have?
A. dark blue
B. green
C. red with metallic hue
D. brown
E. *colourless
948.
Material from a patient with an intestinal infection was inoculated on the
Ploskirev medium. What colour do colonies of Escherichia coli have?
A. colourless
B. brown
C. green
D. dark blue
E. *red
949.
Material from a patient with shigellosis (dysentery) was inoculated on the
Ploskirev medium. What colour do colonies of lactose-negative Shigella have?
A. dark blue
B. green
C. red with metallic hue
D. brown
E. *colourless
950.
Material from a sick woman must be inoculated on selective nutrient medium
for
diagnosis of vaginal candidiasis. Indicate this medium?
A. Mueller medium
B. 1 % alkaline peptone water
C. Ru medium
D. Blood agar
E. *Saburo medium
951.
It is possible to verify the proteolytic properties of bacteria in a medium
with
gelatin. What signs will show the positive results?
A. compression of gelatin column
B. discolouration of gelatin column
C. appearance of dark precipitate in gelatin column
D. brightening of gelatin column.
E. *liquefacience of gelatin column
952.
Sample of a sick woman must be inoculated on selective nutrient medium for
diagnosis of vaginal candidiasis. Indicate this medium.
A. Mueller medium
B. 1 % alkaline peptone water
C. Ru medium
D. Blood agar
E. *Saburo medium
953.
Material from patient with typhoid fever must be inoculated on selective
nutrient
medium. Choose most optimum from them:
A. 1 % alkaline peptone water
B.
C.
D.
E.
Ru medium
Blood agar
Saburo medium
*Mueller medium
954.
What nutrient media will you recommend to study ability of microbes to
utilize
glucose, saccharose, lactose, production of hydrogen sulphide and utilization
of urea?
A. Ru medium
B. Leffler medium
C. Endo medium
D. Ploskirevmedium
E. *Olkenitsky medium
955.
What will changes be observed in Olkenitsky medium to confirm glucose and
lactose utilization by Escherichia coli inoculated in it one?
A. The changes of the colour from a pinky to yellow in the column of agar
B. The changes of the colour from a pinky to yellow of slant surface of agar
C. The changes of the colour from a pinky to red of slant surface of agar
D. The changes of the colour from a pinky to red of the column of agar
E. *The changes of the colour from a pinky to yellow of the column of agar
and
slant surface of agar
956.
Microorganisms that are used as an energy source chemical reaction in the
medium belong to:
A. heterotrophs
B. autotrophs
C. lithotrophs
D. organotrophs
E. *chemotrophs
957.
On the Endo medium colonies Escherichia and Salmonella were isolated.
According to what signs is it possible to make differentiation between them?
A. sizes
B. character of the edges
C. character of surfaces
D. consistency
E. *colour
958.
On the oral mucous membranes of human there are corynebacteria. What
selective medium for their cultivation?
A. Mueller medium
B. 1 % alkaline peptone water
C. Ru medium
D. Saburo medium
E. *Agar with furazolidonum and tween
959.
One of the asepsis measures is disinfection. What does mean "disinfection"?
A. deleting bacterial and fungal spores of the objects of environment
B. elimination of pathogenic microorganisms in a wound
C. diminishing the degree of microbial contamination of skin and mucous
membranes
D. elimination viruses in the environment
E. *decontamination of pathogenic microorganisms
960.
PBP2a, the penicillin-binding protein which causes methicillin-resistance in
Staphylococcus aureus, causes this resistance because it:
A. blocks entry of methicillin into cells
B. hydrolyzes (inactivates) methicillin
C. binds methicillin and prevents it from reaching its target site(s)
D. by any way
E. *helps synthesize cell wall in the presence of methicillin
961.
Choose amoung listed biological method to create anaerobic conditions for
cultivation of anaerobes.
A. Zeissler method
B. Shukevitch method
C. Veinberg method
D. Pasteur method
E. *Fortner method
962.
Salmonella typhi, which utilizes glucose to acid was inoculated in Ressel
medium. What changes will be observed in nutrient medium, which verify
utilization of
glucose?
A. Changes of the colour from a pinky to dark blue of the slant surface of
agar
B. Changes of the colour from a pinky to red of the slant surface of agar
C. Changes of the colour from a pinky to dark blue in the column of agar and
slant
part of agar
D. Changes of the colour from a pinky to red in the column of agar
E. *Changes of the colour from a pinky to dark blue in the column of agar
963.
Select the ONE lettered option that MOST closely associated with inhibition
of
protein synthesis by binding to the 30S ribosomal subunit.
A. Penicillins
B. Chloramphenicol
C. Rifampin
D. Sulfonamides
E. *Aminoglycosides
964.
Serological identification is examination of bacterial species by _______:
A. Bacteria staining
B. laboratory animals
C. biochemical signs
D. character of growth on/in nutrient media
E. *agglutinating sera
965.
Some microorganisms lost their capacity for a synthesis of some necessary
growth factors, therefore they do not can grow on minimum nutrient media.
They are
called:
A. paratrophs
B. prototrophs
C. myxotrophs
D. oligotrophs
E. *auxotrophs
966.
What changes will be observed in Ressel medium for utilization of glucose and
lactose verification by Escherichia coli?
A. Changes of the colour from a pinky to dark blue in the column of agar
B. Changes of the colour from pinky to dark blue of the slant surface of agar
C. Changes of the colour from a pinky to red of the slant surface of agar
D. *Changes of the colour from a pinky to dark blue in the column of agar and
slant
surface of agar
E. Changes of the colour from a pinky to red of the column of agar
967.
The proportion of antibiotic resistant bacteria has increased along with the
widespread use of antibiotics. This is due to the fact that antibiotics:
A. are unstable in vivo
B. are mainly bacteriostatic in vivo
C. are powerful mutagens
D. All are correct
E. *act as agents of selection for resistant organisms
968.
The pure culture of microorganisms grows and develops at presence of oxygen
in an atmosphere (no less what 20 % oxygen). What group of microorganisms
according
to their respiration does this culture belong to?
A. Obligate anaerobes
B. Facultative anaerobes
C. Microaerophilic bacteria
D. Capneic bacteria
E. Obligate aerobes
969.
The pure culture of microorganisms grows and develops at presence of oxygen
in an atmosphere (no less what 20 % oxygen). What group of microorganisms
according
to their respiration does this culture belong to?
A. Obligate anaerobes
B. Facultative anaerobes
C. Microaerophilic bacteria
D. Capneic bacteria
E. *Obligate aerobes
970.
The structure of colonies can be examined by such method:
A. In an electronic microscope
B. By a "hanging" drop technique
C. In a phase-contrast microscope
D. By the immersion system of microscope
E. *In the passing light, small objective of microscope
971.
The wild strain of bacteria can synthesize all necessary for them substances
from
a limited number of organic compounds, for example, from glucose and ammonium
salts. They are called:
A. auxotrophs
B. paratrophs
C. myxotrophs
D. oligotrophs
E. *prototrophs
972.
What chemicals can you offer for separation of staphylococci and bacilli from
other microbes according to biological principle?
A. magnesium sulfate
B. aluminium nitrate
C. ascetic lead
D. oxalic acid
E. *7,5-10 % sodium chloride
973.
In MPB it is necessary to verify the peptolytic properties of bacteria. What
signs
will show the positive results – indole production?
A. to insert the indicator, saturated with a litmus; an indole is changed its
colour, it
becomes red
B. to insert the indicator paper, saturated with an oxalic acid; an indole is
changed
its colour, it becomes dark blue
C. to insert the indicator paper, saturated with a litmus; a indole is
changed its
colour, it becomes dark blue
D. the colour of medium becomes rose
E. *to insert the indicator paper, saturated with an oxalic acid; a indole is
changed
by its colour, it becomes rose
974.
What component is it necessary to take for carrying out presumptive
agglutination test for examination of bacterial antigens?
A.
B.
C.
D.
E.
Precipitating serum
Neutralizing serum
Hemolytic serum
no correct answer
*Agglutinating serum
975.
What end products do appear in the semi-solid Hiss media after sugar
utilization?
A. acid
B. alkali
C. gas
D. *acid and gas
E. fixing the finished products is not succeeded
976.
What indicator is utilized in a medium with an urea according to Christensen?
A. Andrede
B. Neutral red
C. Bromthymol blue
D. Thymol blue
E. Phenol red
977.
What indicator may be added into nutrient medium for cultivation of
anaerobes,
which can testify expedience of its use for inoculation of tested material?
A. Fuchsine
B. Brilliant green
C. Methylene green
D. Fenolrot
E. *Rezazurin
978.
What is a sign of proteolytic properties of bacteria in a column gelatin?
A. Formation of diffuse turbidity
B. Formation of pellicle
C. Dissolution of casein clot
D. Formation of sediment
E. *Gelatin liquefacience
979.
What is it necessary to take into consideration during collection of tested
material for isolation of anaerobic causative agents?
A. Warning neutralizing effect of antibodies
B. Warning influence of disinfectants
C. Warning changes of tinctorial properties
D. Warning changes of serologic properties
E. *Warning toxic effect of oxygen
980.
What microorganisms can produce such enzymes, as fibrinolysin,
streptodornase, streptokinase, which are used for patients’ treatment?
A. staphylococci
B. neisseria
C. branchamella
D. gemella
E. *streptococci
981.
What medium is used to isolate the culture of causative agents from sample of
patient with gas anaerobic infection?
A. Leffler medium
B. Petrov medium
C. Makkoy-Chepin medium
D. Petranyani medium
E. *Kitt-Tarozzi medium
982.
What changes of the Olkenitsky medium will be observed when an urea
producting microbes were inoculated in it?
A. appearance of red precipitate in the column of agar
B. appearance of greenish precipitate in the column of agar
C. to fix the urease production in this medium is impossible
D. appearance of brightly-violet precipitate in the column of agar
E. turn to pink
983.
Which of following is biological one to create anaerobic conditions for
cultivation of proper bacteria?
A. Zeissler method
B. Shukevich method
C. Veinberg method
D. Pasteur method
E. *Fortner method
984.
What substance is it necessary to add to peptone water for creation the
optimum
conditions for the isolation of cholerae vibrio?
A. 1 % sodium sulfate
B. 1 % acids
C. 1 % iron sulfate
D. 1 % sodium chloride
E. *1 % alkali
985.
What nutrient media is 7,5-10 % sodium chloride added in?
A. Buchin’s medium
B. Ru medium
C. Endo medium
D. Blood-sugar agar
E. *Yolk-salt agar
986.
What changes will be observed in Ressel medium to verify Salmonella typhi
utilization of glucose?
A. Changes of the colour from a pinky to dark blue of the slant surface of
agar
B. Changes of the colour from a pinky to red of the slant surface of agar
C. Changes of the colour from a pinky to dark blue in the column of agar and
slant
part of agar
D. Changes of the colour from a pinky to red in the column of agar
E. *Changes of the colour from a pinky to dark blue in the column of agar
987.
What nutrient media will you recommend to verify the hemolytic properties of
bacteria?
A. simple MPA
B. simple MPB
C. Endo medium
D. serum MPA
E. *blood MPA
988.
The Gram-positive and Gram-negative cell walls generally differ in that the
Gram-positives exclusively possess:
A. peptides
B. lipid
C. no correct answers
D. carbohydrates
E. *teichoic acids
989.
All of the following are enteroviruses EXCEPT:
A. coxsackieviruses B
B. *rhinoviruses
990.
991.
992.
993.
994.
995.
996.
997.
C. coxsackieviruses A
D. echoviruses
E. polioviruses
The prophylaxis of mumps is conducted by:
A. *live vaccine
B. immunoglobulin
C. toxoid
D. the killed vaccine
E. antitoxic serum.
For specific prophylaxis of a tick-borne encephalitis is used:
A. immunoglobulin
B. *formolated vaccine
C. toxoid
D. specific hyperimmune serum
E. live vaccine.
Choose among listed viruses which are not enteroviruses:
A. coxsackieviruses
B. echoviruses
C. *Rhinoviruses
D. polioviruses I
E. polioviruses II
Identify the true statement cited below concerning the Sabin (live) polio
vaccine.
A. Infectious progeny virus cannot be disseminated from the vaccinated
individual.
B. *The vaccine confers humoral and intestinal immunity.
C. Induction of lifelong immunity is not possible.
D. It is administered parenterally.
E. It can be given to immunodeficient individuals without reservation.
Retrospective diagnostics of measles and mumps includes:
A. CFT with specific serums
B. HIT with specific serums
C. *HIT with paired serums
D. rhinocytoscopy
E. NT (colour test) with with paired serums
Rabies can be acquired as a result of:
A. inhalation of aerosolized virus
B. a bite of a rabid animal
C. inoculation through mucous membranes
D. transplantation of infected tissue
E. *all of the above
Choose the true statement concerning the Sabin polio vaccine.
A. Induction of lifelong immunity is not possible.
B. Infectious progeny virus cannot be disseminated from the vaccinated
individual.
C. It is administered parenterally.
D. It can be given to immunodeficient individuals without reservation.
E. *The vaccine confers humoral and intestinal immunity.
For serological diagnostics of mumps such material is used:
A. saliva
B. blood
C. urine
D. spinal liquid
E. *paired serums.
998.
Which one of the following is a common feature of infection by
cytomegalovirus?
A. Rapid progression to encephalitis and death
B. Chronic progressive tissue damage for many years
C. *Dissemination to many body sites, including trans-placental passage
D. Severe vesiculation of epithelium and mucous membranes
E. Spontaneous recovery with life-long latency
999.
The vaccine for influenza has the following components:
A. *Hemagglutinin and neuraminidase of influenza A and B
B. Hemagglutinin and neuraminidase of influenza A,B, and C
C. Nucleoprotein and neuraminidase of influenza A,B, and C
D. Hemagglutinin and neuraminidase of influenza B only
E. Hemagglutinin and neuraminidase of influenza B and C
1000.
What characteristic is shared by the Salk and the Sabin vaccines?
A. Both are attenuated, active vaccines.
B. *Both are composed of virus serotypes 1, 2, and 3.
C. Both are administered by injection.
D. Both are capable of reversion to virulence
E. Both are shed in the feces of vaccinees.
1001.
What characteristic is correct concerning the Salk and the Sabin vaccines?
A. Both are shed in the feces of vaccinees.
B. Both are capable of reversion to virulence
C. Both are administered by injection.
D. *Both are composed of poliovirus serotypes 1, 2, and 3
E. Both are killed, active vaccines.
1002.
RS virus causes:
A. destruction of tissue culture
B. *formation of syncytio in tissue cultures
C. death of chicken embryo
D. hemagglutination of red cells
E. hemolysis
1003.
Infection by these viruses is preventable with a licensed vaccine
A. Coxsackievirus A
B. Coxsackievirus B
C. Enterovirus 72
D. *Poliovirus
E. Rhinovirus
1004.
Which of the listed viruses are associated with hepatitits?
A. Coxsackievirus A
B. Coxsackievirus B
C. *Enterovirus 72
D. Poliovirus
E. Rhinovirus
1005.
Bacteria use ____________ to defend themselves against bacteriophages.
A. macrophages
B. lymphokines
C. *restriction enzymes
D. antibodies
E. antibiotics
1006.
Rabies can be happened in human as a result of:
A. a bite of an animal with rabies
B. inhalation of aerosolized virus
C. inoculation through mucous membranes
D. transplantation of infected tissue
E. *all ANSWERs are correct
1007.
From which viruses do vaccine obtain ?
A. Coxsackievirus A
B. Coxsackievirus B
C. Enterovirus 68
D. *Poliovirus
E. Rhinovirus
1008.
What is the shape of rabies virus?
A. Spherical
B. Polygonal
C. *Bullet-shaped.
D. Tubular
E. Stick-like
1009.
The vaccine for influenza consists of the following components:
A. Hemagglutinin and neuraminidase of influenza B only
B. Hemagglutinin and neuraminidase of influenza A,B, and C
C. Nucleoprotein and neuraminidase of influenza A,B, and C
D. *Hemagglutinin and neuraminidase of influenza A and B
E. Hemagglutinin and neuraminidase of influenza B and C
1010.
Which of the listed viruses are associated with common cold?
A. Coxsackievirus A
B. Coxsackievirus B
C. Enterovirus 72
D. Poliovirus
E. *Rhinovirus
1011.
The nucleoprotein of the orthomyxoviruses is responsible for assignment of:
A. *Type classification
B. Subtype classification
C. Hemagglutinin classification
D. Neuraminidase classification
E. RNA polymerase classification
1012.
Which of the following infects oropharynx, but not the intestine because the
virus is sensitive to low pH?
A. *Rhinovirus
B. Poliovirus
C. Enterovirus 72
D. Coxsackievirus B
E. Coxsackievirus A
1013.
The following characterizes the genome of the orthomyxoviruses:
A. Nonsegmented RNA genome
B. *Segmented RNA genome
C. Nonsegmented DNA genome
D. Segmented DNA genome
E. Supercoiled double stranded DNA genome
1014.
Which of the following viruses are sensitive to low pH?
A. Coxsackievirus A
B. Coxsackievirus B
C. Poliovirus
D. *Rhinovirus
E. Enterovirus 72
1015.
The genome of the orthomyxoviruses contain:
A. Nonsegmented RNA genome
B. Nonsegmented DNA genome
C. *Segmented RNA genome
D. Segmented DNA genome
E. Supercoiled double stranded DNA genome
1016.
A respiratory disease spread by the droplet aerosol route during winter,
causing
severe headache, cough, fever, malaise and congestion is most compatible with
which of
the following as an etiological agent?
A. Mumps
B. Influenza C
C. Chicken pox
D. Hepatitis A Virus
E. *Influenza A virus
1017.
Immunization of an infant with this vaccine causes shedding of vaccine strain
virus in stool:
A. *Sabin vaccine
B. Salk vaccine
C. Measles
D. Influenza
E. Neither
1018.
After immunization of a children with this vaccine often we can find of
vaccine
strain virus in stool:
A. Influenza
B. Measles
C. Salk vaccine
D. *Sabin vaccine
E. None of the above
1019.
Paralytic disease is a risk factor when using this vaccine.
A. *Sabin vaccine
B. Salk vaccine
C. Influenza
D. Measles
E. None of the above
1020.
Influenza viruses are predominantly transmitted by:
A. The aerosol route during the summer
B. Close physical contact during the summer
C. The fecal-oral route during the winter
D. *The aerosol route and close physical contact during the winter
E. sexual route in winter
1021.
Vaccine associated poliomyelitis is a risk factor when using this vaccine.
A. *Sabin vaccine
B. Salk vaccine
C. Influenza
D. Measles
E. All of the above
1022.
During the epidemic of flu Influenza viruses are predominantly transmitted
by:
A. The aerosol route during the summer
B. Close physical contact during the summer
C. The fecal-oral route during the winter
D. *The aerosol route and close physical contact during the winter
E. through the bites of flies
1023.
Diseases causally associated with Epstein-Barr virus include:
A. *Burkitt’s Lymphoma and nasopharyngeal carcinoma
B. acquired immunodeficiency syndrome
C. adult T-cell leukemia
D. genital warts
E. none of the above
1024.
Choose among following diseases associated with Epstein-Barr virus:
A. acquired immunodeficiency syndrome
B. T-cell leukemia
C. *Infectious mononucleosis
D. herpes zoster
E. all of the above
1025.
Which of the following viruses has oncogenic ability?
A. CMV
B. HIV-1
C. *HTLV-1
D. HSV-1
E. Neither
1026.
Choose viruses which have oncogenic properties:
A. CMV
B. HIV-1
C. HSV-1
D. *HTLV-2
E. Neither
1027.
Which of the following viruses can cause B cell lymphoma in humans?
A. Herpes simplex virus
B. Human papilloma viruses
C. *Epstein-Barr Virus
D. Human T-cell lymphotropic virus
E. Hepatitis B virus
1028.
What test should be used for examination of Infleunza virus type?
A. Haemagglutination test
B. Haemagglutination inhibition test
C. *Complement fixation test
D. Precipitation test
E. Neutralization test
1029.
Herpes simplex virus type 1 most commonly causes cold sores. The site of
reactivation for this virus is the
A. vagus nerve
B. B lymphocyte
C. epidermal cell
D. *trigeminal nerve
E. eighth cranial nerve
1030.
In a minus-sense RNA virus:
A. the virus does not need RNA polymerase
B. *the virus has an RNA-dependent RNA polymerase
C. the virus's RNA acts as messenger RNA
D. the virus has a DNA-dependent RNA polymerase
E. the virus contains double-stranded RNA
1031.
Which antiviral agent is commonly used to combat herpes simplex virus
infections?
A. azidothymidine
B.
C.
D.
E.
*acyclovir
foscarnet
idoxuridine
interferon
1032.
What is the specimen from which herpes simplex virus is most likely to be
cultured:
A. saliva
B. spinal fluid.
C. *vesicle fluid.
D. swab from ulcer.
E. blood
1033.
Which of the following anti-viral drugs is used to prevent recurrent HSV
infections?
A. ribavirin
B. amantidine
C. phosphonoformate
D. *acyclovir
E. azidothymidine
1034.
Which of the following is not true?
A. viral nucleic acid can be single-stranded
B. viral nucleic acid can be double-stranded
C. viral genomes can be DNA
D. viral genomes can be RNA
E. *viruses contain DNA and RNA
1035.
At which specimen we can indicate herpes zoster virus more frequently?
A. blood
B. saliva
C. spinal fluid.
D. swab from ulce
E. *vesicle fluid.
1036.
The shape of rabies virus is:
A. *Bullet-shaped
B. Polygonal
C. Spherical
D. Stick-like
E. Tubular.
1037.
What is the reactivation of herpez zoster virus in an adult called?
A. infectious mononucleosis.
B. *shingles.
C. aphthous stomatitis.
D. chronic fatigue syndrome.
E. neuritis
1038.
Choose among listed a common feature of infection caused by cytomegalovirus?
A. Spontaneous recovery with life-long latency
B. Severe vesiculation of epithelium and mucous membranes
C. Rapid progression to encephalitis and death
D. *Dissemination to many body sites, including trans-placental passage
E. Chronic progressive tissue damage for many years
1039.
Which of the following clinical specimens can be used for the demonstration
of
rabies antigen by direct immunofluorescence antemortem?
A. salivary smears.
B. corneal smears.
C. blood
D. hairs
E. *a and d corect
1040.
What does disease occur after contamination by varicella zoster virus in
children?
A. aphthous stomatitis.
B. chronic fatigue syndrome
C. infectious mononucleosis.
D. *varicella.
E. neuritis
1041.
Where we can detect of rabies antigen by direct immunofluorescence
antemortem?
A. hippocampal neuron
B. *corneal scrapings.
C. blood
D. hairs
E. brain
1042.
Match yellow fever virus (Arbovirus) with its family.
A. Reoviridae
B. *Flaviviridae
C. Bunyaviridae
D. Togaviridae
E. Rhabdoviridae
1043.
Choose among following Family of yellow fever virus
A. Bunyaviridae
B. *Flaviviridae
C. Reoviridae
D. Rhabdoviridae
E. Togaviridae
1044.
Choose in the following statements the correct serological status at chronic
carrier of HBV and HDV:
A. anti-HBs positive, anti-HDV negative, anti-HBc negative
B. HBsAg negative, anti-HDV negative, anti-HBc negative
C. HBsAg positive, anti-HDV negative, anti-HBc positive
D. HBsAg negative, anti-HDV negative, anti-HBc positive
E. *HBsAg positive, anti-HDV positive, anti-HBc positive
1045.
Select the predominant route of transmission for Hepatitis B, C, and D
viruses.
A. airborne
B. *parenteral
C. contaminated food
D. fecal-oral
E. fomites
1046.
Which herpesviruses cause mononucleosis?
A. VZV-1 and HSV-2
B. *EBV and CMV
C. EBV and herpes B virus
D. CMV and VZV
E. HSV-1and HSV-2
1047.
Choose in the listed the predominant route of transmission for Hepatitis B
and C
viruses.
A. fomites
B. contaminated food
C. fecal-oral
D. *parenteral
E. airborne
1048.
Negri bodies are found in cells infected with:
A. *rabies virus.
B. vaccinia virus
C. fowlpox virus.
D. paramyxoviruses.
E. measles virus
1049.
Identify the clinical status of a patient who has the following hepatitis B
virus
(HBV) serology findings: anti-HBc positive, anti-HBs positive, HBsAg
negative.
A. Current acute infection with HBV
B. No past exposure to HBV
C. Recent past infection with HBV
D. *Chronic infectIon with HBV
E. Concurrent chronic infection with HBV and hepatitis D virus
1050.
Which hepatitis viruses are predominantly transmitted by the fecal-oral
route?
A. HAV, HBV, HEV
B. HAV, HBV
C. HBV, HCV, HDV
D. HBV, HDV, HEV
E. *HAV, HEV
1051.
At which diseases we can detect Negri bodies?
A. herpes
B. *rabies
C. smalpox
D. poliomyelitis
E. measles
1052.
Choose in the listed which hepatitis viruses are transmitted by the fecaloral
route?
A. HBV, HDV, HEV
B. *HAV, HEV
C. HBV, HCV, HDV
D. HAV, HBV
E. HAV, HBV, HEV
1053.
Which herpesvirus among listed does cause Burkitt’s lymphoma?
A. VZV
B. *EBV
C. polyovirus
D. HSV-2
E. HSV-1
1054.
Which hepatitis viruses are predominantly transmitted by the parenteral
route?
A. HAV, HBV, HEV
B. HAV, HBV
C. *HBV, HCV, HDV
D. HAV, HEV
E. HBV, HDV, HEV
1055.
An acute respiratory disease spread by the droplet aerosol route during
winter is
most compatible with which of the following as an etiological agent?
A. Chicken pox
B. Mumps
C. Influenza C
D. *Influenza A
E. Hepatitis B Virus
1056.
Which of the following statements best describes antigenic shift within the
orthomyxoviruses?
A. A minor antigenic change of the envelope proteins
B. *A major antigenic change of the envelope proteins
C. A medium antigenic change of the envelope proteins
D. A minor lipid change of the envelope proteins
E. A major lipid change of the envelope proteins
1057.
Choose in the listed statements best describes antigenic shift within the
orthomyxoviruses?
A. A minor lipid change of the envelope proteins
B. A major lipid change of the envelope proteins
C. A medium antigenic change of the envelope proteins
D. A minor antigenic change of the envelope proteins
E. *A major antigenic change of the envelope proteins
1058.
The virus associated with nasopharyngeal carcinoma is:
A. *EBV
B. flue virus
C. HIV
D. VZV
E. HHV-6
1059.
The vaccine for influenza has the following components:
A. *Haemagglutinin and neuramidase
B. Fusion protein
C. Viral RNA
D. Haemagglutinine
E. None of the above
1060.
A girl is infected with a paramyxovirus. The cytopathic effect for this virus
is
A. *syncytia formation
B. major destruction of the tissue culture monolayer
C. intranuclear inclusion formation
D. Negri body formation
E. cellular degranulation
1061.
Which of these statements is not true about the retroviridae?
A. *they contain one copy of plus-sense RNA
B. they contain two copies of plus-sense RNA
C. they contain a reverse transcriptase
D. they contain an RNA-dependent DNA polymerase
E. they incorporate into the host chromosome
1062.
Which of the following viruses cause/s genital infection?
A. Molluscum contagiosum.
B. Human papillomavirus type 6.
C. Herpes simplex virus type 2
D. adenovirus serotype 37
E. *All ANSWERs are correct
1063.
Which of the following viruses can cause prenatal infection?
A. Rubella
B. Cytomegalovirus
C. Varicella-zoster.
D. *All of the above.
E. none all of the above
1064.
A.
B.
C.
D.
E.
F.
The term virus was first used by
Koch
van Leewuenhoek
Fleming
*Pasteur
Redi
Redi
Which virus was the first one to be purposely eradicated from the face of the
1065.
Earth?
A. chickenpox
B. swinepox
C. *smallpox
D. cowpox
E. avianpox
1066.
The viral protein coat is also referred to as a(n
A. core
B. cell membrane
C. *capsid
D. envelope
E. cell wall
1067.
A virion refers to
A. a core
B. an envelope, if present
C. *all of the above
D. a coat
E. none of the above
1068.
Which of the following is not true about latency?
A. is a universal property of herpes viruses
B. has the ability to remain in host cells for long periods
C. retains the ability to replicate
D. is reactivated as a result of psychological or physical factors
E. *usually remains in nephrons
1069.
Which protein is predominantly responsible for attachment of the influenza
virus
to susceptible epithelial cells located in the upper respiratory tract?
A. Neuraminidase
B. *Hemagglutinin
C. Matrix protein
D. Nucleoprotein
E. Fusion protein
1070.
Depending on the virus, projections referred to as spikes may or may not
extend
from the viral envelope. These spikes are composed of
A. phospholipids
B. carbohydrates
C. *glycoproteins
D. glycolipids
E. proteins
1071.
Which of the following is not in the proper order for viral replication?
A. adsorption
B. *biosynthesis
C. penetration
D. maturation
E. release
1072.
Which of the following is the most common polyhedral shape of a virus?
A. bullet-shaped
B. dodecahedral
C. helical
D. complex
E. *icosahedral
1073.
Viral specificity takes into account all of the following, except
A. the specific receptors on the host cells
B. the availability of appropriate host enzymes
C. the specific attachment structures on the viral capsid or envelope
D. *the nature of the viral genome
E. the availability of host organelles for replication
1074.
Which of the following is not true about adsorption and the penetration of
bacteriophage T4?
A. adsorption is a chemical reaction
B. the tail fibers bend to allow the pins to touch the cell surface
C. adsorption requires specific receptor sites on the host cells
D. adsorption requires protein-recognition factors in the phage tail fibers
E. *the entire virus is delivered into the host cell
1075.
Which of the following is mismatched?
A. viscerotropic - infecting organs of the digestive tract
B. dermotropic - infecting the skin
C. neurotropic - infecting nervous tissue
D. pneumotropic - infecting the respiratory system
E. *genotropic - infecting the chromosome
1076.
Which of the following is mismatched?
A. *Adenovirus - infectious mononucleosis
B. HIV - AIDS
C. HIV - Kaposi's sarcoma
D. Coxsakcie virus - herpangina
E. Orthomixovirus - influensa
1077.
The International Committee on Taxonomy of Viruses allows naming of viruses
to include all of the following except
A. the place of discovery
B. the special trait
C. *the name of the discoverer
D. the morphological appearance
E. the physical size
1078.
If you had a phage suspension in a test tube, which of the following
techniques
would you use to determine the number of viruses in the tube?
A. *plaque assay
B. spectrophotometric analysis
C. count from electron micrographs
D. light microscopy and a counting chamber
E. most probable number
1079.
Which of the following specifically prevents the adsorption or biosynthesis
of
phages of the type whose DNA is already carried by the lysogen?
A. lysogenic cycle
B. temperate phages
C. *lysogenic conversion
D. lytic cycle
E. induction
1080.
Which of the following is not true about viral replication in animals?
A. adsorption is very specific
B. enveloped viruses have spikes that interact with host-cell surfaces
C. naked viruses have attachment sites on the surfaces of their capsid
D. *specificity involves the interaction of the virus with the animal cell
wall
E. both the nucleic acid and the capsid normally penetrate the animal cells
1081.
Which of the following is mismatched?
A. *immortal cell lines - continuous cell lines that contain genetically
identical cells
B. cell strain - genetically identical cells subcultured from a primary cell
line
C. primary cell cultures - come directly from the animal
D. subculturing - cells from an existing culture are transferred to fresh
media
E. continuous cell line - cells that reproduce for an extended number of
generations
1082.
Which of the following virus is not teratogenic in humans?
A. cytomegalovirus
B. rubella virus
C. herpes simplex virus type 2
D. herpes simplex virus type 1
E. *phage T4
1083.
Which of the following is not true about viroids?
A. they exist inside of cells without capsids or envelopes
B. *they produce protein from its genome
C. they do not require a helper phage
D. their genome is always copied in the host-cell nucleus
E. each consists of a single circular RNA molecule
1084.
Which of the following neurodegenerative diseases has not been proved to be
prion-caused?
A. kuru
B. scrapie
C. Creutzfeldt-Jakob disease
D. bovine spongiform encephalopathy
E. *Alzheimer's disease
1085.
Which of the following is mismatched?
A. cancer - an uncontrolled, invasive growth of abnormal cells
B. *malignant - a noncancerous growth
C. cancer cells - cells that divide repeatedly
D. tumor - a neoplasm or localized accumulation of cells
E. metastasize - tumors and tumor cells spreading to other tissues of the
body
1086.
Which of the following is not true about proto-oncogenes?
A. they can cause uncontrolled cell division
B. *they are needed for virus replication
C. they come from the normal host-cell chromosome during viral replicatio
D. they are under the control of a virus
E. they are similar to oncogenes
1087.
All viruses have
A. RNA
B. envelopes
C. *host specificity
D. glycoprotein spikes
E. DNA
1088.
Prions:
A. *are infectious particles not destroyed by DNase or RNase
B. are easily inactivated at 90 degrees centigrade
C. are also called viroids
D. are infectious pieces of RNA
E. is the name given to latent viruses
1089.
Which of the following will not support the growth of viruses?
A. embryonated eggs
B. continuous cell cultures
C. animals
D. primary cell cultures
E. *blood agar
1090.
What of the influenza-enveloped virus appear to be involved in attachment to
the
host cell receptor site?
A. Pili
B. Fimbriae
C. Flagellae
D. *Hemagluttinin
E. Neuraminidase
1091.
The hepadnaviruses such as hepatitis B virus are quite different from other
DNA
viruses with respect to genome replication. They replicate their DNA using:
A. Reverse transcriptase
B. *DNA-dependent DNA polymerase
C. RNase
D. RNA-dependent DNA polymerase
E. DNA ligase
1092.
Intracellular structures formed during many viral infections are:
A. Procaryotes
B. Chromosomal disruptions
C. *Inclusion bodies
D. Cytocidal bodies
E. All of the above
1093.
Which of the following has been linked to Kaposi's sarcoma?
A. Epstein-Barr virus
B. Human T-cell lymphotropic virus
C. Human immunodefficiency virus
D. Human papilloma virus
E. *Human herpes virus 8
1094.
Which of the following is the agent associated with development of
neurodegenrative disease in livestock and humans?
A. Viroids
B. virions
C. Virinos
D. *Prions
E. Viruses
1095.
Which of the following is necessary for a virus to reproduce
A. a high concentration of protein
B. *a living host cell
C. a vaccine
D. a high body temperature
E. sunlight, water, and food
1096.
What of the following is necessary for reproduction of viruses?
A. a high concentration of protein
B. a high body temperature
C. *a living host cell
D. a vaccine
E. sunlight, water, and food
1097.
Vaccines protect us against dangerous viral diseases by training the body to
recognize and destroy specific invading viruses. Vaccines are made from:
A. *weakened or dead versions of a dangerous virus.
B. antibiotics
C. human white blood cells.
D. medicines that cure the symptoms of viral diseases.
E. DNA of virus
1098.
Virus vaccines are made from:
A. antibiotics
B. *weakened or dead versions of a dangerous virus.
C. human white blood cells.
D. medicines that cure the symptoms of viral diseases.
E. DNA of virus
1099.
Which of the following diseases has not been reduced or eliminated by
vaccines
A. polio
B. smallpox
C. *AIDS
D. measles
E. mumps
1100.
Which of the following disease has not prevented by vaccines
A. *AIDS
B. measles
C. tick-born encephalitis
D. yellow fever
E. rabies
1101.
A natural vaccination happens when:
A. a doctor gives a person a measles vaccine.
B. *a person becomes immune to chicken pox after contracting it.
C. a person catches a cold.
D. after contracting influenza once, a person can contract it again.
E. a doctor gives a person a immunoglobulins.
1102.
A natural active immunity vaccination occurs after:
A. immunisation by live vaccine
B. introduction of immunoglobulins
C. introduction of patient serum
D. with breast milk
E. *contact with sick man
1103.
Viruses are carried within the fluids and tissues of the body. They are
spread
between people in different ways, including:
A. contact with contaminated blood.
B. insects that transmit infected blood.
C. contact with contaminated saliva or mucus.
D. contact with contaminated vaginal secrets
E. *all of the above.
1104.
Viruses are spread between people in different ways, including:
A. contact with contaminated blood.
B. insects that transmit infected blood.
C. contact with contaminated saliva or mucus.
D. contact with contaminated vaginal secrets
E. *all of the above.
1105.
The immune system protects the body from disease. Because the AIDS virus
weakens the immune system,
A. *people with AIDS are more vulnerable to other diseases.
B. the AIDS virus is spread very easily.
C. people with AIDS are immune to all other diseases.
D. the AIDS virus activates nonspecific resistance of organism
E. all of the above.
1106.
Because the AIDS virus weakens the immune system:
A. people with AIDS are immune to all other diseases.
B. *people with AIDS are more vulnerable to other diseases.
C. the AIDS virus is spread very easily.
D. the AIDS virus activates nonspecific resistance of organism
E. all ANSWERs are right
1107.
Viruses in the body are destroyed by:
A. red blood cells.
B. *white blood cells.
C. vaccines
D. antibodies
E. B cells
1108.
Some types of viruses are spread between animals and humans. An example of
this is:
A. *a dog infected with rabies can infect people by biting them.
B. all colds are spread from cats to people.
C. horses often spread chicken pox to children.
D. cows infected with measles will infect anyone who walks near it.
E. flu is spread from pigs to people.
1109.
What viruses spread between animals and humans.
A. horses often spread chicken pox to children
B. all colds are spread from cats to people.
C. *a dog infected with rabies can infect people by biting them
D. a cows infected with measles will infect anyone who walks near it.
E. flu is spread from pigs to people.
1110.
What is the current recommendation for drug therapy for an AIDS patie
A. One nucleoside reverse transcriptase inhibitor
B. One protease inhibitor
C. Two protease inhibitors
D. *One protease inhibitor and two nucleoside inhibitor
E. Two protease inhibitors and ? interferon
1111.
An individual will be diagnosed with AIDS when they have a positive HIV test
and:
A. Mononucleosis
B. *CD4-cell count of under 200 per milliliter of blood.
C. A T-cell count of under 500 per milliliter.
D. A B-cell count of under 400 per milliliter.
E. None of the above.
1112.
We can make the conclusion that some person has AIDS when he has a positive
HIV test and:
A. A T-cell count of under 500 per milliliter.
B. A B-cell count of under 400 per milliliter.
C. *CD4-cell count of under 200 per milliliter of blood.
D. Mononucleosis.
E. None of the above
1113.
Which of the following is/are congenital diseases occured in childrens born
to
women infected with rubella?
A. Neurosensory deafness.
B. Congenital heart disease
C. Retinopathy
D. Hepatosplenomegaly
E. *All of the above.
1114.
Which of the following viral genera is/are not arth-ropod-borne?
A. Alphavirus
B. Flavivirus
C. *Rubivirus
D. Togavirus
E. All of the above
1115.
Which of the following arthropodes act/s as vector for Japanese encephalitis
virus?
A. *mosquito.
B. flea
C. fly
D. tick
E. all ANSWERs are right
1116.
Choose insect as vector for Japanese encephalitis virus?
A. flea.
B. fly
C. tick
D. *mosquito
E. All of the above.
1117.
Which of the following act as amplifying hosts of Japanese encephalitis
virus?
A. Water birds and man.
B. Pigs and man.
C. *Water birds and pigs.
D. Cows and goats
E. Cats and dogs.
1118.
Which of the following are reservoir of Japanese encephalitis virus?
A. Cats and dogs.
B. Cows and goats.
C. Pigs and man.
D. Water birds and man.
E. *Water birds and pigs.
1119.
Which is the vector for West Nile virus?
A. flea
B. fly
C. louse
D. *mosquito
E. tick
1120.
Choose inside listed the vector for West Nile virus?
A. *mosquito
B. flea
C. fly
D. louse
E. batterfly
1121.
By which protein the influenza virus can attach to mucous cells of the upper
respiratory tract?
A.
B.
C.
D.
E.
Fusion protein
Nucleoprotein
Matrix protein
*Hemagglutinin
Neuraminidase
1122.
Which proteins of influenza viruses are included in vaccine preparations?
A. Neuraminidase
B. Neuraminidase and fusion proteins
C. Hemagglutinin
D. *Hemagglutinin and neuraminidase
E. Hemagglutinin, neuraminidase and fusion proteins
1123.
Which proteins of influenza viruses does flu vaccine contain ?
A. Hemagglutinin, neuraminidase and fusion proteins
B. Neuraminidase and fusion proteins
C. *Hemagglutinin and neuraminidase
D. Hemagglutinin
E. Neuraminidase
1124.
If an influenza infection has been diagnosed within a family, which antiviral
drug could be administered to other family members as a prophylactic measure?
A. Acyclovir
B. Ganciclovir
C. *Amantadine
D. Cyclosporin A
E. Foscarnet
1125.
Which antiviral drug could be administered to other family members as a
prophylactic measure if an influenza infection has been diagnosed within a
family?
A. Foscarnet
B. Ganciclovir
C. Cyclosporin A
D. *Amantadine
E. Acyclovir
1126.
Which of the following is true concerning the orthomyxoviruses?
A. Influenza A, B, and C cause epidemics
B. Neuraminidase and hemagglutinin are nucleocapsid proteins
C. *The genome is segmented, composed of eight RNA units
D. The matrix protein is important for vaccine inclusion
E. Dogs are a reservoir of influenza hemagglutinin and neuraminidase subtypes
1127.
Choose among listed a correct statement concerning the orthomyxoviruses?
A. Dogs are a reservoir of influenza hemagglutinin and neuraminidase subtypes
B. Influenza A, B, and C cause epidemics.
C. Neuraminidase and hemagglutinin are nucleocapsid proteins
D. *The genome is segmented, composed of eight RNA units
E. The matrix protein is important for vaccine inclusion
1128.
Which of the following statements best describes antigenic shift within the
orthomyxoviruses?
A. A minor antigenic change of the envelope proteins
B. *A major antigenic change of the envelope proteins
C. A medium antigenic change of the envelope proteins
D. A minor lipid change of the envelope proteins
E. A major lipid change of the envelope proteins
1129.
Choose among listed statements best describes antigenic shift within the
orthomyxoviruses?
A.
B.
C.
D.
E.
*A major antigenic change of the envelope proteins
A major lipid change of the envelope proteins
A medium antigenic change of the envelope proteins
A minor lipid change of the envelope proteins
A minor antigenic change of the envelope proteins.
1130.
The syncytial-forming capability that all members of the paramyxovirus family
have in common is a consequence of which of the following viral proteins?
A. hemagglutinin
B. neuraminidase
C. *fusion protein
D. polymerase
E. thymidine kinase
1131.
The syncytial-forming capability of the paramyxovirus is associated with:
A. *fusion protein
B. thymidine kinase
C. polymerase
D. neuraminidase
E. hemagglutinin
1132.
Antigenic drift of orthomyxoviruses is best characterized as:
A. A major change in the neuraminidase or hemagglutinin proteins
B. A major change in the matrix protein
C. *A minor change in the neuraminidase or hemagglutinin proteins
D. A minor change in the matrix protein
E. A minor change in envelope
1133.
Cause of antigenic drift of influenza A viruses is:
A. A major change in the hemagglutinin proteins
B. A major change in the matrix protein
C. A minor change in the matrix protein
D. A minor change in envelope
E. *A minor change in the neuraminidase or hemagglutinin proteins
1134.
The following is true of influenza viruses:
A. Both influenza A and B viruses have antigenic shift and drift
B. Neither influenza A nor B viruses have antigenic shift or drift
C. Influenza A and B viruses have antigenic shift; only influenza A has
antigenic
drift
D. Influenza A and B viruses have antigenic drift; only influenza B has
antigenic
shift
E. *Influenza A and B viruses have antigenic drift; only influenza A has
antigenic
shift
1135.
Which of the following is/are true about the influenza vaccine?
A. The vaccine is recommended for health care workers, patients with
cardiopulmonary complications, and geriatric patients.
B. The constitution of the vaccine changes every year.
C. Both influenza A and B virus strains are included in the vaccine.
D. The viruses used in the vaccine are propagated in embryonated chicken
eggs.
E. *All of the above ANSWERs.
1136.
Choose the correct statement about the influenza vaccine?
A. The viruses used in the vaccine are propagated in embryonated chicken
eggs.
B. Both influenza A and B virus strains are included in the vaccine.
C. The constitution of the vaccine changes every year.
D. The vaccine is recommended for health care workers, patients with
cardiopulmonary complications, and geriatric patients.
E. *All of the above ANSWERs.
1137.
Respiratory syncytial virus (RSV) is most associated with which of the
following
syndromes:
A. Bronchiolitis of young adults
B. *Bronchiolitis of young infants
C. Upper lobe infiltrates of young adults
D. Upper lobe infiltrates of young children
E. Lower lobe infiltrates of young adults
1138.
Respiratory syncytial virus (RSV) causes one of the following syndromes:
A. Bronchiolitis in adults
B. *Bronchiolitis in infants
C. Upper lobe infiltrates of young adults
D. Upper lobe infiltrates of young children
E. Lower lobe infiltrates in adults
1139.
The antiviral used for treatment of severe respiratory syncytial disease
(RSV)
infection is:
A. acyclovir
B. amantadine
C. *ribavirin
D. immune globulin
E. AZT
1140.
Which antiviral drug we can use for treatment of severe respiratory syncytial
disease (RSV) infection :
A. acyclovir
B. amantadine
C. AZT
D. immune globulin
E. *ribavirin
1141.
An adenovirus:
A. has a neuraminidase
B. *has a hemagglutinin
C. has a double shelled capsid
D. has an RNA genome
E. has an envelope
1142.
An adenovirus contains:
A. a neuraminidase
B. fibrinolysisn
C. a double shelled capsid
D. *a DNA genome
E. an envelope
1143.
Choose among following the method of detecting of a specific serotype of
adenovirus which have caused acute respiratory infection?
A. By detecting nuclear inclusions in infected cells collected in a throat
swab.
B. By detecting the presence of viral double stranded DNA.
C. By monitoring the response of the infection to steroid therapy
D. By using electron microscopy to detect viral particles in a throat swab.
E. *By using serotype antibody to test inhibition of hemagglutination .
1144.
The hemagglutinin of adenovirus is found in which viral protein?
A. The core
B. The hexon
C. *The penton
D. The M protein
E. The DNA protein
1145.
In which component of adenovirus the hemagglutinin is found?
A. The hexon
B. The core
C. The DNA protein
D. The M protein
E. *The penton with fiber
1146.
In addition to acute respiratory diseases, adenoviruses also cause:
A. Tumors in humans
B. Diabetes
C. Otitis media
D. Gastric ulcers
E. *Acute diarrhea in children
1147.
Which family contains rubella virus and what group of individuals is most at
risk
for severe sequelae from rubella infections?
A. Paramyxovirus family; grade school children
B. *Togavirus family; infants in utero during the first trimester
C. Paramyxovirus family; infants in utero during the first trimester
D. Togavirus family; grade school children
E. Togavirus family; middle age adults
1148.
Choose family which contains rubella virus and what individuals have most
risk
for dangerous complications from rubella infections?
A. Paramyxovirus family; grade school children
B. Paramyxovirus family; infants in utero during the first trimester
C. *Togavirus family; infants in utero during the first trimester
D. Togavirus family; grade school children
E. Togavirus family; middle age adults
1149.
Rubella infection causes long term serious sequelae in which of the following
populations?
A. Women of childbearing age
B. *Children infected in the first trimester in utero
C. Children infected during the first grade (age 6-7 yrs)
D. Children who have received the attenuated vaccine
E. Children who have received the interferon
1150.
For whom rubella infection causes long term dangerous complications?
A. Children who have received the interferon
B. Children who have received the attenuated vaccine
C. Children infected during the first grade (age 6-7 yrs)
D. *Children infected in the first trimester in utero
E. Women of childbearing age
1151.
Which of the following statements accurately describes the envelope
constituents
of the parainfluenza and mumps viruses?
A. Hemagglutinin, but no neuraminidase or fusion proteins present
B. Neuraminidase, but no hemagglutinin protein present
C. Hemagglutinin and neuraminidase, but no fusion protein present
D. *Hemagglutinin, neuraminidase and fusion proteins present
E. Fusion protein, but no hemagglutinin or neuraminidase present
1152.
Which of the following proteins are present in the envelope of the
parainfluenza
and mumps viruses?
A. Hemagglutinin, but no neuraminidase or fusion proteins present
B. *Hemagglutinin, neuraminidase and fusion proteins present
C. Hemagglutinin and neuraminidase, but no fusion protein present
D. Neuraminidase, but no hemagglutinin protein present
E. Fusion protein, but no hemagglutinin or neuraminidase present
1153.
Measles is a member of the paramyxovirus family, but it differs from mumps
and the parainfluenza viruses structurally with respect to envelope proteins.
How do
they differ?
A. Measles has a neuraminidase but no hemagglutinin; mumps and parainfluenza
have only hemagglutinin
B. Measles has no hemagglutinin or neuraminidase, mumps and parainfluenza
have
only neuraminidase
C. Measles has a neuraminidase but no hemagglutinin; mumps and parainfluenza
have hemagglutinin and neuraminidase
D. *Measles has a hemagglutinin but no neuraminidase; mumps and parainfluenza
have hemagglutinin and neuraminidase
E. Measles has no hemagglutinin or neuraminidase, mumps and parainfluenza
have
hemagglutinin and no neuraminidase
1154.
Find the correct statement about envelope proteins of measles, mumps and the
parainfluenza viruses:
A. *Measles has a hemagglutinin but no neuraminidase; mumps and parainfluenza
have hemagglutinin and neuraminidase
B. Measles has no hemagglutinin or neuraminidase, mumps and parainfluenza
have
only neuraminidase
C. Measles has a neuraminidase but no hemagglutinin; mumps and parainfluenza
have hemagglutinin and neuraminidase
D. Measles has a neuraminidase but no hemagglutinin; mumps and parainfluenza
have only hemagglutinin
E. Measles has no hemagglutinin or neuraminidase, mumps and parainfluenza
have
hemagglutinin and no neuraminidase
1155.
Why is infection with HDV of concern:
A. *HDV exacerbates HBV infection
B. HDV exacerbates HAV infection
C. HDV exacerbates HCV infection
D. HDV exacerbates HEV infection
E. HDV itself causes severe viral hepatitis
1156.
When infection of HDV is most dangerous?
A. When HDV is together with HAV
B. When HDV is together with HEV
C. When HDV is together with HCV
D. *When HDV is together with HBV infection
E. When HDV itself causes severe viral hepatitis
1157.
The serological profile following HBV vaccination is:
A. Anti HBc positive
B. Anti HBe positive
C. Anti HBs and anti HBc positive
D. *Anti HBs positive
E. Anti HBs positive, anti HBc positive and anti HBe positive
1158.
After HBV vaccination the serological profile of the person is:
A. Anti HBc positive
B. Anti HBe positive
C. *Anti HBs positive
D. Anti HBs and anti HBc positive
E. Anti HBs positive, anti HBc positive and anti HBe positive
1159.
The presence of anti-HCV antibody in a person’s serum indicates:
A.
B.
C.
D.
E.
Past infection, currently immune to HCV
Past vaccination status with HCV
*Current or chronic infection with HCV
Hepatitis B must be present as a helper virus in the hepatocytes
The patient’s blood is safe for transfusion
1160.
When we can indicate anti-HCV antibody in a person’s serum:
A. *Current or chronic infection with HCV
B. Hepatitis B must be present as a helper virus in the hepatocytes
C. Past vaccination status with HCV
D. Past infection, currently immune to HCV
E. The patient’s blood is safe for transfusion
1161.
What would be the serological status of a patient successfully immunized with
the HBV vaccine?
A. HbsAg positive; anti-HBs positive
B. anti-HBc positive; anti-HBs positive
C. anti-HBc positive; HbsAg positive
D. *anti-HBc negative, anti-HBs positive
E. anti-Hbe positive
1162.
What can we detect in the patient’s serum after immunization with the HBV
vaccine?
A. HbsAg positive
B. anti-HBc positive; anti-HBs positive
C. anti-HBc positive; HbsAg positive
D. anti-Hbe positive
E. *anti-HBc negative, anti-HBs positive
1163.
A chronic carrier of HBV and HDV may have the following serological status:
A. *HBsAg positive, anti-HDV positive, anti-HBc positive
B. HBsAg negative, anti-HDV negative, anti-HBc negative
C. anti-HBs positive, anti-HDV negative, anti-HBc negative
D. HBsAg positive, anti-HDV negative, anti-HBc positive
E. HBsAg negative, anti-HDV negative, anti-HBc positive
1164.
Choose in the listed which hepatitis viruses are transmitted by the
parenteral
mechanism?
A. HAV, HBV, HEV
B. HAV, HBV
C. HAV, HEV
D. *HBV, HCV, HDV
E. HBV, HDV, HEV
1165.
A patient with acute HBV infection may have the following serologic profile:
A. HBsAg negative, anti-HBs antibody positive, anti-HBc antibody positive
B. *HBsAg positive, anti-HBs antibody negative, anti-HBc antibody positive
C. HBsAg positive, anti-HBs antibody positive, anti-HBc antibody positive
D. HBsAg negative, anti-HBs antibody negative, anti-HBc antibody positive
E. HBsAg negative, anti-HBs antibody positive, anti-HBc antibody negative
1166.
Find among listed which serologic profile a patient with acute HBV infection
may have:
A. HBsAg negative, anti-HBs antibody positive, anti-HBc antibody positive
B. HBsAg negative, anti-HBs antibody negative, anti-HBc antibody positive
C. HBsAg positive, anti-HBs antibody positive, anti-HBc antibody positive
D. *HBsAg positive, anti-HBs antibody negative, anti-HBc antibody positive.
E. HBsAg negative, anti-HBs antibody positive, anti-HBc antibody negative
1167.
Which of the following hepatitis viruses is classified as defective, (e.g.
the virus
will replicate only in the presence of another different, replicating virus)?
A. Hepatitis A virus (HAV)
B. Hepatitis B virus (HBV)
C. Hepatitis C virus (HCV)
D. *Hepatitis D virus (HDV)
E. Hepatitis E virus (HEV)
1168.
Each of the following pathogens is likely to establish chronic or latent
infection
EXEPT:
A. Cytomegalovirus
B. *Hepatitis A virus
C. Hepatitis B virus
D. Herpes simplex virus I
E. Herpes simplex virus I I
1169.
Choose among the following hepatitis viruses which is defective:
A. *Hepatitis D virus (HDV)
B. Hepatitis C virus (HCV)
C. Hepatitis E virus (HEV)
D. Hepatitis B virus (HBV)
E. Hepatitis A virus (HAV)
1170.
Each of the following clinical syndromes is associated with infection by
picornaviruses EXCEPT:
A. myocarditis/pericarditis
B. hepatitis
C. *mononucleosis
D. minor febrile ilness
E. meningitis
1171.
For indication of the viruses in tested material we can use all tests EXCEPT:
A. *Complement fixation test
B. Haemagglutination test
C. Haemadsorption test
D. Inoculation of laboratory animal
E. Inoculation of chicken embryo
1172.
There are such components of haemagglutination test:
A. *Viruses, chicken red cells, electrolyte
B. Viruses, chicken red cells
C. Viruses, chicken red cells, specific serum
D. Viruses, chicken red cells, paired patient’s serums
E. Viruses, chicken red cells, viral diagnosticum, electrolyte
1173.
Choose necessary components of haemagglutination inhibition test for
virologic
diagnosis:
A. Unknown viruses, erythrocytes, electrolyte, specific immune serum,
complement
B. Patient’s paired serums, viral diagnosticum, erythrocytes, electrolyte
C. Patient’s paired serums, unknown viruses, erythrocytes, electrolyte
D. Known viruses, erythrocytes, electrolyte, specific immune serum
E. *Unknown viruses, erythrocytes, electrolyte, specific immune antiviral
serum
1174.
About 5 % of adults secrete this virus in urine:
A. *CMV
B. EBV
C. HSV-2
D. HHV-6
E. VZV
1175.
The specimen from which varicella-zocter virus is most likely to be recovered
is:
A. Saliva
B. Swab from ulcer
C. *Vesicle fluid
D. Spinal fluid
E. Serum
1176.
From which specimen herpes simplex virus type II is most likely to be
recovered:
A. Saliva
B. Serum
C. Spinal fluid
D. Swab from ulcer
E. *Vesicle fluid
1177.
What is the BEST explanation for the selective action of acyclovir in herpes
simplex virus-infected cells?
A. Acyclovir binds specifically to herpesvirus receptors on the cell surface
B. *Viral phosphokinase (thymidine kinase) phosphorylates acyclovir more
efficiently than does the host cell phosphokinase
C. Acyclovir exhibits the RNA polymerase in the viral particle
D. Acyclovir blocks the matrix protein of the virus, thereby preventing
release by
budding
E. Acyclovir inhibits capsid formation
1178.
What is the mechanism of the selective action of acyclovir in herpes simplex
virus-infected cells?
A. Acyclovir binds specifically to herpesvirus receptors on the cell surface
B. Acyclovir blocks the matrix protein of the virus, thereby preventing
release by
budding
C. Acyclovir exhibits the RNA polymerase in the viral particle
D. Acyclovir inhibits capsid formation
E. *Viral phosphokinase (thymidine kinase) phosphorylates acyclovir more
efficiently than does the host cell phosphokinase
1179.
How are the herpes viruses classified?
A. Single stranded DNA, non enveloped
B. *Double stranded DNA, enveloped
C. Positive stranded RNA, enveloped
D. Positive stranded RNA, non enveloped
E. Double stranded DNA, non enveloped
1180.
Which viruses have tegument?
A. Polioviruses
B. *Herpesviruses
C. Adenoviruses
D. Influenza viruses
E. Measels viruses
1181.
Which one of the following is a common feature of infection by herpes simplex
virus?
A. Rapid progression to encephalitis and death
B. Chronic progressive tissue damage for many years
C. Dissemination to many body sites, including trans-placental passage
D. *Spontaneous recovery with life-long latency
E. Chronic progressive fatigue
1182.
Choose among listed a most common feature of infection caused by herpes
simplex virus?
A.
B.
C.
D.
E.
1183.
A.
B.
C.
D.
E.
1184.
A.
B.
C.
D.
E.
1185.
A.
B.
C.
D.
E.
1186.
A.
B.
C.
D.
E.
1187.
A.
B.
C.
D.
E.
1188.
A.
B.
C.
D.
E.
1189.
A.
B.
C.
D.
E.
1190.
A.
B.
C.
D.
Chronic progressive fatigue
Chronic progressive tissue damage for many years
Dissemination to many body sites, including trans-placental passage
Rapid progression to encephalitis and death
*Spontaneous recovery with life-long latency
Latency is a feature of which group of viruses?
Polioviruses
Poxviruses
*Herpesviruses
Paramyxoviruses
Hepatitis A Viruses
Choose among listed which viruses don’t cause latency infection.
Cytomegalovirus
Herpes simplex virus
Varicella-zoster virus
*Polioviruses
Ebstain-Barr virus
The primary means of spread of varicella zoster virus is:
saliva
direct contact with lesions
*respiratory droplets
fecal-oral route
direct contact with sick child
Choose among listed the primary means of spread of varicella zoster virus
saliva
*respiratory droplets
fecal-oral route
direct contact with sick child
direct contact with lesions
The most common viral cause of congenital defects is:
*cytomegalovirus
mumps virus
measles virus
herpes simplex virus
adenovirus
Which virus does cause congenital defects?
adenovirus
*rubivirus
herpes simplex virus
measles virus
mumps virus
Exanthem subitum (roseola) is caused by:
*human herpes virus 6
human herpes virus 7
measles virus
varicella zoster virus
cytomegalovirus
Which virus does cause gingivostomatitis?
cytomegalovirus
polyovirus
*herpes simplex virus 1
measles virus
E. mumps virus
1191.
Which virus does cause genital herpes?
A. Herpes simplex virus-1
B. *Herpes simplex virus-2
C. cytomegalovirus
D. human herpes virus 6
E. human herpes virus 8
1192.
For which of the following viruses is the presence of heterophile antibody
diagnostic?
A. *EBV
B. CMV
C. HSV-1
D. HHV-7
E. varicella zoster virus
1193.
Which of the following viruses can stimulate production of the heterophile
antibodies?
A. CMV
B. *EBV
C. HSV-1
D. HHV-7
E. varicella zoster virus
1194.
Which virus is most likely to infect B lymphocytes?
A. Human immunodeficiency virus
B. *Epstein Barr virus
C. Hepatitis B virus
D. Human T cell leukemia virus
E. Hepatitis A virus
1195.
Which of the following virus is associated with Kaposi’s sarkoma?
A. Adenovirus
B. Hepatitis A virus
C. Hepatitis B virus
D. Human T cell leukemia virus
E. *Human herpesvirus 8
1196.
Which of the following is NOT an effect of acyclovir?
A. *Elimination of latent virus
B. Suppression of recurrent herpes infections, if taken daily in high doses
C. Activation by a viral enzyme inside infected cells
D. Active topically or orally or intravenously
E. Active for treatment all herpes infections
1197.
What tested material should be taken for serologic diagnosis of viral
infection?
A. saliva
B. swabs from a throat
C. *paired sera
D. blood
E. feces
1198.
The smallest known viruses are:
A. Adenovirus
B. *Picornavirus
C. Enterovirus
D. Orthomyxovirus
E. Paramyxovirus
1199.
Which viruses have bullet-shaped form?
A.
B.
C.
D.
E.
Adenovirus
Orthomyxovirus
Paramyxovirus
*Rhabdovirus
Rotavirus
1200.
What is main component of all mammalian viruses?
A. an envelope
B. a polymerase
C. an icosahedral capsid
D. a lipid
E. *a nucleic acid
1201.
Retroviruses are unique among all viruses that infect humans because only
retroviruses:
A. are associated with human cancers
B. *reverse transcriptase which transcribe their genetic material
C. are enveloped viruses
D. contain specific surface antigens
E. contain segmented RNA
1202.
Which HIV protein molecules do interact with CD4 receptors of lympocytes?
A. p 8
B. p 17
C. p 24
D. gp 41
E. *gp 120
1203.
Azidothymidine (AZT) inhibits retroviral replication by:
A. blocking proteolysis by the viral protease
B. inhibiting proteolysis by the cellular protease
C. *causing chain termination during reverse transcription
D. binding the cell surface receptor and preventing viral entry
E. binding viral integrase protein to prevent integration into the host
chromosome
1204.
What test is used for primary diagnosis of HIV carriers?
A. indirect haemagglutination test
B. complement fixation test
C. *ELISA
D. neutralization test
E. haemagglutination inhibition test
1205.
Within which of the following size ranges do most virions fall?
A. 2-35 nm
B. *20-350 nm
C. 20-1,000 nm
D. 10-200 µm
E. 200-350 µm
1206.
An alternative term for the protein coat of a virus particle is:
A. *capsid
B. capside
C. capsule
D. caspid
E. cuspid
1207.
An alternative term for 'virus particle' is:
A. virian
B. virin
C. *virion
D. viron
E. virone
1208.
Which of the following terms is used to describe a visible clear area,
indicating
the death of virus-infected cells, in a layer of uninfected cells?
A. placebo
B. plack
C. colony
D. planchette
E. *plaque
1209.
Coxsackie B Virus is associated with the following
A. Paralytic illness
B. Myocarditis
C. Bornholm's disease
D. Meningitis
E. *All
1210.
A multinucleate giant cell formed as a result of cell fusion is called a:
A. sincitium
B. sincity
C. sinecytium
D. *syncytium
E. synsytium
1211.
Which of the following molecules is the cell receptor for human
immunodeficiency virus-1?
A. CD1
B. *CD4
C. CD44
D. ICAM-1
E. ICAM-4
1212.
The clumping of red blood cells by virus particles
A. *haemagglutination
B. haemadsorption
C. haemogglutination
D. hemygglutination
E. hemigglutination
1213.
Reverse transcriptases are:
A. DNA-dependent DNA polymerases
B. DNA-dependent RNA polymerases
C. *RNA-dependent DNA polymerases
D. RNA-dependent RNA polymerases
1214.
The release of the virus genome from a virion when
called:
A. decoating
B. *uncoating
C. undercoating
D. undressing
E. unicoating
1215.
A phage genome that resides in a bacterial host in
a:
A. prephage
B. prephagenome
C. *prophage
D. prophagenome
E. provirus
1216.
The number of capsomeres in the herpesvirus capsid
A.
B.
C.
D.
E.
1217.
is the:
A.
B.
C.
D.
E.
1218.
A.
B.
C.
D.
E.
1219.
A.
B.
C.
is known as:
a cell is infected is
a latent state is known as
is:
D.
E.
1220.
A.
B.
C.
D.
E.
1221.
A.
B.
C.
D.
E.
1222.
A.
B.
C.
D.
E.
1223.
A.
B.
C.
D.
E.
1224.
A.
B.
C.
132
150
158
*162
168
In the herpesvirus virion the layer of protein between the capsid and the
envelope
tegament
tegumeant
*tegument
tegumint
togament
HSV and varicella-zoster virus initiate latent infections in:
endothelial cells
epithelial cells
B lymphocytes
*neurones
T lymphocytes
SV40 is the abbreviation of:
simbian virus 40
*simian virus 40
similar virus 40
symbiotic virus 40
syncytial virus 40
The virus associated with adult T cell leukaemia is:
human immunodeficiency virus 1
human immunodeficiency virus 2
*human T lymphotropic virus 1
human T lymphotropic virus 2
human T lymphotropic virus 3
Retroviral oncogenes were derived from cell genes known as:
pre-oncogenes
pro-oncogenes
proper-oncogenes
*proto-oncogenes
prototype-oncogenes
The attenuated polio vaccine was developed by:
Maurice Chevalierc
Albert Einstein
Maurice Hilleman
*Albert Sabin
Jonas Salk
Aciclovir is used against:
hepatitis B virus
hepatitis C virus
*herpesviruses
HIV
influenza virus
The inactivated polio vaccine was developed by:
Maurice Chevalier
Albert Einstein
Maurice Hilleman
D. Albert Sabin
E. *Jonas Salk
1225.
The target viral activity of AZT is:
A. entry into the cell
B. *reverse transcription
C. DNA replication
D. transcription
E. translation
1226.
Viral genetic material can be composed of
A. single-stranded RNA
B. single-stranded DNA
C. double-stranded RNA
D. double-stranded DNA
E. *All of these
1227.
A bacteriophage may infect a host cell without producing progeny in a
___________infection.
A. lytic
B. virulent
C. nucleic
D. *lysogenic
E. deficient
1228.
The _____________ state of some human viruses is where the viral DNA is
hidden within the host chromosomes.
A. *lysogenic
B. latent
C. invasive
D. lytic
E. virulent
1229.
A ____________ is a protein that can take one of two forms, one of which
causes disease.
A. viroid
B. *prion
C. restriction enzyme
D. virion
E. bacteriophage
1230.
Viruses
A. Contain both DNA and RNA
B. *May have an envelope
C. Have their own metabolism
D. May contain enzymes for nutrition
E. Cell wall
1231.
The following are DNA viruses
A. *Herpesviruses
B. Orthomyxoviruses
C. Enteroviruses
D. Flaviviruses
E. Rotaviruses
1232.
The following are RNA viruses
A. *Picornaviruses
B. Adenoviruses
C. Papillomaviruses
D. Poxviruses
1233.
1234.
1235.
1236.
1237.
1238.
1239.
1240.
1241.
E. Herpesviruses
Live attenuated vaccines are available against the following viruses
A. Influenza A Virus
B. Hepatitis B Virus
C. *Rubella Virus
D. Rhabies Virus
E. Tick-born encephalitis Virus
The following antiviral agents is active against the following virus, except:
A. Lamivudine and HIV
B. Lamivudine and HBV
C. *Amantidine and influenza B virus
D. Ribavirin and RSV
E. Acyclovir and HSV
Herpes Simplex Encephalitis
A. Commonly affect the spinal cord
B. Is usually diagnosed by culture of the CSF
C. May be diagnosed by precipitation of the CSF
D. Should be treated with ribavirin
E. *May be diagnosed by the finding of specific antibody in the CSF
The following statements are true of Varicella-Zoster Virus
A. Causes a maculopapular rash
B. Respond to AZT therapy
C. *Remains latent in sensory ganglia following primary infection
D. Recurrent episodes of Shingles usually occur
E. Patients with shingles are not infectious
Cytomegalovirus (CMV), all correct, except:
A. Primary infection is usually symptomatic
B. An infectious mononucleosis-like syndrome may occur during primary
infection.
C. May cause severe infection in immunocompromised individuals
D. Is not teratogenic
E. *Causes parotitis
Epstein-Barr Virus (EBV) is associated with all, except:
A. Infectious Mononucleosis
B. *Hepatitis B
C. Burkitt's lymphoma
D. Nasopharyngeal carcinoma
E. Oral leukoplakia
HHV-6 is associated with
A. Burkitt's lymphoma
B. *Roseala Infantum
C. Kaposi's Sarcoma
D. Infectious Mononucleosis
E. Oral leukoplakia
Adenoviruses, all are correct, except:
A. *Are associated with genital cancers
B. May cause gastroenteritis
C. May cause conjunctivitis
D. May cause pneumonia
E. May cause urethritis
Human Papillomaviruses
A. HPV-6 and HPV-11 are associated with genital cancers
B. *HPV-16 and HPV-18 are associated with genital cancers
1242.
1243.
1244.
1245.
1246.
1247.
1248.
1249.
C. Warts caused by papillomaviruses may respond to antibiotics therapy
D. Papillomavirus infection is commonly diagnosed by viral culture
E. Are associated with progressive multifocal leucoencephalopathy (PML)
Influenza A Virus, all are correct, except
A. May undergo antigenic shift and antigenic drift
B. May cause pandemics
C. Respond to rimantidine
D. Respond to neuraminidase inhibitors
E. *Vaccination confers lifelong protection
Paramyxoviruses may cause
A. *Croup
B. Maculopapular rash
C. Gastritis
D. Hepatitis
E. Diarrhoea
Respiratory Syncytial Virus (RSV), all are correct, except:
A. Has only F protein
B. May cause bronchiolitis
C. May cause croup
D. May cause pneumonia
E. *May be prevented by vaccination
Measles Virus Infection
A. Causes a vesicular rash
B. May cause parotitis
C. May respond to acyclovir
D. May be prevented by toxoid
E. *May be prevented by vaccination
Human T-lymphotropic virus 1 (HTLV-1) is associated with
A. Burkitt's lymphoma
B. *Adult T-cell lymphoma
C. Kaposi's Sarcoma
D. Multiple Sclerosis
E. Hodgkin's lymphoma
HIV Infection may lead to
A. Dementia
B. Chronic Diarrhoea
C. CMV retinitis
D. Oesophageal candidiasis
E. *All are correct
The following may be useful for prognostic purposes in HIV-infected
individuals
A. HIV envelope antibody level
B. HIV-gp120 antigen
C. HIV-gp41 antigen
D. *CD4 count
E. HIV pro-viral DNA in leucocytes
A chronic carrier state may occur in the following:
A. Measles Infection
B. Hepatitis A
C. *Hepatitis B
D. Hepatitis E
E. Smallpox Infection
1250.
The following markers are usually present in a hepatitis B carrier with
chronic
active hepatitis, except:
A. HbeAg
B. Anti-HBc IgG
C. *Anti-HBc IgM
D. HBV-DNA
E. HbsAg
1251.
The following statements are true, except:
A. Chronic HBV infection may respond to interferon therapy
B. Chronic HCV infection may respond to interferon therapy
C. Chronic HCV infection may respond to ribavirin therapy
D. Hepatitis Delta infection may be prevented by vaccination against HBV
E. *Hepatitis E Infection may be prevented by vaccination
1252.
The following virus can be transmitted by fecal-oral way
A. *HAV
B. HIV
C. HTLV-1
D. HBV
E. HCV
1253.
Regarding viral infection of the central nervous system (CNS), all are
correct,
except;
A. Meningitis may occur together with encephalitis
B. Enteroviruses are one of the commonest causes of CNS infections in
childhood
C. *Mosquitos are the vector of tick-born encephalitis viruses
D. Measles encephalitis is a postinfectious encephalomyelitis
E. The detection of antibody in the CSF is a useful diagnostic marker
1254.
One of the following viruses is associated with gastroenteritis
A. Measles viruses
B. RS- viruses
C. Herpes viruses
D. Poxviruses
E. *Rotaviruses
1255.
The following viruses are transmitted from animals to humans
A. *Rabies Virus
B. Polioviruses
C. CMV
D. Measles Virus
E. Rotavirus
1256.
The following is true of rabies virus
A. The majority of cases world-wide result from bat bites
B. *Infection may be prevented by active and passive immunisation
C. Human Rabies vaccine is a live attenuated vaccine
D. The animal reservoir is the same in all country
E. May be diagnosed only postmortem
1257.
Viruses may contain, except
A. DNA
B. RNA
C. Glycoprotein
D. Enzymes
E. *Cell wall
1258.
Viruses are
A. *Obligate intracellular parasites
B.
C.
D.
E.
May divide by binary fission
Have their own metabolism
May contain plasmides
May have a peptidoglycan
1259.
The following are direct detection methods
A. *Detection of rotavirus antigen in faecal specimens
B. Complement fixation test with patient’s serum
C. Radal hemolysis test
D. Indirect hemagglutination test
E. Coombs test
1260.
A serological diagnosis of a primary viral infection may be made by
A. Detection of viral-specific IgA
B. Detection of viral-specific IgD
C. Detection of viral-specific IgE
D. *Detection of viral-specific IgM
E. Detection of viral-specific IgG
1261.
Poliovirus can be typed by
A. Single radial haemolysis (SRH)
B. Haemagglutination inhibition test (HAI)
C. Haemagglutination test
D. *Neutralization test
E. Cytopathic (CPE) effect
1262.
Immunofluorescence techniques can be used to detect the following directly
from the specimen.
A. Chlamydia
B. CMV
C. *All are correct
D. Influenza virus
E. Rabies virus
1263.
The following statements are true for the haemagglutination-inhibition (HAI)
test, except
A. *Not a quantitative test
B. Treatment of patient serum is necessary to remove non-specific inhibitors
C. Animal blood is necessary
D. Usually more specific than complement fixation tests (CFT)
E. May be used for the diagnosis of rubella infection
1264.
Regarding cell culture
A. Viruses can only be cultured using cell lines
B. The presence of cytopathic effect is the only way to detect a virus
C. *The neutralization test is the mainstay of identification of a poliovirus
isolate
D. The haemagglutination inhibition test is the mainstay of identification of
a
respiratory syncytial virus (RSV) isolate
E. Whole blood is the specimen of choice for many common viruses
1265.
The following viruses are not associated with respiratory infections
A. *Rotaviruses
B. Adenoviruses
C. Influenza viruses
D. Parainfluenza viruses
E. RSV
1266.
The following viruses are transmitted from animals to humans
A. *Influenza A H5N1, Rabies
B. HTLV-1, HIV
C. Influenza B, RS virus
D. Poliomyelitis, rhinoviruses
E. HBV, HDV
1267.
A chronic carrier state may occur in the following, except:
A. *Hepatitis A
B. Hepatitis B
C. Hepatitis C
D. Hepatitis Delta
E. Hepatitis G
1268.
The following viruses are transmitted by blood
A. Rubella
B. Polyomielitis
C. Measles
D. HEV
E. *HCV
1269.
The following may be useful for prognostic purposes in HIV-infected
individuals
A. HIV envelope antibody
B. HIV-p17 antigen
C. *CD4 count
D. CD8 count
E. HIV viral ligase
1270.
The following markers are usually present in a hepatitis B carrier with
chronic
active hepatitis
A. Anti-HBsAg IgM
B. HBxAg
C. Anti-HBc IgM
D. HBcAg
E. *HBeAg
1271.
Regarding rash illnesses
A. *Varicella-zoster virus (VZV) infection may cause a vesicular rash
B. The rash caused by parvovirus B19 is due to the presence of virus in the
skin
C. Measles is usually diagnosed by viral culture
D. Herpes simplex virus (HSV) infection is usually diagnosed by
hemagglutination
test
E. VZV cannot be grown in cell culture
1272.
Rubella infection, all are correct, except
A. Can be asymptomatic
B. May be indistinguishable from parvovirus B19
C. *Can have serious side effects when occurring in a woman in the third
trimester
of pregnancy
D. Is usually preventable by vaccination
E. May be acquired by having close contact with an infant with congenital
rubella
syndrome
1273.
The following are DNA viruses
A. Influenza A virus
B. Rubella Virus
C. *Herpesviruses
D. Hepatitis A Virus
E. Rabies Virus
1274.
The following are positively stranded RNA viruses
A. *Picornaviruses
B. Orthomyxoviruses
1275.
1276.
1277.
1278.
1279.
1280.
1281.
1282.
1283.
C. Paramyxoviruses
D. Bunyaviruses
E. Rhabdoviruses
The following viruses have segmented genomes, except
A. Orthomyxoviruses
B. *Herpesviruses
C. Arenaviruses
D. Bunyaviruses
E. Rotaviruses
The following are enveloped viruses
A. Picornaviruses
B. Adenoviruses
C. Rotaviruses
D. Parvoviruses
E. *Orthomyxoviruses
All viruses
A. Contain DNA
B. Contain RNA
C. Contain hemagglutinin
D. Are susceptible to lipid solvents
E. *Are intracellular parasites
The following viruses may cause congenital infection
A. HSV-2
B. Rubella
C. HIV
D. CMV
E. *All are correct
The following viruses have reverse transcriptase, except
A. HIV
B. HTLV-1
C. HBV
D. *CMV
E. HTLV-2
The following viruses replicate in the nucleus
A. *Herpesviruses
B. Poxviruses
C. Picornaviruses
D. Rhabdoviruses
E. Paramyxoviruses
The following are is not viral zoonoses
A. *Rubella
B. Rabies
C. Hantaviruses
D. Lassa Fever
E. Japanese Encephalitis
Reassortment of genes may occur with
A. HSV-1
B. HBV
C. HIV
D. *Influenza A Virus
E. Adenoviruses
The following statements are true of prions
A.
B.
C.
D.
E.
1284.
A.
B.
C.
D.
E.
1285.
A.
B.
C.
D.
E.
1286.
A.
B.
C.
D.
E.
1287.
A.
B.
C.
D.
E.
1288.
A.
B.
C.
D.
E.
1289.
A.
B.
C.
D.
E.
1290.
A.
B.
C.
D.
E.
1291.
A.
B.
C.
D.
Readily inactivated by autoclave at 121 oC
Contain RNA
*Have long incubation periods
Highly immunogenic
May be readily cultured
The following are "slow virus" diseases, except
Tick-born Encephalitis
Creutzfeldt-Jacob disease
Subacute Scelerosing Panecephalitis (SSPE)
*Rabies
Progressive multifocal leucoencephalopathy (PML)
HSV-1 infection may result in all, except
Encephalitis
Gingivostomatitis
Genital Herpes
Corneal Ulcers
*Shingles
The following viruses are associated with a vesicular rash
Rubella Virus
Measles Virus
Parvovirus
*Varicella Zoster virus
Rubella, measles, and parvoviruses cause a maculopapular rash
Cytomegalovirus (CMV)
Primary infection is usually asymptomatic
An infectious mononucleosis-like syndrome may occur during primary infection
May cause severe infection in immunocompromised individuals
May cause congenital infection
*All are correct
HHV-8 is associated with
Fifth disease
Roseala Infantum
*Kaposi's Sarcoma
Nasopharyngeal Carcinoma
Oral leukoplakia
The following statements are true
*Influenza A may undergo antigenic shift
Influenza B may undergo antigenic shift
Influenza C may undergo antigenic drift
Influenza C may respond to amantidine
Influenza B may respond to amantidine
Respiratory Syncytial Virus
Cause disease mainly in adults
*May cause bronchiolitis
May be prevented by vaccination
May be treated by amantidine
May cause latent infections
HIV may respond to
Nucleoside analogues
Protease inhibitors
Azidothymidin
Reverse transcriptase inhibitors
E. *All are correct
1292.
Hepatitis A infection
A. May result in chronic infection
B. May be prevented by acyclovir
C. *May be prevented by vaccination
D. Is highly infectious during the jaundice phase
E. May result in cirrhosis
1293.
Hepatitis B infection
A. May result in chronic infection
B. May result in cirrhosis of the liver
C. May result in hepatocellular carcinoma
D. Is highly infectious when positive for HBeAg
E. *All are correct
1294.
Choose incorrect statement about Hepatitis C virus
A. May be transmitted by blood
B. Is associated with hepatocellular carcinoma
C. May respond to interferon therapy
D. *Has one stable genotype only
E. Can cause chronic infection
1.
2.
3.
4.
Situation tasks
A 20 y.o. girl complains about genital discharge. At the microscopic
examination of this
discharge a lot of leucocytes and coffe-shaped diplococci into of them are
found. What
these microorganisms are?
A. Staphylococci
B. Pneumococci
C. Meningococci
D. Enterococci
E. *Gonococci
A 30 y.o. woman complains about genital discharge. By the microscopic
examination of
dischages a lot of leucocytes and been-shaped diplococci into them were
found. What
these microorganisms are?
A. Staphylococci
B. Pneumococci
C. Meningococci
D. Enterococci
E. *Gonococci
A 9 y.o. boy has got acute osteomyelites of right leg. In laboratory by the
bacteriological
examination staphylococci are found. What morphological properties of this
family?
A. look as a tetrads
B. look as a chain
C. look as a single coccus
D. look as pairs
E. *look as a grape
A boy, 12 years old, is in a hospital with suspicion on food poisoning. After
inoculation
of patient’s feces on the Endo medium colourless colonies grew. There were
gramnegative, small, straight rods. What these microbes are?
A. Vibrio cholerae
B. Staphylococcus
C. Streptococcus
D. Meningococcus
E. *Proteus
5. A bacteriologist isolated and then identified the pathogenic microbes from
patient’s
material. It was Streptococcus pyogenes. What morphological and tinctorial
properties
do these microbes have?
A. Gram-positive cocci with single appearance in smear
B. Gram-positive diplococci
C. Gram-positive cocci looked as a grape
D. Gram-positive rods
E. *Gram-positive cocci in chains
6. A bacteriologist found in a material from a skin wound plenty of
leucocytes and
irregular groups of gram-positive cocci under the microscope. Name these
microorganisms.
A. Tetracocci
B. Streptococcus
C. Gonococcus
D. Meningococcus
E. *Staphylococcus
7. A new-born child has a scalded fever. After the bacteriologic examination
of a pus
staphylococci are found. What morphological and tinctorial properties do
these
microbes have?
A. Gram-negative cocci
B. Gram-negative rods
C. Gram-positive rods
D. Gram-negative spiral bacteria
E. *Gram-positive cocci
8. A 30 y.o. woman complains about genital discharge. At the microscopic
examination of
discharge a lot of leucocytes and been-shaped diplococci into them are found.
What
these microorganisms are?
A. Staphylococci
B. Pneumococci
C. Meningococci
D. Enterococci
E. *Gonococci
9. A newborn has got blenorrhea. By the microscopic examination of patient’s
material
leucocytes and been-shaped diplococci into them were found. What these
microorganisms are?
A. Staphylococci
B. Pneumococci
C. Meningococci
D. Enterococci
E. *Gonococci
10. A patient D., 8 y.o. has got acute respiratory infection and a rash on a
skin. From the
skin rash dark blue diplococci stained by simple method are found. What is
the
causetive agent of disease?
A. Staphylococci
B. Monococci
C. Pneumococci
D. Streptococci
E. *Meningococci
11. A patient complains about a chill and thorax pain. A previous diagnosis
is pneumonia.
By microscopy of sputum leucocytes and staphylococcus were found. What is
morphology of these microbes?
A. in a smear look like pair
B. single gram-negative rods
C. sporulate, does not perceive aniline stain
D. gram-positive, chains
E. *gram-positive, groups
12. A patient complains about attack-like stomach-aches, frequent liquid
stool with the
admixtures of mucus and blood. He is ill second day but does not accept any
medicines.
The culture of shigella is isolated. What characteristics these
microorganisms have?
A. in a smear look like pair
B. sporulate
C. gram-positive, in a smear look like chains
D. gram-positive, in a smear look like groups
E. *single gram-negative rods
13. A patient complains on pain in a thorax, cough. In the sputum of patient
lancet-shaped
gram-positive diplococci, were found. What microorganisms were found?
A. Staphylococcus aureus.
B. Klebsiella pneumoniae.
C. Neisseria meningitidis.
D. Streptococcus pyogenes.
E. *Streptococcus pneumoniae.
14. A patient has got urethritis. Gram-positive microorganisms in pair, short
chains or as
clasters were found. What these microorganisms are?
A. Pneumococci
B. Staphylococci
C. Micrococci
D. Spiral
E. *Enterococci
15. A woman complains about festerings discharge from genitals, pain in the
bottom of
stomach. She is ill the second week. A bacteriologic examination found out
plenty of
leucocytes and been-shaped diplococci which were into leucocytes. What these
microorganisms are?
A. Neisseria flava
B. Neisseria sicca
C. Neisseria meningitides
D. Neisseria subflava
E. *Neisseria gonorrhoeae
16. After a sanitary bacteriological examination Escherichia coli is
isolated. What
morphological and tinctorial properties do this microorganism have?
A. Gram-negative cocci
B. Gram-positive cocci
C. Gram-positive rods
D. Gram-negative spiral bacteria
E. *Gram-negative rods
17. Patient’s diagnosis is furunclosis of the left forearm. In a pus were
found spherical in
cluster. What these microorganisms are?
A. Diplococci.
B. Micrococci.
C. Streptococci.
D. Tetracocci.
E. *Staphylococci.
18. After inoculation of patient’s feces on the Endo medium overnight dark
red colonies
with metallic blisten grew. What morphological and tinctorial properties does
Escherichia coli have?
A. Gram-negative cocci
B. Gram-positive cocci
C. Gram-positive rods
D. Gram-negative spiral bacteria
E. *Gram-negative rods
19. After operative interference concerning acute appendicitis, at patient,
22 years old,
festering complication of operating wound developed. From patient’s material
a smear
was prepared. By a microscopic examination bacteriologist found out
leucocytes and
grape-shaped cocci. What family these microorganisms belong to?
A. Planococcus
B. Enterococcus
C. Streptococcus
D. Micrococcus
E. *Staphylococcus
20. By the bacteriological examination of kindergarten workers bacteria of
Staphylococcus
family were isolated from one of them. What morphological and tinctorial
properties do
these microbes have?
A. tetrads of Gram-positive cocci
B. Gram-positive diplococci
C. Gram-positive cocci in chains
D. Gram-positive rods
E. grape-shaped Gram-positive cocci
21. After the conducted surgical operation a patient has got a sepsis as
complication which
caused by staphylococci. Which of following properties have staphylococci?
A. in a smear look like pair
B. single gram-negative rods
C. sporulate, does not aniline stained
D. gram-positive, in chains
E. *gram-positive, in irregular groups
22. By the XR examination of woman, 55 years old, in lungs numeral abscesses
were found.
By microscopic examination of the sputum smear irregular groups of grampositive
spherical bacteria were identified. What microorganisms were isolated?
A. Streptococcus pyogenes
B. Streptococcus pneumoniae
C. Streptococcus agalactiae
D. Escherichia coli
E. *Staphylococcus aureus
23. At darkfield microscopic examination of material from the surface of the
pudendal
labium ulcer of patient T., 18 y.o, is found thin spiral microorganisms. What
these
microorganisms are?
A. Treponema refringens
B. Treponema phagedenis
C. Treponema denticola
D. *Treponema pallidum
E. Treponema dentium
24. At the bacteriological examination of healthy man from an oral cavity the
culture of
gram-positive diplococci with a capsule were isolated. The carrier of what
pathogenic
microorganism is this man?
A. Legionelia pneumoniae
B. Klebsiella pneumoniae
C. Streptococcus pyogenes
D. Streptococcus agalactiae
E. *Streptococcus pneumoniae
25. Child, 5 years old, has a furuncle on the left shoulder. The
bacteriologic examination of
pus from furuncle staphylococci are found. What morphological properties of
these
microbes?
A. in a smear look like pair
B. single gram-negative rods
C. sporulate, does not perceive aniline stain
D. gram-positive, chains
E. *gram-positive, groups
26. At microscopic examination of a patient T. an ulcer on a pudendal labium
spiral shaped
microbes are found. What is the method to prove lues?
A. electronic microscopy
B. Gram staining
C. methylene blue staining
D. all is correct
E. *darkfield microscopy
27. After 6 hours of a patient’s feces cultivation in the 1% alkaline peptone
water a culture
of microbes is isolated. They are mobile gram-negative, small, coma-shaped
rods. What
these microbes are ?
A. Pseudomonas spp.
B. Escherichia coli
C. Shigella spp.
D. Salmonella spp.
E. *Vibrio cholerae
28. Boy, 7 years old, has vomiting, acute diarrhoea. The previous diagnosis
was cholera.
But after inoculation of feces on Endo medium dark red colonies with metallic
shine
grow. Under the microscope they are gram-negative, small, straight rods. What
these
microbes are?
A. Vibrio cholerae
B. Pseudomonas spp.
C. Shigella spp.
D. Salmonella spp.
E. *Escherichia coli
29. In a maternity hospital an examination is conducted on an in-hospital
infection.
Staphylococcus aureus was found on some objects. What properties belong to
this
microbe?
A. in a smear look like pair
B. single gram-negative rods
C. sporulate, does not perceive aniline stain
D. gram-positive, chains
E. *gram-positive, groups
30. In a school some cases of food poisoning are registered. The examination
of stuff and
students is conducted. Gram positive microorganisms which are located in
pair, short
chains or as clasters are found. What these microorganisms are?
A. Pneumococci
B. Staphylococcus
C. Micrococci
D. Spiral
E. *Enterococci
31. On the top of fever patient’s blood is taken for microscopic examination.
Blood smear
was prepared and stained by Romanovsky-Giemsa method. At the microscopy dark
blue
thin microorganisms with 3-4 large curves were found among blood cells. What
these
microorganisms are?
A. Treponema pallidum
B. Leptospira interrogans
C. Borrelia duttoni
D. Borrelia burgdorferi
E. *Borrelia recurrentis
32. Patient F., 20 y.o. has got furunculosis. The laboratory diagnostics of
festerings
discharge found out staphylococci. Which properties they have?
A. in a smear look in pair
B. single gram-negative rods
C. sporulate, does not perceive aniline stain
D. gram-positive in chains
E. *gram-positive in irregular groups
33. Patient P., 25 y.o. has got furunculosis. The laboratory diagnostics of
festerings
discharge found out staphylococcus. Which properties they have?
A. in a smear look in pair
B. single gram-negative rods
C. sporulate, does not perceive aniline stain
D. gram-positive in chains
E. *gram-positive in irregular groups
34. The 20-years-old student of college appealed to the hospital with
complaints about
dysuria. Recently she began to live by sexual life. After inoculation of
urine the pure
culture of Escherichia coli is isolated. What properties do these
microorganisms have?
A. Gram-negative cocci
B. Gram-positive cocci
C. Gram-positive rods
D. Gram-negative spiral bacteria
E. *Gram-negative rods
35. The 60-years-old man got an urinary catheter. An infectious process in
the urinary
system, caused by gram-negative rods, developed. What is the pathogen causing
this
disease?
A. Vibrio cholerae
B. Staphylococcus
C. Streptococci
D. Gonococci
E. *Escherichia coli
36. The bacteriologic examination of patient’s material taken from a girl
with a scarlet fever
is performed. Gram-positive spherical microorganisms were isolated. What
these
microbes are?
A. Vibrio cholerae
B. Staphylococcus
C. Proteus
D. Meningococcus
E. *Streptococcus
37. The first cases of acute intestinal disease, which was accompanied a
frequent liquid
stool (to 25 times per days) which reminded a "rice-water", vomiting,
dehydration of
organism, are registered in port town. The pure culture of microorganisms is
isolated.
There were gram-negative, small, slightly cuvered rods, mobile. What these
microbes
are?
A. Pseudomonas spp.
B. Escherichia coli
C. Shigella spp.
D. Salmonella spp.
E. *Vibrio cholerae
38. The mother of one-month child complains about festerings discharge from
the
conjunctiva of child’s eyes. Plenty of leucocytes and coffe-shaped diplococci
which
were into leucocytes discovered at the microscopy of discharge. What these
microorganisms are?
A. Neisseria flava
B. Neisseria sicca
C. Neisseria meningitides
D. Neisseria subflava
E. *Neisseria gonorrhoeae
39. Vomiting, diarrhoea, high temperature appeared for a 1,5-years-old child.
Feces were
inoculated on the Endo medium. At a microscopy found out gram-negative rods.
What
is the pathogen causing this disease?
A. Vibrio cholerae
B. Staphylococcus
C. Streptococci
D. Gonococci
E. *Escherichia coli
40. A 23-year-old medical student had headache and fever one evening. By the
next
morning she had become very ill, with high fever, severe headache, and stiff
neck. She
was admitted to the hospital. Later that day she developed rash, at first
petechial and
then pupuric. Gram stain of CSF showed many white cells and Gram-negative
cocci,
many in pairs. Which organism is most likely to be the cause of her
infection?
A. Escherichia coli.
B. Haemophilus influenzae.
C. Streptococcus agalactiae.
D. Streptococcus pneumoniae.
E. *Neisseria meningitidis.
41. A 23-year-old medical student had headache and fever one evening. By the
next
morning she had become very ill, with high fever, severe headache, and stiff
neck. She
was admitted to the hospital. Later that day she developed rash, first
petechial and then
pupuric. Her white count was 26,000/ull, with 80% neutrophils and 10%
'bands'. Gram
stain of CSF showed many white cells and numerous bacteria. If the pathogen
is
Neisseria meningitidis, how would you expect it to appear on Gram stain?
A. Gram-positive cocci, many in pairs.
B. Gram-positive cocci, many in clusters.
C. Isolated Gram-positive cocci.
D. Gram-negative cocci, many in clusters.
E. *Gram-negative cocci, many in pairs.
42. A 25-year-old man complains of a urethral discharge. You perform a Gram
stain on a
specimen of the discharge and see neutrophils but no bacteria. Of the
organisms listed,
the one MOST likely to cause the discharge is
A. Treponema pallidum
B. Candida albicans
C. Coxiella burnetii
D. All are true
E. *Chlamydia trachomatis
43. A doctor has to make laboratory diagnosis of syphilis in patient F., 26
years old.
Appearance of a hard chancre is characteristic of:
A. secondary syphilis.
B. latent syphilis.
C. tertiary syphilis.
D. complication of syphilis
E. *primary syphilis.
44. A doctor isolated spore-forming bacteria from the soil. Microorganisms,
which produce
spores and diameter of spores is less then diameter of microbial cell, are
called:
A. Clostridia
B. Sarcina
C. Bacteria
D. Vibrio
E. *Bacilli
45. A doctor wants to utilize Gram’s staining for examination of
Mycobacteria. But acid
fast microbes are resistante to acid because they contain in cell wall:
A. lipopolysaccharides
B. polyphosphates
C. acetylglucosamine
D. diaminopimelic acid
E. waxes, fatty acid
46. A student has to compare presence of different chemicals in bacterial
cell wall. The
Gram-positive and Gram-negative cell walls generally differ in that the Grampositives
exclusively possess:
A. carbohydrates
B. peptides
C. lipid
D. none of the above
E. *teichoic acids
47. A student has to point some important properties of bacteria. Each of the
following
statements concerning the surface structures of bacteria is correct EXCEPT:
A. Pili mediate the interaction of bacteria with mucosal epithelium
B. Polysaccharide capsules retard phagocytosis
C. Both gram-negative rods and cocci have lipopolysaccharide ("endotoxin") in
their cell wall
D. None of the above
E. *Bacterial flagella are nonantigenic in humans because they closely
resemble
human flagella in chemical composition
48. A student stained Mycobacteria by Ziehl-Neelsen technique. What color has
Mycobacterim tuberculosis after staining by Ziehl-Neelsen technique
A. blue
B. violet
C. brown
D. green
E. *red
49. A tetanus was diagnosed in patient D. Bacterioscopic examination of
tested material
revealed the presence of spore-forming microbes. The diameter of the spores
was more
then diameter of cells. What microbes are there in the smear?
A. Vibrio
B. Bacilli
C. Helicobacter
D. Bacteria
E. *Clostridia
50. Anthrax was diagnosed in patient S. What is the causative agent of this
disease?
A. Borrelia recurrentis
B. Treponema pallidum
C. Sarcina ventriculi
D. Staphylococcus aureus
E. *Bacillus anthracis
51. Anthrax was diagnosed in patient V. What morphological type of bacteria
do these
microbes belong to?
A. Streptococci
B. Streptobacteria
C. Bacteria
D. Staphylococci
E. *Streptobacilli
52. From a patient with pneumonia capsule-producing microbes were isolated.
Show the
genes of microorganisms wich can have a capsule constantly?
A. shigella
B. salmonella
C. brucella
D. vibrio
E. *klebsiella
53. From child’s sputum causative agent of pertussis was found. It does not
belong spiralshaped bacteria. Find it.
A. Causative agent of cholera
B. Causative agent of leptospirosis
C. Causative agent of borreliosis
D. Causative agent of sodoku disease
E. *Causative agent of whooping cough
54. In the child S. of 2 years old mycotic stomatitis was diagnosed. Specify
the causative
agent of this disease.
A. Staphylococci
B. Streptococci
C. Sarcina
D. Treponema
E. *Candida
55. For checking the quality of filters in an experiment a fluid containing
test-microbe was
filtered. Later a filter was laid on the surface of nutrient medium and put
into the
incubator. How much time is it necessary to hold it on medium to give a final
answer
about its quality?
A. at an optimum temperature, 1 day
B. optimum temperature, 2 days
C. optimum temperature, 3 days
D. optimum temperature, 4 days
E. *at an optimum temperature, 5 day
56. In a hospital chief of department decided to check the quality of
instruments sterilization
in an autoclave by biological method. What bacteria can be used for this
test?
A. Pathogenic
B. Capsule-forming
C. Acid fast
D. Thermophilic
E. Spore-forming
57. In a laboratory for acceleration of sterilization of media with sugar by
live steam this
procedure was made in one day: in the morning, in the day-time and in the
evening for
30 min. How it was necessary to sterilize these media correctly?
A. To sterilize 1 hour
B. To sterilize 15 minutes
C. To sterilize 45 minutes
D. To sterilize twice for a days
E. To sterilize three times with 24 hour interval
58. In an autoclave at 112 °С nutrient media were sterilized. What testmicrobe is it
possible to utilize for sterilization control?
A. Bacillus cereus
B. Bacillus subtilis
C. Clostridium perfringens
D. Clostridium difficile
E. *Clostridium sporogenes
59. Material from a patient with suspicion on dysenterywas inoculated on Endo
medium.
What colour does colony of lactose-negative Shigella have?
A. dark blue
B. green
C. red with metallic hue
D. brown
E. *colourless
60. From patient with gas anaerobic infection it is necessary to select the
culture of
causative agents. What medium can be chosen?
A. Leffler medium
B. Petrov medium
C. Makkoy-Chepin medium
D. Petranyani medium
E. *Kitt-Tarozzi medium
61. From patient’s blood the culture Salmonella typhi was isolated. What
cultural properties
characterize this microbe?
A. Formation of red colonies with metallic hue on Endo medium
B. Formation of colourless colonies on Bismuth-sulfite agar
C. Formation of haemolytic zone in a blood agar
D. Formation of tender pellicle on alkaline peptone water
E. *Formation of colourless or pinky colonies on Endo and Ploskirev nutrient
media
62. In a bacteriological laboratory it is necessary to nutrient media, which
contain matters,
changing at a temperature higher then 100 °С (urea, carbohydrates, proteins).
What
method of sterilization can you offer?
A. autoclave, pressed steam
B. boiling
C. tindalization
D. pasteurization
E. *by steam
63. In a hospital chief of department decided to check the quality of
instruments sterilization
in an autoclave by biological method. What bacteria can be used for this
test?
A. Pathogenic
B. Capsule-forming
C. Acid fast
D. Thermophilic
E. *Spore-forming
64. In a laboratory for acceleration of sterilization of media with sugar by
live steam this
procedure was made in one day: in the morning, in the day-time and in the
evening for
30 min. How it was necessary to sterilize these media correctly?
A. To sterilize 1 hour
B. To sterilize 15 minutes
C. To sterilize 45 minutes
D. To sterilize twice for a days
E. *To sterilize three times with 24 hour interval
65. Material from a patient with suspicion on typhoid fever was inoculated on
Endo
medium. What colour do colony of lactosonegative Salmonella typhi have?
A. dark blue
B. red with metallic hue
C. brown
D. green
E. *colourless
66. Material from a patient with dysentery was inoculated on Endo medium.
What colour do
colony of lactose-negative Shigella have?
A. dark blue
B. green
C. red with metallic hue
D. brown
E. *colourless
67. Material from a patient with an intestinal infection was inoculated on
the Ploskirev
medium. What colour do colonies of Escherichia coli have?
A. colourless
B. brown
C. green
D. dark blue
E. *red
68. Material from a patient with shigellosis (dysentery) was inoculated on
the Ploskirev
medium. What colour do colonies of lactose-negative Shigella have?
A. dark blue
B. green
C. red with metallic hue
D. brown
E. *colourless
69. Material from a sick woman must be inoculated on selective nutrient
medium for
diagnosis of vaginal candidiasis. Indicate this medium?
A. Mueller medium
B. 1 % alkaline peptone water
C. Ru medium
D. Blood agar
E. *Saburo medium
70. Sample of a sick woman must be inoculated on selective nutrient medium
for diagnosis
of vaginal candidiasis. Indicate this medium.
A. Mueller medium
B. 1 % alkaline peptone water
C. Ru medium
D. Blood agar
E. *Saburo medium
71. Material from patient with typhoid fever must be inoculated on selective
nutrient
medium. Choose most optimum from them:
A.
B.
C.
D.
E.
1.
2.
3.
4.
5.
6.
7.
8.
1 % alkaline peptone water
Ru medium
Blood agar
Saburo medium
*Mueller medium
Test’s questions to figures
Who is there in this figure? Fig. 1
A. Hans Jansen
B. Zacharian Jansen
C. *Antony van Leeuwenhoek
D. E. Duclaux
E. Louis Pasteur
Who is there in this figure? Fig. 2
A. Zacharian Jansen
B. *Louis Pasteur
C. E. Duclaux
D. Edward Jenner
E. Robert Koch
Who is there in this figure? Fig. 3
A. Zacharian Jansen
B. Louis Pasteur
C. E. Duclaux
D. Edward Jenner
E. *Robert Koch
Who is there in this figure? Fig. 4
A. *I. Metchnikov
B. Louis Pasteur
C. E. Duclaux
D. Edward Jenner
E. Robert Koch
Who is there in this figure? Fig. 5
A. I. Metchnikov
B. Louis Pasteur
C. E. Duclaux
D. *D. Ivanovsky
E. Robert Koch
What microbes are there in figure? Fig. 6
A. *Micrococci
B. Staphylococci
C. Streptococci
D. Vibrios
E. Pneumococci
What microbes are there? Fig. 7
A. Micrococci
B. *Diplococci
C. Staphylococci
D. Streptococci
E. Tetracocci
What microbes are there? Fig. 8
A. Micrococci
B. Diplococci
C. *Staphylococci
D. Streptococci
E. Tetracocci
9. What microbes are there? Fig. 9
A. Micrococci
B. Diplococci
C. Staphylococci
D. *Streptococci
E. Tetracocci
10. What microbes are there? Fig. 10
A. Micrococci
B. *Sarcina
C. Staphylococci
D. Streptococci
E. Tetracocci
11. What microbes are there? Fig. 11
A. *Monobacteria
B. Monobacilli
C. Diplobacteria
D. Streptobacteria
E. Streptobacilli
12. What microbes are there? Fig. 12
A. Streptobacilli
B. Streptobacteria
C. Diplobacteria
D. *Monobacilli
E. Monobacteria
13. What microbes are there ? Fig. 13
A. Streptobacilli
B. *Streptobacteria
C. Diplobacteria
D. Monobacilli
E. Monobacteria
14. What microbes are there ? Fig. 14
A. *Streptobacilli
B. Streptobacteria
C. Diplobacteria
D. Monobacilli
E. Monobacteria
15. What microbes are there ? Fig. 15
A. Streptobacteria
B. *Streptobacilli
C. Diplobacteria
D. Monobacilli
E. Monobacteria
16. What microbes are there ? Fig. 16
A. Treponema
B. Borrelia
C. Leptospira
D. Spirilla
E. *Vibrio
17. What microbes are there ? Fig. 17
A. Treponema
B. Borrelia
C. Leptospira
D. *Spirilla
E. Vibrio
18. What microbes are there ? Fig. 18
A. Treponema
B. *Borrelia
C. Leptospira
D. Spirilla
E. Vibrio
19. What microbes are there ? Fig. 19
A. Treponema
B. Borrelia
C. *Leptospira
D. Spirilla
E. Vibrio
20. What microbes are there ? Fig. 20
A. *Treponema
B. Borrelia
C. Leptospira
D. Spirilla
E. Vibrio
21. There is the staining of volutin granules according to: Fig. 21
A. Grey’s technique
B. Ziehl-Neelsen’s technique
C. *Loeffler’s technique
D. Peshkov’s technique
E. Gram’s technique
22. There is the structure of: Fig. 22
A. Gram-negative cell wall
B. *Gram-positive cell wall
C. Inner membrane
D. Correct all
E. No correct answer
23. There are bacteria with: Fig. 24
A. Nucleoids
B. *Capsules
C. Spores
D. Volutine granules
E. No correct answer
24. What microbes according to flagella localization are marked by the letter
(a)? Fig. 26
A. *Monotrichous
B. Lophotrichous
C. Amphitrichous
D. Peritrichous
E. No correct answer
25. What microbes according to flagella localization are marked
(b)? Fig. 26
A. Monotrichous
B. *Lophotrichous
C. Amphitrichous
D. Peritrichous
E. No correct answer
26. What microbes according to flagella localization are marked
(c)? Fig. 26
A. Monotrichous
B. Lophotrichous
C. *Amphitrichous
D. Peritrichous
E. No correct answer
27. What microbes according to flagella localization are marked
(d)? Fig. 26
A. Monotrichous
B. Lophotrichous
C. Amphitrichous
D. *Peritrichous
E. No correct answer
28. What is there in the figure? Fig. 27
A. *Bacterial spore
B. Bacterial mesosome
C. Bacterial capsules
D. Correct all
E. No correct answer
29. What type of the spore localization is in this figure? Fig.
A. *Central
B. Terminal
C. Subterminal
D. Correct all
E. No correct answer
30. What type of the spore localization is in this figure? Fig.
A. Central
B. *Terminal
C. Subterminal
D. Correct all
E. No correct answer
31. What type of the spore localization is in this figure? Fig.
A. Central
B. Terminal
C. *Subterminal
D. Correct all
E. No correct answer
32. There is such spore localization: Fig. 31
A. Terminal
B. *Subterminal
C. Central
D. Correct all
E. No correct answer
33. What microbes are there? Fig. 32
A. Treponema
by the letter
by the letter
by the letter
15
29
30
B. Borrelia
C. Leptospira
D. *Actinomycetes
E. Staphylococci
34. What microbes are there in the cell? Fig. 33
A. Treponema
B. Staphylococcus
C. Leptospira
D. Actinomycetes
E. *Chlamidia
35. What microbes are there? Fig. 34
A. Staphylococci
B. Streptococci
C. Sarcina
D. *Candida
E. Micrococci
36. What fungi are there in figure? Fig. 35
A. Aspergillus
B. Penicillium
C. *Mucor
D. Trichophyton
E. Microsporum
37. What fungi are there in figure? Fig. 36
A. *Aspergillus
B. Penicillium
C. Mucor
D. Trichophyton
E. Microsporum
38. What fungi are there in figure? Fig. 37
A. Penicillium
B. *Aspergillus
C. Mucor
D. Trichophyton
E. Microsporum
39. What fungi are there in figure? Fig. 38
A. Aspergillus
B. Mucor
C. *Penicillium
D. Trichophyton
E. Microsporum
40. What equipment is shown in this figure? Fig. 39
A. autoclave
B. serum coagulator
C. thermostat
D. sterilizer
E. *heat oven
41. What equipment is shown in this figure? Fig. 40
A. *autoclave
B. serum coagulator
C. thermostat
D. sterilizer
E. dry-heat closet
42. That equipment is shown in figure? Fig. 41
A. autoclave
B. *serum coagulator
C. thermostat
D. sterilizer
E. dry-heat closet
43. For what purpose can we use this device? Fig. 42
A. *for sterilization by filtration
B. for study of bacterial adhesive properties
C. for sterilization by gases
D. for sterilization by live steam
E. for control of sterilization by pressed steam
44. For what purpose can this test-system be used? Fig. 43
A. for in a heat oven sterilization control
B. *for control of sterilization by autoclaving
C. for mechanical sterilization control
D. for gas sterilization control
E. for control of ultraviolet rays sterilization
45. What method is shown in the figure? Fig. 44
A. inoculation by a spatula
B. *inoculation by streaks technique
C. by serial dilution in solid nutrient media
D. by a tampon
E. none of offered
46. How can we use this test-system? Fig. 45
A. for verification of sterilization
B. for identification of bacteria
C. for phage typing
D. for bacteria staining
E. *for creation of anaerobic condition
47. How can we use this jar? Fig. 46
A. for cultivation of aerobic bacteria
B. *for cultivation of anaerobic bacteria
C. for cultivation of facultative anaerobic bacteria
D. for sterilization of Petry’s plate with microbes
E. for cultivation of aerotolerant bacteria
48. Which bacteria properties you can find on this medium? Fig. 47
A. cultural
B. serological
C. sugarlytic
D. biological
E. *hemolytic
49. What sign does show the presence of growth in the left test tube? Fig. 48
A. formation of sediment
B. formation of pellicle
C. *formation of diffuse turbidity
D. growth in the left test tube is absent
E. formation of «stalactites»
50. What property of bacteria is tested on the Endo’s medium? Fig. 13
A. ability of glucose utilization
B. ability of saccharose utilization
C. *ability of lactose utilization
D. ability of maltose utilization
E. ability of mannitol utilization
51. What properties of bacteria are tested on this medium (blood agar)? Fig.
49
A. saccharolytic
B. lipolytic
C. proteolytic
D. *hemolytic
E. reductive
52. There is yolk agar on the picture. What property of bacteria is it
possible verify? Fig. 50
A. proteolytic
B. hemolytic
C. *lecithinase production
D. gelatin hydrolysis
E. saccharolytic
53. There are different stages of microbial population development in liquid
nutrient
medium. Choose the name of the phase that is marked as number 1. Fig. 25
A. phase of negative acceleration
B. Log- phase
C. Stationary
D. Decline
E. *Lag- phase
54. There are different stages of microbial population development in liquid
nutrient
medium. Choose the name of the phase that is marked as number 2. Fig. 25
A. Lag- phase
B. Decline
C. Stationary
D. *Log- phase
E. Phase of negative acceleration
55. There are different stages of microbial population development in liquid
nutrient
medium. Choose the name of the phase that is marked as number 3. Fig. 25
A. *Stationary
B. Log- phase
C. Lag- phase
D. Decline
E. Phase of negative accelerate
56. There are different stages of microbial population development in liquid
nutrient
medium. Choose the name of the phase that is marked as number 4. Fig. 25
A. Lag- phase
B. Log- phase
C. Stationary
D. Phase of negative acceleration
E. *Decline
57. Different types of bacterial colonies are produced on solid nutrient
medium when
dissociation occurs. What type of colonies do these ones belong to? Fig. 51
A. S
B. *R
C. D
D. M
E. P
58. What is presented in the figure? Fig. 57
A. Structure of F factors
B. *Structure of R factors
C. Structure of S factors
D. Structure of T factors
E. No correct answer
59. What is there in the figure? Fig. 58
A. Bacterial cell
B. *Bacteriophage
C. Protozoa
D. Fungus
E. No correct answer
60. There is scheme of: Fig. 59
A. Transformation
B. Transduction
C. *Conjugation
D. All are correct
E. No correct answer
61. What type of recombination is shown in the figure? Fig. 95
A. *Conjugation
B. Transduction
C. Transformation
D. All are correct
E. No correct answer
62. What type of conjugation is shown in the figure? Fig. 60
A. *Mechanism of Hfr x F- Crosses
B. Mechanism of F+ x F- Crosses
C. Mechanism of F′ x F- Crosses
D. All are correct
E. No correct answer
63. What is there in the figure? Fig. 96
A. Antigen structure of bacteria
B. Biochemical properties of bacteria
C. Chromosome cap
D. *Chromosome map
E. No correct answer
64. What type of colonies are there in the figure? Fig. 61
A. *S
B. R
C. D
D. T
E. F
65. What type of colonies are there in the figure? Fig. 62
A. S
B. *R
C. D
D. T
E. F
66. What type of colonies is there in the figure? Fig. 63
A. D
B. F
C. *R
D. S
E. T
67. What scientist is there in the figure? Fig. 64
A. P. Ehrlich
B. I. Metchnikov
C. L. Pasteur
D. R. Koch
E. *A. Fleming
68. What is there in the figure? Fig. 65
A. Examination of antibiotic susceptility
B. *Examination of bacterial antagonism
C. Examination of susceptility to phage
D. All are correct
E. No correct answer
69. What is there in the figure? Fig. 100
A. Bacteria
B. Protozoa
C. *Penicillium
D. Aspergillus
E. No correct answer
70. What is there in the figure? Fig. 66
A. Structure of bacterial inner membrane
B. Structure of bacterial outer membrane
C. Structure of gram-positive bacterial cell wall
D. *Structure of gram-negative bacterial cell wall
E. Structure of bacterial capsule
71. What test is shown in this figure? Fig. 67
A. Examination of antibiotics susceptibility by dilution technique in solid
nutrient
medium
B. Examination of antibiotics susceptibility by dilution technique liquid
nutrient
medium
C. *Examination of antibiotics susceptibility by disc diffusion technique
D. Examination of antibiotics susceptibility by express technique
E. No correct answer
72. What test is shown in this figure? Fig. 97
A. Examination of antibiotics susceptibility by dilution technique in solid
nutrient
medium
B. Examination of antibiotics susceptibility by dilution technique liquid
nutrient
medium
C. Examination of antibiotics susceptibility by express technique
D. *Examination of antibiotics susceptibility by disc diffusion technique
E. No correct answer
73. What test is shown in this figure? Fig. 98
A. Examination of antibiotics susceptibility by dilution technique in solid
nutrient
medium
B. *Examination of antibiotics susceptibility by dilution technique liquid
nutrient
medium
C. Examination of antibiotics susceptibility by express technique
D. Examination of antibiotics susceptibility by disc diffusion technique
E. No correct answer
74. What test is shown in this figure? Fig. 99
A. *Examination of antibiotics susceptibility by dilution technique in solid
nutrient
medium
B. Examination of antibiotics susceptibility by dilution technique liquid
nutrient
medium
C. Examination of antibiotics susceptibility by express technique
D. Examination of antibiotics susceptibility by disc diffusion technique
E. No correct answer
75. What type of immune response is shown in this figure? Fig. 23
A. *Humoral immune response
B. Immediate hypersensitivity
C. Delayed hypersensitivity
D. Cell mediated immune response
E. Cytotoxic rection
76. This is the scheme of the immune response. What cell is marked
1? Fig. 23
A. B-lymphocyte
B. T-killer
C. T-helper
D. *Antigen presenting cell
E. Plasma cell
77. This is the scheme of the immune response. What cell is marked
2? Fig. 23
A. *B-lymphocyte
B. T-killer
C. T-helper
D. Antigen presenting cell
E. Plasma cell
78. This is the scheme of the immune response. What cell is marked
3? Fig. 23
A. B-lymphocyte
B. T-killer
C. T-helper
D. Antigen presenting cell
E. *Plasma cell
79. This is the scheme of the immune response. What cell is marked
4? Fig. 23
A. B-lymphocyte
B. T-killer
C. *T-helper
D. Antigen presenting cell
E. Plasma cell
80. When may be all these symptoms take place? Fig. 28
A. This is the result of the anaphylaxis
B. These symptoms are associated with autoimmune diseases
C. These symptoms are the result of the exotoxin action
D. *These symptoms are the result of the endotoxin action
E. These symptoms belong to the delayed hypersensitivity
81. What process is shown in this figure? Fig. 74
A. Interaction between the B-cell and T-cell
B. Action of the natural killer
C. Activation of T-killer by the macrophage
D. *APC presents antigen for the T-helper
E. T-helper stimulates T-killer
82. What cell is marked by number 1 in this figure? Fig. 74
A. *Antigen-presenting cell
B. B-lymphocyte
C. T-killer
D. T-helper 1
E. NK-cell
83. What cell is marked by number 2 in this figure? Fig. 74
A. Antigen-presenting cell
B. B-lymphocyte
C. T-killer
D. *T-helper 1
E. NK-cell
84. What type of immune response is shown in this figure? Fig. 76
A. Humoral immune response
B. Immediate hypersensitivity
C. Delayed hypersensitivity
D. *Cell mediated immune response
by number
by number
by number
by number
E. Target cell is destroyed by the natural killer
85. This is the cell mediated immune response. What cell is marked by number
1? Fig. 76
A. B-lymphocyte
B. T-killer
C. *T-helper-2
D. NK-cell
E. Dendritic cell
86. This is the cell mediated immune response. What substance released by the
cell is
marked by number 2? Fig. 76
A. IL-1
B. Interferon
C. *Perforine
D. Granzyme
E. TNF
87. This is the cell mediated immune response. What is the mechanism of
target cell killing
marked by number 4? Fig. 76
A. Complement depended cytolysis
B. *Apoptosis
C. Macrophage digests infected cell
D. Antibody dependent cytotoxicity
E. All of the above
88. What type of hypersensitivity is shown in this figure? Fig. 85
A. Immune-complex reaction
B. Immediate hypersensitivity
C. *Delayed hypersensitivity
D. Cell mediated immune response
E. Cytotoxic rection
89. What immunological process is demonstrated in this figure? Fig. 88
A. Cytotoxic, cytolytic reaction
B. Antibody dependent cytotoxicity
C. *Immune-complex reaction
D. Atopic reaction
E. Delayed hypersensitivity
90. This is an example of one type of allergy.What is marked by number 1?
Fig. 88
A. Agglutination reaction
B. Precipitation reaction
C. Formation of the antibody-antigen-complement complex
D. *Immune-complex formation
E. Interaction between the cells and antibodies
91. What process is shown in this figure? Fig. 89
A. Anaphylactic reaction
B. Cell mediated immune response
C. *Desensitisation
D. Humoral immune response
E. Antibody dependent cytotoxicity
92. This is the scheme of the desensitisation. What is marked by number 1?
Fig. 89
A. Antigen-antibody complex
B. Immune complex
C. IgE binds with allergen
D. *blocking antibodies IgG
E. All of the above
93. What cell is marked by number 1? Fig. 91
A. B-cell
B. T-cell
C. T-killer
D. T-helper 1
E. *T-helper 2
94. What cell is marked by number 2? Fig. 91
A. B-cell
B. T-cell
C. T-killer
D. *T-helper 1
E. T-helper 2
95. What structure is marked by number 1? Fig. 92
A. Ribosome
B. mRNA
C. Class I MHC protein
D. *Class II MHC protein
E. Epitope
96. What formation is marked by number 2? Fig. 92
A. Complex of the IgM and antigen
B. Class I MHC protein with the peptide
C. Class I MHC protein
D. Class II MHC protein
E. *Class II MHC protein with the epitope
97. What cell is marked by number 1 in this figure? Fig. 93
A. *Target cell
B. B-lymphocyte
C. T-killer
D. T-helper 1
E. NK-cell
98. What cell is marked by number 2 in this figure? Fig. 93
A. Antigen-presenting cell
B. B-lymphocyte
C. *T-killer
D. T-helper 1
E. NK-cell
99. Where does the fixation of the complement according to the picture take
place? Fig. 86
A. *in the first three test tubes
B. in a 3-6 test tube
C. in 4-8 test tubes
D. in 9-10 test tubes
E. in the last three test tubes
100.
Where does not the fixation of the complement according to the picture take
place? Fig. 86
A. in the first three test tubes
B. in a 3-6 test tube
C. *in 4-8 test tubes
D. in 9-10 test tubes
E. in the last three test tubes
101.
How many times did increase of title of antibodies in CFT according to the
picture? Fig. 87
A.
B.
C.
D.
E.
102.
A.
B.
C.
D.
E.
103.
A.
B.
C.
D.
E.
104.
A.
B.
C.
D.
E.
105.
A.
B.
C.
D.
E.
106.
A.
B.
C.
D.
E.
107.
A.
B.
C.
D.
E.
108.
A.
B.
C.
D.
E.
109.
A.
B.
C.
D.
*in 4 times
in 8 times
in 2 times
in 6 times
growth of title of antibodies did not take a place
In which small wells of the first line of test tubes is CFT positive? Fig. 87
*in the first
in two first
in 2-4 small holes
in 4 small holes
in all of small holes
In which small wells of the first line of test tubes of is CFT negative? Fig.
87
in the first
in two first
*in 2-4 small holes
in all of small holes
in any small hole
What clinical type of allergic reaction is presented on a picture? Fig. 90
*phenomenon of Artyus
anaphylactic shock
hemolytic illness
reaction of tuberculine of type
delayed type
Specify, what letter is marked Ig A? Fig. 75
Letter of A
*Letter of B
Letter of C
Letter of D
Letter of E
Specify, what letter is marked Ig M? Fig. 75
Letter of A
Letter of B
*Letter of C
Letter of D
Letter of E
Specify, what letter is marked Ig G? Fig. 75
*Letter of A
Letter of B
Letter of C
Letter of D
Letter of E
Specify, what letter is marked Ig E? Fig. 75
Letter of A
Letter of B
Letter of C
Letter of D
*Letter of E
What class of immunoglobulin is presented on a picture? Fig. 69
Ig G
*Ig M
Ig D
Ig E
E. Ig A
110.
What class of immunoglobulin is presented on a picture? Fig. 72
A. *Ig G
B. Ig M
C. Ig D
D. Ig E
E. Ig A
111.
How many active centers do have an immunoglobulin presented on a picture?
Fig. 72
A. *2
B. 4
C. 6
D. 8
E. 12
112.
Choose from presented, what is marked by letter A. Fig. 71
A. Active center
B. *Heavy chain
C. Fab- fragment
D. Variable region
E. Fc-fragment
113.
Choose from presented,what is marked by letter B. Fig. 71
A. Active center
B. *Stable part of immunoglobuline
C. Fab- fragment
D. Variable region
E. Fc-fragment
114.
Choose from presented, what is marked by letter C. Fig. 71
A. Active center
B. Stable part of immunoglobuline
C. Fab- fragment
D. Variable region
E. *Fc-fragment
115.
Choose from presented, what is marked by letter D. Fig. 71
A. *Active center
B. Stable part of immunoglobulin
C. Fab- fragment
D. Variable region
E. Fc-fragment
116.
Choose from presented, what is marked by letter E. Fig. 71
A. Active center
B. Stable part of immunoglobulin
C. *Fab- fragment
D. Variable region
E. Fc-fragment
117.
The schematic image of what immunoglobulin is presented? Fig. 70
A. Ig G
B. Ig M
C. Ig D
D. Ig E
E. *Ig A
118.
Choose from presented, what is marked by letter A. Fig. 70
A. Active center
B.
C.
D.
E.
119.
A.
B.
C.
D.
E.
120.
A.
B.
C.
D.
E.
121.
A.
B.
C.
D.
E.
Heavy chain
*J chain
Secretory component
Fc-fragment
Choose from presented, what is marked by letter B. Fig. 70
Active center
Heavy chain
J chain
*Secretory component
Fc-fragment
Choose from presented, what is marked by letter C. Fig. 70
*Active center
Heavy chain
J chain
Secretory component
Fc-fragment
Choose from presented, what is marked by letter D. Fig. 70
Active center
*Heavy chain
J chain
Secretory component
Fc-fragment
What structures of immunoglobulin are presented on a picture under a letter
A.
122.
Fig. 69
A. Active center
B. *Heavy chain
C. J chain
D. Light chain
E. Fc-fragment
123.
What structures of immunoglobulin are presented on a picture under the letter
B.
Fig. 69
A. Active center
B. Heavy chain
C. *J chain
D. Light chain
E. Fc-fragment
124.
What structures of immunoglobulin presented on a picture under the letter C.
Fig. 69
A. *Active center
B. Heavy chain
C. J chain
D. Light chain
E. Fc-fragmen
125.
What structures of immunoglobulin are presented on a picture under the
D.
Fig. 69
A. Active center
B. Heavy chain
C. J chain
D. Light chain
E. *Fc-fragment
126.
What structures of immunoglobulin are presented on a picture under the
E.
Fig. 69
A.
B.
C.
D.
E.
Active center
Heavy chain
J chain
*Light chain
Fc-france
127.
What structural part of immunoglobulin is presented on a picture under
letter
A. Fig. 68
A. Light chain
B. *Variable region
C. Disulphide bonds
D. Complement binding region
E. Receptor for placental passage
128.
What structural part of immunoglobulin is presented on a picture under
letter
B. Fig. 68
A. *Light chain
B. Variable region
C. Disulphide bonds
D. Complement binding region
E. Receptor for placental passage
129.
What structural part of immunoglobulin is presented on a picture under
letter
C. Fig. 68
A. Light chain
B. Variable region
C. *Disulphide bonds
D. Complement binding region
E. Receptor for placental passage
130.
What structural part of immunoglobulin is presented on a picture under
letter
letter
letter
a
the
the
the
D. Fig. 68
A. Light chain
B. Variable region
C. Disulphide bonds
D. *Complement binding region
E. Receptor for placental passage
131.
What structural part of immunoglobulin is presented on a picture under the
letter
E. Fig. 68
A. Light chain
B. Variable region
C. Disulphide bonds
D. Complement binding region
E. *Receptor for placental passage
Download
1.
2.
3.
4.
Science
Biology
Cell Biology
Bacteria
Morphology, physiology of microorganisms.
Virology.doc
Significance of microbiology in doctors practical activity1
Biological diagnostics, control and treatment of the cholerae.
02 Classification and Morphology of Microorganisms
Significance of microbiology in nurses' practice
Special, clinical and ecological microbiology.doc
What groups of methods for creation of anaerobic conditions for
Special bacteriology
Lecture_1_The role of microbiology_Physiology of microorganism
BEYOND COLD WAR POLITICS: - Gettysburg College Alumni Server
Special microbiology
Test eng
Federal Agency for Social Development
Document
Material from a patient with suspicion on colienteritis was inoculated
NTM - WHO Western Pacific Region
Microbial physiology
Study Guide 092004 - American Society for Microbiology
MODULE. 1 GENERAL MICROBIOLOGY, VIROLOGY AND
NRM Study Guide - American Society for Microbiology
AliKat Productions Capabilities Deck
Shigella_Vibrio
Notes on Bacteria
studylib © 2017
DMCA Report